Вы находитесь на странице: 1из 94

Cap tulo 8 T opicos de Algebra Linear.

I
Conte udo
8.1 8.2 Propriedades B asicas de Determinantes e Inversas de Matrizes . . . . . . No c oes B asicas sobre o Espectro de uma Matriz . . . . . . . . . . . . . . . 8.2.1 Autovalores e Polin omios Caracter sticos de Matrizes . . . . . . . . . . . . . 8.2.2 Autovetores . . . . . . . . . . . . . . . . . . . . . . . . . . . . . . . . . . . . 8.2.3 O Tra co de uma Matriz . . . . . . . . . . . . . . . . . . . . . . . . . . . . . . 8.2.3.1 Algumas Rela c oes entre Determinantes e Tra cos de Matrizes . . . . . Polin omios de Matrizes . . . . . . . . . . . . . . . . . . . . . . . . . . . . . . 8.3.1 O Teorema de Hamilton-Cayley . . . . . . . . . . . . . . . . . . . . . . . . . Matrizes Diagonaliz aveis e o Teorema Espectral . . . . . . . . . . . . . . . 8.4.1 Diagonaliza c ao Simult anea de Matrizes . . . . . . . . . . . . . . . . . . . . . Matrizes Auto-Adjuntas, Normais e Unit arias . . . . . . . . . . . . . . . . 8.5.1 Matrizes Positivas . . . . . . . . . . . . . . . . . . . . . . . . . . . . . . . . . 8.5.2 O Teorema de In ercia de Sylvester. Superf cies Quadr aticas . . . . . . . . . Matrizes Triangulares . . . . . . . . . . . . . . . . . . . . . . . . . . . . . . . O Teorema de Decomposi c ao de Jordan e a Forma Can onica de Matrizes 8.7.1 Resultados Preparat orios . . . . . . . . . . . . . . . . . . . . . . . . . . . . . 8.7.2 O Teorema da Decomposi c ao de Jordan . . . . . . . . . . . . . . . . . . . . . 8.7.3 Matrizes Nilpotentes e sua Representa c ao Can onica . . . . . . . . . . . . . . 8.7.4 A Forma Can onica de Matrizes . . . . . . . . . . . . . . . . . . . . . . . . . Algumas Representa c oes Especiais de Matrizes . . . . . . . . . . . . . . . . 8.8.1 A Decomposi c ao Polar de Matrizes . . . . . . . . . . . . . . . . . . . . . . . 8.8.2 A Decomposi c ao em Valores Singulares . . . . . . . . . . . . . . . . . . . . . 8.8.3 O Teorema da Triangulariza c ao de Schur . . . . . . . . . . . . . . . . . . . . 8.8.4 A Decomposi c ao QR e a Decomposi c ao de Iwasawa (KAN) . . . . . . . . A Pseudo-Inversa de Moore-Penrose. Optimiza c ao Linear . . . . . . . . . 8.9.1 Outras Propriedades da Pseudo-Inversa de Moore-Penrose . . . . . . . . . . 8.9.1.1 A Regulariza c ao de Tikhonov. Exist encia . . . . . . . . . . . . . . . . 8.9.1.2 A Pseudo-Inversa de Moore-Penrose e o Teorema Espectral . . . . . . 8.9.2 A Pseudo-Inversa de Moore-Penrose e Problemas de Optimiza c ao Linear . . 8.9.3 Exist encia e Decomposi c ao em Valores Singulares . . . . . . . . . . . . . . . Propriedades Especiais de Determinantes . . . . . . . . . . . . . . . . . . . 8.10.1 Expans ao do Polin omio Caracter stico . . . . . . . . . . . . . . . . . . . . . 8.10.2 A Desigualdade de Hadamard . . . . . . . . . . . . . . . . . . . . . . . . . . Exerc cios Adicionais . . . . . . . . . . . . . . . . . . . . . . . . . . . . . . . . . . . . . . . . . . . . . . . . . . . . . . . . . . . . . . . . . . . . . . . . . . . . . . . . . . . . . . . . . . . . . . . . . . . . . . . . . . . . . . . . . . . . . . . . . . . . . . . . . . . . . . . . . . . . . . . . . . . . . . . . . . . . . . . . . . . . . . . . . . . . . . . . . . . . . . . . . . . . . . . . . . . . . . . . . . . . . . . . . . . . . . . . . . . . . . . . . . . . . . . . . . . . . . . . . . . . . . . . . . . . . . . . . . . . . . . . . . . . . . . . . . . . . . . . . . . . . . . . . . . . . . . . . . . . . . . . . . . . . . . . . . . . . . . . . . . . . . . . . . . . . . . . . . 295 305 . 305 . 308 . 310 . 312 313 . 315 319 . 331 334 . 340 . 342 346 349 . 350 . 354 . 357 . 360 363 . 363 . 365 . 365 . 368 370 . 372 . 375 . 377 . 378 . 379 381 . 381 . 382 385

8.3 8.4 8.5

8.6 8.7

8.8

8.9

8.10

8.11

principal objetivo deste cap tulo e apresentar a demonstra c ao do Teorema Espectral para matrizes diagonaliz aveis, em particular, para matrizes auto-adjuntas (resultado de grande relev ancia para a Mec anica Qu antica) e a demonstra ca o do Teorema de Decomposi ca o de Jordan. Sempre trabalharemos no contexto de espa cos vetoriais de dimens ao nita Cn sobre o corpo dos complexos. A leitura deste cap tulo pressup oe que alguns conceitos b asicos de Algebra Linear, tais como o conceito de matriz, de produto de matrizes, de determinante de uma matriz, suas propriedades e m etodos de c alculo, sejam familiares ao leitor, mas uma breve revis ao e apresentada na Se ca o 8.1. Na Se ca o 8.2, p agina 305, apresentamos a no ca o de espectro e a de polin omio caracter stico de uma matriz. Na Se ca o 8.5, p agina 334, introduzimos as no co es de matrizes auto-adjuntas, normais e unit arias, de import ancia, por exemplo, na Mec anica Qu antica. Na Se ca o 8.8, p agina 363, apresentamos algumas representa co es de matrizes de interesse em

294

JCABarata. Curso de F sica-Matem atica

Vers ao de 19 de junho de 2011.

Cap tulo 8

295/1943

diversos contextos (por exemplo, na teoria de grupos). Na Se c ao 8.9, p agina 370, estudamos a chamada pseudo-inversa de Moore-Penrose, de interesse, por exemplo, em problemas de optimiza ca o linear. Este cap tulo ser a continuado no Cap tulo 9, p agina 388, onde outros aspectos de algebras de matrizes ser ao explorados.

8.1

Propriedades B asicas de Determinantes e Inversas de Matrizes

A presente se ca o desenvolve a teoria b asica de inversas e determinantes de matrizes. Sua leitura pode, provavelmente, ser dispensada por aqueles que julgam dispor desses conhecimentos b asicos, mas a nota ca o que aqui introduzimos ser a empregada alhures. Propriedades mais avan cadas de determinantes ser ao estudadas na Se ca o 8.10, p agina 381. Fatos elementares sobre matrizes e alguma nota c ao

O conjunto de todas as matrizes mn (m linhas e n colunas) com entradas complexas ser a denotado por Mat (C, m, n). O conjunto de todas as matrizes quadradas n n com entradas complexas ser a denotado simplesmente por Mat (C, n). Uma matriz A Mat (C, m, n) e freq uentemente representada na forma de um arranjo como A11 . . A = . Am1 A1n . . . . . . Amn ... .

Mat (C, m, n) e um espa co vetorial complexo, com a opera ca o de soma denida por (A1 + A2 )ij := (A1 )ij + (A2 )ij , A1 , A2 Mat (C, m, n), i {1, . . . , m}, j {1, . . . , n}, e a opera ca o de multiplica ca o por escalares (complexos) denida por (A)ij := Aij Sejam m, n, p N e sejam A Mat (C, m, n) e B Mat (C, n, p). Denotamos por AB a matriz de Mat (C, m, p) cujos elementos s ao dados por
n

C, A Mat (C, m, n) e i {1, . . . , m}, j {1, . . . , n}.

AB

ij

:=
k=1

Aik Bkj

(8.1)

f para todos i {1, . . . , m}, j {1, . . . , p}. A express ao (8.1) e denominada regra de produto de matrizes. E acil constatar (fa ca-o!) que valem as propriedades distributivas (1 A1 + 2 A2 )B = 1 A1 B + 2 A2 B , 1 AB1 + 2 AB2 ,

A(1 B1 + 2 B2 ) =

para todos 1 , 2 , 1 , 2 C, todas A, A1 , A2 Mat (C, m, n) e todas B, B1 , B2 Mat (C, n, p). tamb E em f acil constatar (fa ca-o!) que se m, n, p, q N valem para todas A Mat (C, m, n), B Mat (C, n, p) e C Mat (C, p, q ) a rela ca o (AB )C = A(BC ) . Para cada n N, e com a opera ca o de produto denida acima, Mat (C, n) e uma algebra associativa, n ao-comutativa

JCABarata. Curso de F sica-Matem atica

Vers ao de 19 de junho de 2011.

Cap tulo 8

296/1943

(exceto se n = 1) e unital, com a unidade sendo dada pela matriz identidade, que denotaremos por neste texto: 1 . . . .. := . 0 0 . . . . 1 (8.2)

Dada uma matriz A Mat (C, m, n) denotamos por AT a matriz de Mat (C, n, m) cujos elementos s ao dados por evidente (A )ij = Aji para todos i {1, . . . , n}, j {1, . . . , m}. A matriz AT e dita ser a matriz transposta de A. E que (AT )T = A. Para todos m, n, p N vale, pela regra de produto de matrizes, a rela ca o (AB )T = B T AT para quaisquer A Mat (C, m, n) e B Mat (C, n, p).
T

Note-se que ij = ij , i, j {1, . . . , n}.

Dado um conjunto de n n umeros complexos 1 , . . . , n , denotaremos por diag (1 , . . . , n ) a matriz A Mat (C, n) cujos elementos Aij s ao denidos da seguinte forma: i , se i = j . Aij = 0, se i = j Uma tal matriz e dita ser diagonal pois apenas os elementos de sua diagonal principal s ao eventualmente n ao-nulos. Na representa ca o usual 1 . . A = . 0 .. . 0 . . . . n

A mais popular dentre as matrizes diagonais e a matriz identidade (8.2): = diag (1, . . . , 1). Denotaremos por a, b Mat (C, m, n) a matriz a b cujos elementos de matriz s ao todos nulos. Denotaremos por l Mat (C, l) a matriz identidade l l. Por vezes, quando n ao houver perigo de confus ao, poderemos omitir os sub- ndices e escrever a, b simplesmente como e l simplesmente como . Vamos tamb em empregar as seguintes deni co es. Para m, n N, sejam Im, m+n Mat (C, m, m + n) e Jm+n, n Mat (C, m + n, n) dadas por Im, m+n :=

m m, n

cujas transpostas s ao dadas por m (Im, m+n )T :=

n , Jm+n, n :=

(8.3)

m, n

(Jm+n, n )T :=

n, m

n n, m

(8.4)

As seguintes identidades u teis ser ao usadas mais adiante e sua demonstra ca o (f acil) e deixada como exerc cio ao leitor: Im, m+n (Im, m+n )T (Jm+n, n )T Jm+n, n = =

m , n ,

(8.5) (8.6)

JCABarata. Curso de F sica-Matem atica

Vers ao de 19 de junho de 2011.

Cap tulo 8

297/1943

Para cada A Mat (C, m, n) podemos associar uma matriz quadrada A Mat (C, m + n) dada por A A := (Im, m+n )T A(Jm+n, n )T = A = Im, m+n A Jm+n, n . Sejam x1 , . . . , xn vetores, representados na base can onica por vetores-coluna xa xa 1 . . = . . xa n

n, n

m, m n, m

Obtemos das rela co es (8.5)(8.6) que

(8.7)

(8.8)

Denotaremos por x1 , . . . , xn seja

a matriz n n constru da de forma que sua a- esima coluna seja o vetor-coluna xa , ou x1 1 .. . xn 1

x1 , . . . , xn

Considerando os vetores da base can onica

. . = . x1 n

. . . . n xn 0 0 . . = . , 0 1

(8.9)

e1

e tamb em evidente que

1 0 = 0 , . . . 0

e2

0 1 = 0 , . . . 0

...,

en

(8.10)

e1 , . . . , en .

(8.11)

A nota ca o acima eu til por permitir a seguinte observa ca o. Seja B uma matriz qualquer. Ent ao, B x1 , . . . , xn = Bx1 , . . . , Bxn . (8.12)

e Essa rela ca o e provada observando-se a regra de multiplica c ao de matrizes: a a- esima coluna de B x1 , . . . , xn


a B11 xa 1 + + B1n xn

. . .

(8.13)

a Bn1 xa 1 + + Bnn xn

JCABarata. Curso de F sica-Matem atica

Vers ao de 19 de junho de 2011.

Cap tulo 8

298/1943

que vem a ser as componentes de Bxa , representado como vetor-coluna na base can onica. Ainda sobre essa nota ca o, vale a seguinte identidade u til, cuja demonstra ca o (elementar) deixamos como exerc cio: se D = diag (d1 , . . . , dn ) e uma matriz diagonal, ent ao x1 , . . . , xn D = d1 x1 , . . . , dn xn . (8.14)

Se v1 , . . . , vk s ao vetores em um espa co vetorial V , denotamos por [v1 , . . . , vk ] o subespa co gerado pelos vetores v1 , . . . , vk , ou seja, a cole ca o de todos os vetores que s ao combina co es lineares dos vetores v1 , . . . , vk : [v1 , . . . , vk ] = 1 v1 + + k vk , 1 , . . . , k C .

Seja V um espa co vetorial dotado de um produto escalar , . Dizemos que dois vetores u e v s ao perpendiculares (em rela ca o ao produto escalar , ) se u, v = 0.

Denotamos por [v1 , . . . , vk ] o subespa co de todos os vetores perpendiculares a todos os vetores de [v1 , . . . , vk ]: [v1 , . . . , vk ] = wV w, (1 v1 + + k vk ) = 0 para todos 1 , . . . , k C .

Uma matriz A Mat (C, n) dene uma aplica ca o linear de Cn sobre si mesmo. Se essa aplica ca o for bijetora, ent ao 1 existe uma aplica ca o inversa, denotada por A : Cn Cn , tal que A1 Ax = x para todo x Cn . A proposi ca o seguinte reune fatos elementares sobre a aplica ca o inversa A1 : Proposi c ao 8.1 Se A Mat (C, n) e bijetora, ent ao A1 e igualmente uma aplica c ao linear de Cn sobre si mesmo, 1 T ou seja, A1 Mat (C, n). Fora isso, A1 eu nica e AT = A1 . Por m, vale armar que A e invers vel se e T somente se A o for. f Prova. E acil constatar que A1 e tamb em uma aplica ca o linear e, portanto, e tamb em um elemento de Mat (C, n). De fato, sejam v1 , v2 elementos arbitr arios de Cn e 1 , 2 C, igualmente arbitr arios. Como A e bijetora, existem u 1 , u 2 Cn , u nicos, tais que Au1 = v1 e Au2 = v2 , ou seja, tais que u1 = A1 (v1 ) e u2 = A1 (v2 ). Assim, usando a linearidade de A, tem-se A1 1 v1 + 2 v2 = A1 1 Au1 + 2 Au2 = A1 A 1 u1 + 2 u2 = 1 u1 + 2 u2 = 1 A1 (v1 ) + 2 A1 (v2 ) ,

Matrizes bijetoras e a no c ao de inversa de uma matriz

o que prova que A1 e tamb em linear e, portanto A1 Mat (C, n). Com isso, podemos armar que A1 Ax = x para todo x Cn e, portanto, AA1 Ax = Ax. Como A e sobrejetora, isso diz-nos que AA1 y = y para todo y Cn . e facilmente estabelecida, pois se B Mat (C, n) e tal Assim, estabelecemos que A1 A = AA1 = . A unicidade ao multiplicando-se AB = ` a esquerda por A1 obt em-se B = A1 . Por m, observemos que BA = AB = , ent que do fato que (M N )T = N T M T para quaisquer matrizes M, N Mat (C, n), segue de A1 A = AA1 = que T T 1 T AT A1 = A1 AT = , o que implica AT = A1 . A u ltima rela ca o implica que se A e invers vel, ent ao T T T A tamb em o e. Como (A ) = A, vale tamb em a rec proca. Mais adiante indicaremos como a matriz A1 pode ser calculada a partir de A. Vide para tal a express ao (8.17) (regra de Laplace) do Teorema 8.1, p agina 299, e tamb em as express oes (8.40), p agina 318, e (8.149), p agina 382. Em parte do que segue estaremos implicitamente usando a seguinte proposi ca o: Proposi c ao 8.2 Uma matriz A Mat (C, n) e bijetora (ou seja, e invers vel) se e somente se Av = 0 valer apenas para v = 0. Prova. Se A e bijetora, ent ao existe A1 . Logo, aplicando-se A1 ` a esquerda na igualdade Av = 0, obt em-se v = 0. Vamos agora provar a rec proca: vamos supor que Av = 0 vale apenas para v = 0 e provar que A e injetora e sobrejetora e, portanto, bijetora.

JCABarata. Curso de F sica-Matem atica

Vers ao de 19 de junho de 2011.

Cap tulo 8

299/1943

Prova-se que A e injetora por absurdo. Se A n ao e injetora, ent ao, existem vetores x e y com x = y mas com Ax = Ay . Como A e linear, isso implica A(x y ) = 0. Pela hip otese que Av = 0 vale apenas para v = 0, segue que x = y , uma contradi ca o. Para provarmos que A e sobrejetora procedemos da seguinte forma. Seja {b1 , . . . , bn } uma base em Cn . Vamos primeiramente mostrar que {Ab1 , . . . , Abn } e um conjunto linearmente independente de vetores em Cn (e, portanto, n uma base em C ). Suponhamos que assim n ao o seja e que existam n umeros complexos 1 , . . . , n , n ao todos nulos, tais que 1 Ab1 + + n Abn = 0. Pela linearidade de A, segue que A (1 b1 + + n bn ) = 0. Novamente, pela hip otese que Av = 0 vale apenas para v = 0, segue que 1 b1 + + n bn = 0. Isso, por em, diz que os vetores {b1 , . . . , bn } s ao linearmente dependentes, o que e absurdo. Logo, {Ab1 , . . . , Abn } e um conjunto de n vetores linearmente independente em Cn e, portanto, e uma base nesse espa co. Assim, qualquer x Cn pode ser escrito como uma combina ca o linear tal como x = 1 Ab1 + + n Abn = A (1 b1 + + n bn ). Isso mostra que x est a na imagem de A. Como x e arbitr ario, segue que A e sobrejetora. Um corol ario evidente e o seguinte: Corol ario 8.1 Uma matriz A Mat (C, n) e n ao-bijetora (ou seja, n ao possui inversa) se e somente se existir um vetor n ao-nulo v tal que Av = 0. O seguinte corol ario indica uma maneira pr atica, necess aria e suciente de se constarar se uma matriz A Mat (C, n) tem inversa. Corol ario 8.2 Seja A Mat (C, n) da forma A = a1 , . . . , an para o conjunto de vetores a1 , . . . , an que representam suas colunas. Ent ao, A e invers vel se e somente se os vetores a1 , . . . , an forem linearmente independentes. Vale tamb em a arma c ao que A e invers vel se e somente se suas linhas forem linearmente independentes.
v1

Prova. Se v Cn e o vetor coluna v =

vn

. . .

, ent ao e f acil constatar (pela regra de produto de matrizes. Fa ca-o!) que

Av = v1 a1 + . . . + vn an . Com isso, vemos que a arma ca o que existe v n ao-nulo tal que Av = 0 equivale ` a arma ca o que os vetores-coluna a1 , . . . , an s ao linearmente dependentes. Como A e invers vel se e somente se AT o for (Proposi ca o 8.1, p agina 298), vale armar que A e invers vel se e somente se suas linhas forem linearmente independentes.

Propriedades b asicas de determinantes de matrizes colunas. O determinante de A, det(A), foi denido em (3.6) como det(A) := det (a1 , . . . , an ) , (8.15) Seja A Mat (C, n) da forma A = a1 , . . . , an para o conjunto de vetores a1 , . . . , an que representam suas

onde det e a forma alternante maximal em n dimens oes, normalizada de sorte que det (e1 , . . . , en ) = 1. Com isso, vale det() = 1. Assim, se Sn denota o conjunto de todas as bije co es de {1, . . . , n} em si mesmo (o chamado grupo de permuta co es de n elementos), tem-se det (ej (1) , . . . , ej (n) ) = sinal(j ) para todo j Sn e, portanto, vale a express ao (3.7): A1j (1) Anj (n) sinal(j ) (8.16) det(A) =
j Sn

freq uentemente denominada f ormula de Leibniz1 para o determinante de uma matriz. O teorema a seguir re une todas as propriedades fundamentais do determinante de matrizes. Teorema 8.1 Para toda matriz A Mat (C, n) valem: 1. det(A) = n det(A) para todo C.
1 Gottfried

Wilhelm von Leibniz (16461716).

JCABarata. Curso de F sica-Matem atica

Vers ao de 19 de junho de 2011.

Cap tulo 8

300/1943

2. det(A) = det AT . Conseq uentemente, o determinante de uma matriz troca de sinal quando da permuta de duas de suas colunas ou linhas. 3. det(AB ) = det(A) det(B ) = det(BA) para qualquer B Mat (C, n). 4. det(A) = det(SAS 1 ) para qualquer S Mat (C, n), invers vel. 5. Se det(A) = 0 ent ao A n ao tem inversa. 6. Se det(A) = 0 ent ao A tem inversa e vale a chamada regra de Laplace2 : A1 = 1 Cof(A)T , det(A) (8.17)

onde Cof(A) Mat (C, n), denominada matriz dos cofatores de A, e a matriz cujos elementos s ao Cof(A)jk = det (a1 , . . . , ak1 , ej , ak+1 , . . . , an ) = det a1 , . . . , ak1 , ej , ak+1 , . . . , an . (8.18)

Em palavras, Cof(A)jk e o determinante da matriz obtida substituindo a k - esima coluna de A pelo vetor ej . No pr oximo item veremos outra caracteriza c ao da matriz dos cofatores Cof(A). Conjuntamente com o item 5, conclu mos que A tem inversa se e somente se det(A) = 0. 7. Os elementos de matriz de Cof(A) s ao dados por Cof(A)ij = (1)i+j Men(A)ij , onde Men(A), chamada de matriz dos menores de A, e a matriz de Mat (C, n) denida de sorte que cada elemento Men(A)ij seja o determinante da matriz (n 1) (n 1) obtida eliminando-se a i- esima linha e a j - esima coluna de A. Se n = 1, convenciona-se denir Men(A) = 1. Assim, para det(A) = 0, a regra de Laplace escreve-se A1
ij

(1)i+j 1 Cof(A)ji = Men(A)ji . det(A) det(A)

(8.19)

8. Para qualquer k {1, . . . , n} valem a expans ao em linhas do determinante


n n

det(A) =
j =1

Akj Cof(A)kj =
j =1

(1)j +k Akj Men(A)kj

(8.20)

e a expans ao em colunas do determinante


n n

det(A) =
j =1

Ajk Cof(A)jk =
j =1

(1)j +k Ajk Men(A)jk .

(8.21)

Em (8.149), p agina 382, apresentaremos outra f ormula expl cita para o c omputo da inversa de matrizes baseada no Teorema de Hamilton-Cayley (Teorema 8.3, p agina 315). Demonstra c ao do Teorema 8.1. Prova de 1. Pela f ormula de Leibniz (8.16), det(A) =
j Sn

(A1j (1) ) (Anj (n) ) sinal(j ) = n det(A) .

Prova de 2. Observemos a f ormula de Leibniz (8.16). Usando o fato elementar que um produto de n umeros complexos n ao depende da ordem dos fatores, podemos escrever A1j (1) Anj (n) = Al(1)j (l(1)) Al(n)j (l(n)) para qualquer l Sn .
2 Pierre-Simon

Laplace (17491827).

JCABarata. Curso de F sica-Matem atica

Vers ao de 19 de junho de 2011.

Cap tulo 8

301/1943

Em particular, escolhendo l = j 1 obtemos A1j (1) Anj (n) = Aj 1 (1)1 Aj 1 (n)n . Assim, pela f ormula de Leibniz (8.16), e usando o fato que sinal(j ) = sinal(j 1 ) para todo j Sn (justique!), vale det(A) =
j Sn

Aj 1 (1)1 Aj 1 (n)n sinal(j 1 ) =

j 1 S n

Aj 1 (1)1 Aj 1 (n)n sinal(j 1 ) =


j Sn

Aj (1)1 Aj (n)n sinal(j ) = det(AT ) .

Quando da permuta de duas linhas ou colunas de A seu determinante troca de sinal devido ` a altern ancia da forma det . A igualdade det(A) = det AT ensina que isso tamb em ocorre quando da permuta de linhas. E. 8.1 Exerc cio. Justique todas as passagens de acima. Prova de 3. Sejam A = a1 , . . . , an e B = b1 , . . . , bn . Temos que AB = Ab1 , . . . , Abn (vide (8.12)). Agora,
n n n

(Abj )i =
k=1

Aik (bj )k =
k=1

(ak )i (bj )k ,

ou seja,

Abj =
k=1

(bj )k ak .

Assim, det(AB ) = = det (Ab1 , . . . , Abn )


n n

det
n n

(b1 )k1 ak1 , . . . ,


k1 =1

(bn )kn akn


kn =1

multi-linearidade

k1 =1

kn =1

(b1 )k1 (bn )kn det (ak1 , . . . , akn )

=
k Sn

(b1 )k(1) (bn )k(n) det ak(1) , . . . , ak(n) (b1 )k(1) (bn )k(n) sinal(k ) det (a1 , . . . , an )

=
k Sn

=
k Sn

(b1 )k(1) (bn )k(n) sinal(k ) det(A)

det(B ) det(A) .

Acima, na passagem da terceira para a quarta linha usamos o fato que det (ak1 , . . . , akn ) anula-se a menos que a k1 , . . . , kn sejam distintos, o que somente ocorre se forem da forma k (1), . . . , k (n), respectivamente, para algum k Sn . Na passagem da quarta para a quinta linha usamos que det ak(1) , . . . , ak(n) = sinal(k ) det (a1 , . . . , an ), pois det e uma forma alternante. Estabelecemos, portanto, que det(AB ) = det(A) det(B ) = det(BA). Prova de 4. Do item 3 segue que, para quaisquer A, S Mat (C, n), com S invers vel, vale det(A) = det((AS 1 )S ) = 1 det(SAS ). ao A n ao pode ter inversa, pois se existisse A1 ter amos 1 = det() = det(AA1 ) = Prova de 5. Se det(A) = 0 ent 1 det(A) det(A ) = 0, absurdo.

JCABarata. Curso de F sica-Matem atica

Vers ao de 19 de junho de 2011.

Cap tulo 8

302/1943

bastante claro que podemos escrever Prova de 6. E


n

ak =
j =1

Ajk ej .

(8.22)

Logo, para qualquer k {1, . . . , n} vale


n

det(A) = det (a1 , . . . , ak1 , ak , ak+1 , . . . , an ) =


j =1

Ajk det (a1 , . . . , ak1 , ej , ak+1 , . . . , an ) .

Note que ej ocorre na k - esima posi ca o. Provamos assim que


n

det(A) =
j =1

Ajk Cof(A)jk ,
n

(8.23)

onde a matriz Cof(A) foi denida em (8.18). Mostremos agora que para l = k a express ao
j =1

Ajl Cof(A)jk e nula. De

fato,
n n

Ajl Cof(A)jk =
j =1 j =1

Ajl det (a1 , . . . , ak1 , ej , ak+1 , . . . , an )

(8.22)

det (a1 , . . . , ak1 , al , ak+1 , . . . , an ) = 0 ,

pois em det (a1 , . . . , ak1 , al , ak+1 , . . . , an ) o vetor al aparece na l- esima e na k - esima posi ca o o que faz det anular-se, por ser uma forma alternante. Provamos, assim, que
n

Ajl Cof(A)jk = kl det(A) .


j =1

(8.24)

Vamos supor que det(A) = 0. Dena-se a matriz G = det(A)1 Cof(A)T , cujos elementos de matriz s ao Gkj = det(A)1 Cof(A)jk . Ent ao, (8.24) diz-nos que
n

Gkj Ajl = kl ,
j =1

ou seja,

GA = .

Isso signica que A e invers vel com A1 = G. Prova de 7. Observemos primeiramente que, supondo provisoriamente k > 1, det (a1 , . . . , ak1 , ej , ak+1 , . . . , an ) = det (a1 Aj 1 ej , . . . , ak1 , ej , ak+1 , . . . , an ) devido ` a linearidade e ao fato que det (ej , . . . , ak1 , ej , ak+1 , . . . , an ) = 0, pelo fato de det ser alternante. Agora, a j - esima linha do vetor-coluna a1 Aj 1 ej e nula. Repetindo esse argumento podemos anular j - esima linha de todas

esima coluna, sem alterar seu determinante. Um as colunas da matriz a1 , . . . , ak1 , ej , ak+1 , . . . , an , exceto a k - pouco de medita ca o nos convence que a matriz resultante e obtida da matriz A anulando-se a k - esima coluna e a j - esima linha, exceto no cruzamento das duas, onde o elemento de matriz vale 1 (elemento jk ). O determinante dessa matriz e Cof(A)jk . Pelo item 2 e pela propriedade de altern ancia, sabemos que o determinante de uma matriz troca de sinal quando permutamos a posi ca o de duas colunas ou duas linhas quaisquer. Com esse tipo de opera ca o podemos transportar o 1

JCABarata. Curso de F sica-Matem atica

Vers ao de 19 de junho de 2011.

Cap tulo 8

303/1943

do elemento jk at e a posi ca o nn da matriz, ao pre co de realizar n k transposi co es de colunas vizinhas e n j de linhas vizinhas, as quais alteram o determinante por fatores (1)nk e (1)nj , respectivamente. Temos com isso que := det A[jk] 0 . . . , 0 1

Cof(A)jk = (1)k+j det(A[jk] )

com

A[jk]

onde A[jk] e a matriz de Mat (C, n 1) obtida eliminando a j - esima linha e a k - esima coluna da matriz A. Pela f ormula de Leibniz (8.16), det A[jk] Como A[jk] =
l S n nl(n)

A[jk]
1l(1)

A[jk]

nl(n)

sinal(l) .

= l(n), n (justique!), segue que det A[jk] =


l S
n 1

A[jk]
1l (1)

A[jk] A[jk]

(n1)l (n1)

sinal(l )

=
l S
n 1

A[jk]

1l (1)

(n1)l (n1)

sinal(l )

det A[jk]

= Men(A)jk .

(Justique por que a soma no lado direito da primeira linha acima e sobre Sn1 e n ao mais sobre Sn ). Provamos, portanto, que Cof(A)jk = (1)k+j Men(A)jk . A rela ca o (8.19) e imediata por (8.17). e imediata por (8.23) e pelo item 7. Eq. (8.20) segue facilmente de (8.21) usando o item 2. Prova de 8. Eq. (8.21)

Menores e cofatores de uma matriz. Propriedades adicionais E. 8.2 Exerc cio. Seja Mat (C, n), = diag + 1, 1, +1, . . . , (1)n+1 , a matriz diagonal cujos elementos s ao alternadamente +1 e 1, ou seja, ij = (1)i+1 ij . Mostre que Cof(A) = Men(A)1 para toda matriz A Mat (C, n). Para uma matriz M Mat (C, n), a transforma ca o de similaridade M M 1 e denominada chessboard transformation, pois com ela os sinais s ao trocados em M como alternam-se as cores das casas em um tabuleiro de xadrez. co es E. 8.3 Exerc cio. Usando a regra de Laplace (8.17), mostre que para toda matriz A Mat (C, n) valem as rela Men(A1 ) = Men(A)1 , Cof(A) = Men(A1 ) , Cof(A1 ) = Cof(A)1 , Men(A) = Cof(A1 ) .

JCABarata. Curso de F sica-Matem atica

Vers ao de 19 de junho de 2011.

Cap tulo 8

304/1943

Se A Mat (C, n) e invers vel, segue da regra de Laplace (8.17) que det(A1 ) = det(Cof(A)) = det(A)n1 . Do Exerc cio E. 8.3, conclui-se tamb em que det(Men(A)) = det(A)n1 . E. 8.4 Exerc cio. Mostre que para toda matriz A Mat (C, n), n 2, vale Cof Cof(A) Do Exerc cio E. 8.3, obt em-se tamb em Men Men(A) Assim, para toda matriz A Mat (C, n) vale Cof Cof(A) = Men Men(A) . Portanto, se det(A) = 1 e n 2, vale Cof Cof(A) = Men Men(A) = A. Um resultado u til = det(A) = det(A)
n2

1 det(A)n

det(Cof(A)) e, portanto, (8.25)

(8.26)

A. A.

n2

Mais abaixo usaremos o seguinte fato:

Proposi c ao 8.3 Seja M Mat (C, n) uma matriz da seguinte forma A M = B

k, nk
C

onde A e uma matriz k k (com k < n), B e uma matriz (n k ) k e C e uma matriz (n k ) (n k ). Ent ao, det(M ) = det(A) det(C ) .

Prova. O primeiro ingrediente da prova e a constata ca o que A B

k, nk
C

k, nk k nk
B

k, nk k nk

k, nk
C

nk, k

nk, k

E. 8.5 Exerc cio. Verique! Com isso, temos pela regra do determinante de um produto de matrizes que det(M ) = det A

k, nk nk

nk, k

k det B

k, nk nk

k det

k, nk
C

nk, k

JCABarata. Curso de F sica-Matem atica

Vers ao de 19 de junho de 2011.

Cap tulo 8

305/1943

Isso completa a prova.

det

Agora, pelas regras (8.20)(8.21) de c alculo de determinantes, e f acil constatar (fa ca-o!) que A

k, nk nk

nk, k

= det(A),

k det

k, nk
C

nk, k

= det(C )

k det B

k, nk nk

= 1.

8.2

No co es B asicas sobre o Espectro de uma Matriz

Seja A Mat (C, n) uma matriz n n com entradas complexas. No estudo das propriedades de A e de grande import ancia saber para quais n umeros complexos a matriz A e invers vel e para quais n ao e. Essa quest ao conduz as seguintes importantes deni ` co es: Deni c ao. O espectro de A Mat (C, n), denotado por (A), e denido como sendo o conjunto de todos os C para os quais a matriz A n ao tem inversa. Assim, um n umero complexo e dito ser um elemento do espectro de ao possuir uma inversa. A Mat (C, n) se a matriz A n e denido como sendo o conjunto de todos os Deni c ao. O conjunto resolvente de A Mat (C, n), denotado por (A), C para os quais a matriz A tem inversa. Assim, um n umero complexo e dito ser um elemento do conjunto resolvente de A Mat (C, n) se a matriz A possuir uma inversa. evidente que (A) e (A) s E ao conjuntos complementares, ou seja, (A) (A) = mas (A) (A) = C.

O espectro de uma matriz

Um fato importante e que A e n ao-invers vel se e somente se det( A) = 0 (vide Teorema 8.1, p agina 299). Assim, um n umero complexo e um elemento do espectro de uma matriz A se e somente se for tal que det( A) = 0. Essa observa ca o conduz-nos ao importante conceito de polin omio caracter stico de uma matriz.

8.2.1

Autovalores e Polin omios Caracter sticos de Matrizes

O polin omio caracter stico de uma matriz

Seja A Mat (C, n) uma matriz cujos elementos de matriz s ao Aij . Para z C a express ao z A11 A 21 pA (z ) := det(z A) = det . . . An1 A12 . . . .. z A22 . A1n A2n . . . z Ann (8.27)

An2

dene, um polin omio de grau n na vari avel z , com coecientes complexos, os quais dependem dos elementos de matriz Aij de A. Isso se constata facilmente pelos m etodos usuais de c alculo de determinantes (por exemplo, as expans oes em linha ou coluna de (8.20) e (8.21)), Esse polin omio e denominado polin omio caracter stico de A e desempenha um papel muito importante no estudo de propriedades de matrizes. O leitor poder a encontrar na Se ca o 8.10.1, p agina 381, uma express ao mais expl cita para

JCABarata. Curso de F sica-Matem atica

Vers ao de 19 de junho de 2011.

Cap tulo 8

306/1943

o polin omio caracter stico em termos dos elementos de matriz Aij de A (vide (8.148), p agina 381), mas por ora n ao precisaremos de maiores detalhes sobre esse polin omio. Como todo polin omio complexo de grau n, pA possui n ra zes, n ao necessariamente distintas no plano complexo (Teorema Fundamental da Algebra). As ra zes do polin omio caracter stico pA s ao denominadas autovalores da matriz A. Assim, o espectro de uma matriz A coincide com o conjunto de seus autovalores. O estudo de autovalores de matrizes e de grande import ancia na Algebra Linear e em suas aplica co es ` a Teoria das Equa co es Diferenciais, ` a Geometria, ` a Teoria dos Sistemas Din amicos e ` a F sica, especialmente ` a F sica Qu antica. Seja A Mat (C, n) uma matriz e sejam 1 , . . . , r , 1 r n, seus autovalores distintos, cada qual com multiplicidade a1 , . . . , ar , respectivamente, ou seja, cada i e uma raiz de ordem ai N do polin omio caracter stico de A:
r

pA (z ) = det(z A) =

i=1

(z i )ai .

A quantidade ai e um n umero inteiro positivo e e denominado multiplicidade alg ebrica do autovalor i . Note-se que como o n umero de ra zes de pA (contando as multiplicidades) e exatamente igual a seu grau, segue facilmente que a seguinte rela ca o e v alida:
r

ai = n ,
i=1

(8.28)

ou seja, a soma das multiplicidades alg ebricas dos autovalores de uma matriz A Mat (C, n) e n. Uma conseq u encia elementar disso e a seguinte proposi ca o u til: Proposi c ao 8.4 Seja A Mat (C, n) uma matriz e sejam 1 , . . . , r , 1 r n, seus autovalores distintos, cada qual com multiplicidade alg ebrica a1 , . . . , ar , respectivamente. Ent ao,
r

det(A) =
k=1

(k )ak .

(8.29)

Prova. Por deni ca o, o polin omio caracter stico de A e pA (z ) = det(z A) = k=1 (z k )ak . Tomando z = 0 e usando r n ak (8.28), teremos det(A) = (1) em, det(A) = (1)n det(A) e a proposi ca o est a demonstrada. k=1 (k ) . Por
r

Sabemos que o determinante e invariante por transforma co es de similaridade, pois para toda matriz A vale det(A) = det(P 1 AP ), mas n ao eou nico objeto associado a uma matriz que e invariante por tais transforma co es. O polin omio caracter stico e, portanto, o conjunto de seus autovalores (incluindo as multiplicidades alg ebricas), tamb em o e. Isso eo conte udo da seguinte arma ca o. Proposi c ao 8.5 Sejam A e B Mat (C, n) duas matrizes similares, ou seja, tais que existe P Mat (C, n), invers vel, com B = P 1 AP . Ent ao, os polin omios caracter sticos de A e de B coincidem: pA = pB . Conseq uentemente, se A e B Mat (C, n) s ao similares, seus autovalores s ao iguais (e, portanto, seus espectros: (A) = (B )), incluindo suas multiplicidades alg ebricas. e pB (z ) = det(z B ). Logo, Prova. O polin omio caracter stico de A e pA (z ) = det(z A) e o de B pA (z ) = det(z A) = det(P 1 (z A)P ) = det(z P 1 AP ) = det(z B ) = pB (z ) ,

Duas matrizes A Mat (C, n) e B Mat (C, n) s ao ditas matrizes similares se existir uma matriz invers vel P Mat (C, n) tal que P 1 AP = B . Para uma matriz invers vel P Mat (C, n) xa, a transforma ca o que leva cada matriz A Mat (C, n) ` a matriz P 1 AP e denominada transforma c ao de similaridade.

Matrizes similares. Transforma co es de similaridade

(8.30)

para todo z C. Acima usamos o fato que para P invers vel e para qualquer matriz M vale det(P 1 M P ) = 1 1 det(P ) det(M ) det(P ) = det(P P ) det(M ) = det() det(M ) = det(M ).

JCABarata. Curso de F sica-Matem atica

Vers ao de 19 de junho de 2011.

Cap tulo 8

307/1943

Coment arios sobre matrizes invers veis e sobre matrizes n ao-invers veis Proposi c ao 8.6 Seja A Mat (C, n) uma matriz arbitr aria e B Mat (C, n) uma matriz invers vel. Ent ao, existem constantes M1 e M2 (dependentes de A e de B ) com 0 < M1 M2 tais que a matriz A + B e invers vel para todo C com 0 < || < M1 e para todo C com || > M2 . a invers vel se e somente Prova. Como B tem inversa, podemos escrever A + B = + AB 1 B . Assim, A + B ser se + AB 1 o for. Seja C AB 1 e sejam {1 , . . . , n } C as n ra zes (n ao necessariamente distintas) do polin omio caracter stico pC da matriz C . Se todos as ra zes forem nulas, tomemos M1 = M2 > 0, arbitr arios. De outra forma, denamos M1 como sendo o menor valor de |k | dentre as ra zes n ao-nulas de pC : M1 := min{|k |, k = 0} e denimos M2 como sendo ao, o conjunto { C| 0 < || < M1 } e o o maior valor de |k | para todos os k s: M2 := max{|k |, k = 1, . . . , n}. Ent conjunto { C| || > M2 } n ao cont em ra zes do polin omio caracter stico de C e, portanto, det( C ) = 0, provando que para esses valores de a matriz C = + AB 1 e invers vel. Uma conseq u encia evidente da Proposi ca o 8.6 e a seguinte arma ca o: Corol ario 8.3 Seja B Mat (C, n) uma matriz invers vel e A Mat (C, n) uma matriz arbitr aria. Ent ao, existem constantes 0 < N1 N2 (dependentes de A e de B ) tais que para toda C com | | < N1 ou com | | > N2 a matriz B + A e tamb em invers vel. Prova. Para = 0 a arma ca o e evidente. Para = 0 a arma ca o segue Proposi ca o 8.6 escrevendo-se B + A = 1 B e tomando-se = 1/ , N1 = 1/M2 e N2 = 1/M1 . A+ O interesse pelo Corol ario 8.3 e devido ao fato de este armar que se B Mat (C, n) uma matriz invers vel ent ao toda matriz pr oxima o suciente da mesma e tamb em invers vel. O estudante mais avan cado h a de reconhecer que essa arma ca o ensina-nos que o conjunto da matrizes invers veis em Mat (C, n) e um conjunto aberto (em uma topologia m etrica adequada). Essa arma ca o ser a generalizada (a saber, para algebras de Banach com unidade) no Corol ario 35.6, p agina 1755. A Proposi ca o 8.6 arma tamb em que e sempre poss vel encontrar uma matriz invers vel pr oxima a uma matriz n ao-invers vel. De fato, se A Mat (C, n) n ao tem inversa a Proposi ca o 8.6 garante que a matriz A + , por exemplo, ser a invers vel para todo C com || pequeno o suciente, mas n ao-nulo.

Uma forma geom etrica de compreender as arma co es de acima e lembrar que conjunto Mat (C, n) e um espa co vetorial n2 -dimensional complexo e as matrizes invers veis s ao um subconjunto (n2 1)-dimensional do mesmo, pois s ao caracterizados pela condi ca o de terem determinante nulo, uma condi ca o polinomial sobre os n2 coecientes das matrizes 2 que dene, portanto, uma uni ao nita de superf cies alg ebricas (n 1)-dimensionais fechadas em Mat (C, n). Desse ponto de vista geom etrico, ca claro que o conjunto das matrizes invers veis e aberto (por ser o complementar das superf cies fechadas mencionadas acima) e ca claro que e sempre poss vel encontrar uma matriz invers vel pr oxima a uma matriz n ao-invers vel, pois estas u ltimas residem em superf cies alg ebricas de dimens ao menor que a dimens ao de Mat (C, n). Uma propriedade dos polin omios caracter sticos

A seguinte proposi ca o, a qual cont em uma arma ca o em nada evidente, e uma conseq u encia da Proposi ca o 8.5, p agina 306, e da Proposi ca o 8.6, p agina 307: Proposi c ao 8.7 Sejam A, B Mat (C, n). Ent ao, o polin omio caracter stico de AB e igual ao polin omio caracter stico de BA, ou seja, pAB = pBA . Conseq uentemente, se A, B Mat (C, n) ent ao as matrizes AB e BA t em os mesmos autovalores (e, portanto, os mesmos espectros: (AB ) = (BA)), com as mesmas multiplicidades alg ebricas. O estudante mais avan cado poder a interesar-se em encontrar na Proposi ca o 35.27, p agina 1756, uma vers ao dos resultados da Proposi ca o 8.7 para o caso de algebras de Banach com unidade.

JCABarata. Curso de F sica-Matem atica

Vers ao de 19 de junho de 2011.

Cap tulo 8

308/1943

Prova da Proposi c ao 8.7. Se A ou B s ao invers veis (ou ambas), ent ao AB e BA s ao similares, pois no primeiro caso teremos AB = A(BA)A1 e no segundo teremos AB = B 1 (BA)B . Nesses casos a arma ca o segue da Proposi ca o 8.5, p agina 306. O u nico caso que resta considerar e aquele no qual nem A nem B s ao invers veis. Nesse caso, por em, temos pela Proposi ca o 8.6, p agina 307, que existe M > 0 tal que a matriz A + e invers vel para todo C pertencente ao aberto 0 < || < M . Assim, para tais valores de valer a, pelo racioc nio acima p(A+)B = pB (A+) . Agora, os ao polin omios em e, portanto, s ao fun co es cont nuas de . Logo, a igualdade coecientes de p(A+)B e de pB (A+) s p(A+)B = pB (A+) permanece v alida no limite 0, fornecendo pAB = pBA , como desej avamos demonstrar. A Proposi ca o 8.7 pode ser generalizada para matrizes n ao-quadradas, como indicado no exerc cio que segue: E. 8.6 Exerc cio. Sejam A Mat (C, m, n) e B Mat (C, n, m), de sorte que AB Mat (C, m) e BA Mat (C, n). Mostre que xn pAB (x) = xm pBA (x). Sugest ao: Considere as matrizes (m + n) (m + n) denidas por A A :=

n, n

m, m n, m

(Vide (8.7), p agina 297). Mostre que

B B :=

m, m m, n

n, n

c ao 8.7, tem-se pA B (x) = pB A (x), Em seguida, prove que pA B (x) = xn pAB (x) e que pB A (x) = xm pBA (x). Pela Proposi de onde segue que xn pAB (x) = xm pBA (x). Segue disso que o conjunto de autovalores n ao-nulos de AB coincide com o conjunto de autovalores n ao-nulos de BA: (AB ) \ {0} = (BA) \ {0} e, portanto, (AB ) e (BA) podem n ao ter em comum apenas o elemento 0.

AB A B =

n, m

m, n n, n

e que

BA B A =

m, n m, m

n, m

8.2.2

Autovetores

Autovetores

Pela deni ca o, um n umero 0 C e um autovalor de uma matriz A se e somente se 0 A n ao tem inversa e, portanto (pelo Corol ario 8.1, p agina 299) se e somente se existir um menos um vetor n ao-nulo v tal que (0 A)v = 0, ou seja, tal que Av = 0 v . Chegamos a mais uma importante deni ca o: Deni c ao. Um vetor n ao-nulo v e dito ser um autovetor de uma matriz A se houver 0 C tal que Av = 0 v . Note-se que se um tal 0 satisfaz a rela ca o acima para algum v = 0 ent ao 0 A n ao tem inversa. 0 e ent ao um elemento do espectro de A, ou seja, um autovalor. 0 e dito ser o autovalor associado ao autovetor v . Uma observa ca o importante e a seguinte. Sejam v1 e v2 dois autovetores aos quais est a associado o mesmo autovalor, ou seja, Av1 = 0 v1 e Av2 = 0 v2 . Ent ao, para quaisquer n umeros complexos c1 e c2 o vetor v = c1 v1 + c2 v2 tamb em satisfaz Av = 0 v . De fato, Av = A(c1 v1 + c2 v2 ) = c1 Av1 + c2 Av2 = c1 0 v1 + c2 0 v2 = 0 (c1 v1 + c2 v2 ) = 0 v . A conclus ao e que, para cada autovalor i de uma matriz A, a cole ca o formada pelo vetor nulo e todos os autovetores de A com autovalor i e um subespa co vetorial. Vamos denotar esse subespa co por E(i ) ou simplesmente Ei . Se i e j s ao autovalores distintos de A ent ao os subespa cos de autovetores E(i ) e E(j ) t em em comum apenas o vetor nulo, ou seja, E(i ) E(j ) = {0}. Isso e f acil de provar, pois se w e tal que Aw = i w e Aw = j w ent ao, subtraindo-se uma rela ca o da outra ter amos 0 = (i j )w, que implica w = 0, j a que i = j .

JCABarata. Curso de F sica-Matem atica

Vers ao de 19 de junho de 2011.

Cap tulo 8

309/1943

Essas considera co es nos levam a mais um conceito importante: o de multiplicidade geom etrica de um autovalor. A multiplicidade geom etrica de um autovalor

Al em do conceito de multiplicidade alg ebrica de um autovalor, h a tamb em o conceito de multiplicidade geom etrica de um autovalor, do qual trataremos agora. Como antes seja A Mat (C, n) uma matriz e sejam 1 , . . . , r , 1 r n, seus autovalores distintos, cada qual com multiplicidade alg ebrica a1 , . . . , ar , respectivamente. Acima introduzimos os subespa cos Ei = E(i ), denidos como sendo os subespa cos gerados por todos os autovetores que t em i como autovalor. A multiplicidade geom etrica de um autovalor i e denida como sendo a dimens ao do subespa co Ei , ou seja, como sendo o n umero m aximo de autovetores linearmente independentes com autovalor i . importante advertir de imediato o leitor do fato que a multiplicidade alg E ebrica e multiplicidade geom etrica de autovalores nem sempre coincidem. Isso e bem ilustrado no seguinte exemplo simples. Seja 0 A = 0 1 . 0

Seu polin omio caracter stico e

Assim, seu ( unico) autovalor e 0 com multiplicidade alg ebrica 2. Quais os seus autovetores? ao aqueles que vetores S

1 = 2 . pa () = det( A) = det 0

0 1 a b a = = 0. ca o Av = 0 signica satisfazem Av = 0. Denotando v como um vetor coluna v = , a rela 0 b 0 0 b com autovalor zero tem dimens ao 1. Assim, a multiplicidade alg ebrica do autovalor zero e 2 mas a sua multiplicidade geom etrica e 1. A multiplicidade alg ebrica e a multiplicidade geom etrica

a Logo, b = 0 e todos os autovetores s ao da forma v = co gerado pelos autovetores , a C. E evidente que o subespa 0

Apesar de a multiplicidade alg ebrica e a multiplicidade geom etrica de um autovalor nem sempre coincidirem, h a uma rela ca o de ordem entre eles. A saber, e poss vel mostrar que a multiplicidade geom etrica de um autovalor e sempre menor ou igual ` a sua multiplicidade alg ebrica. Isso segue das seguintes considera co es. Seja 0 um autovalor de A Mat (C, n) e E(0 ) o subespa co gerado pelos autovetores com autovalor 0 , e cuja dimens ao denotaremos por d. Vamos escolher uma base v1 , . . . , vd , vd+1 , . . . , vn onde os primeiros d vetores s ao elementos de E(0 ). Nessa base a matriz A tem a forma D A3

d, nd
A4

e uma matriz (n d) (n d) e A3 e uma matriz onde D e uma matriz d d diagonal D = diag 0 , . . . , 0 , A4


d vezes (n d) d. Alguns segundos (minutos?) de medita ca o, usando a Proposi ca o 8.3 da p agina 304, nos levam a concluir que o polin omio caracter stico de A e dado por

det( A) = ( 0 )d det( A4 ) .

JCABarata. Curso de F sica-Matem atica

Vers ao de 19 de junho de 2011.

Cap tulo 8

310/1943

Isso mostra que a multiplicidade alg ebrica de 0 e pelo menos igual a d, sua multiplicidade geom etrica. E. 8.7 Exerc cio. Realize a medita c ao sugerida acima. Matrizes simples

O que foi exposto acima leva-nos naturalmente ao conceito de matriz simples que, como veremos mais adiante, est a intimamente ligado ao problema da diagonalizabilidade de matrizes. Deni c ao. Uma matriz A Mat (C, n) e dita ser uma matriz simples se cada autovalor de A tiver uma multiplicidade alg ebrica igual ` a sua multiplicidade geom etrica. Deixamos para o leitor provar o seguinte fato: toda matriz diagonal e simples. E. 8.8 Exerc cio. Prove isso. Adiante faremos uso da seguinte proposi ca o. Proposi c ao 8.8 Se A Mat (C, n) e uma matriz simples e P Mat (C, n) e invers vel ent ao P 1 AP e tamb em simples. Prova. J a vimos na Proposi ca o 8.5, p agina 306, que A e P 1 AP t em o mesmo polin omio caracter stico e, portanto, os mesmos autovalores, incluindo suas multiplicidades alg ebricas. Seja 0 um desses autovalores com multiplicidade alg ebrica d e sejam v1 , . . . , vd um conjunto de d autovetores linearmente independentes de A. Os vetores P 1 v1 , . . . , P 1 vd s ao autovetores de P 1 AP com autovalor 0 . De fato, P 1 AP P 1 vi = P 1 Avi = 0 P 1 vi . Fora isso os d vetores P 1 v1 , . . . , P 1 vd s ao tamb em linearmente independentes. Para ver isso, suponha houvesse constantes c1 , . . . , cd tais que c1 P 1 v1 + + cd P 1 vd = 0 . Multiplicando-se ` a esquerda por P ter amos c1 v1 + + cd vd = 0. Como v1 , . . . , vd s ao linearmente independentes as constantes ci t em que ser todas nulas, provando que os vetores P 1 v1 , . . . , P 1 vd s ao tamb em linearmente independentes. Isso prova que a multiplicidade geom etrica do autovalor 0 e pelo menos igual a d. Como ela n ao pode ser maior que d (p agina 309), conclui-se que e igual a d provando a proposi ca o. A seguinte proposi ca o elementar e por vezes u til para vericar se uma matriz e simples. Proposi c ao 8.9 Se todos os n autovalores de uma matriz A Mat (C, n) forem distintos ent ao A e simples. Prova. Se os autovalores de A s ao 1 , . . . , n , todos distintos, ent ao cada um tem multiplicidade alg ebrica igual a 1. For cosamente, sua multiplicidade geom etrica e tamb em igual a 1, j a que a multiplicidade geom etrica n ao pode ser maior que a alg ebrica. Ressaltemos que a rec proca da proposi ca o acima n ao e verdadeira: uma matriz pode ser simples e possuir autovalores com multiplicidade alg ebrica maior que 1.

8.2.3

O Tra co de uma Matriz

O tra co de uma matriz

Seja A Mat (C, n), cujos elementos de matriz s ao Aij , i, j = 1, . . . n. Sejam 1 , . . . , n seus n autovalores (n ao necessariamente distintos e repetidos conforme sua multiplicidade).

JCABarata. Curso de F sica-Matem atica

Vers ao de 19 de junho de 2011.

Cap tulo 8

311/1943

Denimos o tra co de A como sendo a soma de seus n autovalores:


n

Tr(A) :=
a=1

a .

Uma conclus ao que se tira dessa deni ca o e que se duas matrizes s ao similares, ent ao ambas t em o mesmo tra co, ou seja, para qualquer matriz invers vel P e qualquer matriz A vale Tr P 1 AP = Tr(A) . (8.31)

A raz ao reside na observa ca o feita acima que duas matrizes similares t em o mesmo conjunto de autovalores e, portanto, o mesmo tra co. Temos a seguinte e importante proposi ca o: Proposi c ao 8.10 O tra co de uma matriz A Mat (C, n) e igual a soma dos elementos de sua diagonal principal, ou seja,
n n

Tr(A) :=
a=1

a =
a=1

Aaa .

(8.32)

Prova. A demonstra ca o consistir a em se calcular o coeciente de n1 no polin omio caracter stico p() de A de dois modos diferentes. O polin omio caracter stico pA () de A e dado por (8.27). As t ecnicas de c alculo de determinantes n (e.g., (8.20) e (8.21)) dizem-nos que o coeciente de n1 e i=1 Aii . Por exemplo, para o caso n = 2 A11 p() = det A21 A12 = 2 (A11 + A22 ) + A11 A22 A12 A21 . A22

E. 8.9 Exerc cio. Conven ca-se da veracidade da armativa acima para o caso de n arbitr ario. Sugest ao: use a expans ao em cofatores (8.20)(8.21) ou leia a Se c ao 8.10.1, p agina 381. Por outro lado, os autovalores de A, 1 , . . . , n , s ao por deni ca o as ra zes do polin omio caracter stico. Logo, p() = ( 1 )( 2 ) ( n ) . Expandindo-se essa express ao, conclui-se que o coeciente de n1 e (1 + + n ) = Tr(A) . E. 8.10 Exerc cio. Certo? Do exposto acima, conclui-se que o coeciente de n1 no polin omio caracter stico de A e
n

o que termina a prova.

i=1

Aii = (1 + + n ) = Tr(A) ,

Essa proposi ca o leva a duas outras propriedades igualmente importantes: a linearidade do tra co e a chamada propriedade c clica do tra co.

JCABarata. Curso de F sica-Matem atica

Vers ao de 19 de junho de 2011.

Cap tulo 8

312/1943

Proposi c ao 8.11 (A Linearidade do Tra co) Sejam A, B Mat (C, n) e , C. Ent ao, Tr(A + B ) = Tr(A) + Tr(B ) .

Prova. A prova e imediata por (8.32). curioso notar que a linearidade do tra E co vista acima e evidente por (8.32), mas n ao e nem um pouco evidente pela deni ca o do tra co de uma matriz como soma de seus autovalores, pois os autovalores individuais de A + B n ao s ao em geral combina co es lineares dos autovalores de A e de B , especialmente no caso em que A e B n ao comutam. Proposi c ao 8.12 (A Propriedade C clica do Tra co) Sejam A, B Mat (C, n). Ent ao, Tr(AB ) = Tr(BA) .

Prova. Pelo que vimos acima, tem-se


n n

Tr(AB ) =
i=1

(AB )ii =
i=1

n j =1

Na segunda e quarta igualdades usamos a regra de produto de matrizes. Na terceira igualdade apenas trocamos a ordem das somas. A propriedade c clica expressa na Proposi ca o 8.12 pode ser provada diretamente da deni ca o do tra co de uma matriz como soma de seus autovalores (incluindo multiplicidades alg ebricas) se recordarmos a Proposi ca o 8.7, p agina 307, que arma que AB e BA t em os mesmos auto-valores com as mesmas multiplicidades alg ebricas.

Aij Bji =

n j =1

Bji Aij
i=1

=
j =1

(BA)jj = Tr(BA) .

8.2.3.1

Algumas Rela c oes entre Determinantes e Tra cos de Matrizes

Proposi c ao 8.13 Seja A() Mat (C, n) uma matriz que depende de forma diferenci avel de uma vari avel (que pode ser real ou complexa) em um certo dom nio e suponhamos que A() seja invert vel para todos os valores de no dom nio considerado. Ent ao, vale d d 1 det A() = Tr A()1 A() . (8.33) d det A() d

Prova. Por (8.16), tem-se d det A() d =


Sn n

sinal( )

d A1(1) () An(n) () + + d

Sn

sinal( )A1(1) ()

d An(n) () d

=
k=1

det Bk () ,

onde Bk () e a matriz obtida substituindo a k - esima linha da matrix A() pela linha Usando a expans ao em linha do determinante, express ao (8.20), temos
n

d d Ak1 ()

d d Akn ()

det Bk ()

=
j =1

d Akj () d

Cof A()

(8.19) kj

det A()
j =1

d Akj () d

A1

jk

JCABarata. Curso de F sica-Matem atica

Vers ao de 19 de junho de 2011.

Cap tulo 8

313/1943

Logo, d det A() d = det A()

k=1 j =1

d Akj () d

A1

jk

= det A() Tr A()1

d A() d

A express ao (8.33) eu til at e mesmo no contexto da Geometria Riemanniana. Para uma aplica ca o naquele contexto, vide express ao (31.93), p agina 1475. Uma das conseq u encias de (8.33) e o seguinte resultado, tamb em muito u til: Proposi c ao 8.14 Seja A Mat (C, n). Ent ao, vale que det eA = eTr(A) . (8.34)

f ca o de exponencial de uma matriz ser a apresentada em (9.20), p agina 394. E acil ver de (9.20) que Nota para o estudante. A no AeA = eA A para qualquer matriz A Mat (C, n). Da Proposi ca o 9.6, p agina 396, segue facilmente que eA e invert vel e que sua inversa e eA tamb em para qualquer A Mat (C, n).
d A d A Prova da Proposi c ao 8.14. Seja A() = eA . Ent ao, d e = AeA = eA A (por (9.20)) e, portanto, eA = A. d e d Dessa forma, (8.33) ca d ln det A() = Tr(A). Integrando-se em entre 0 e 1 e lembrando que A(1) = eA e que e o que quer amos provar. A(0) = , teremos ln det eA = Tr(A), que 1

Uma segunda demonstra ca o da Proposi ca o 8.14 ser a encontrada na Proposi ca o 9.7, p agina 398.

8.3

Polin omios de Matrizes

Seja p um polin omio de grau m: p(x) = am xm + + a1 x + a0 com x C, aj C e am = 0. Para uma matriz A Mat (C, n) denimos o polin omio matricial p(A) por p(A) = am Am + + a1 A + a0 . Obviamente p(A) e tamb em uma matriz n n com entradas complexas.
r

Polin omios de matrizes

Se as ra zes do polin omio p forem 1 , . . . , r , com multiplicidades m1 , . . . , mr , respectivamente, ent ao p(x) = am


j =1

(x j )mj ,

f para todo x C. E acil provar, ent ao, que


r

p(A) = am
j =1

(A j )mj .

E. 8.11 Exerc cio. Justique isso. e um polin omio ent ao E. 8.12 Exerc cio. Mostre que se D = diag (d1 , . . . , dn ) e q q (D) = diag (q (d1 ), . . . , q (dn )) .

JCABarata. Curso de F sica-Matem atica

Vers ao de 19 de junho de 2011.

Cap tulo 8

314/1943

e um polin omio mostre que E. 8.13 Exerc cio. Suponha que A = P 1 DP , onde D = diag (d1 , . . . , dn ). Se q q (A) = P 1 q (D)P = P 1 diag (q (d1 ), . . . , q (dn )) P .

O polin omio m nimo

Vamos mostrar que para cada matriz A Mat (C, n) sempre existe pelo menos um polin omio p com a propriedade que p(A) = . Para tal notemos primeiramente que Mat (C, n) e um espa co vetorial complexo de dimens ao n2 . De fato toda a matriz A Mat (C, n), cujos elementos de matriz s ao Aij C pode ser trivialmente escrita na forma
n n

A =
a=1 b=1 ab

Aab E ab

onde E Mat (C, n) s ao matrizes cujos elementos de matriz s ao (E ab )ij = i,a j,b , ou seja, todos os elementos de ab matriz de E s ao nulos, exceto o elemento a, b, que vale 1. E. 8.14 Exerc cio. Certo? Assim, vemos que as matrizes {E ab , a = 1, . . . , n, b = 1, . . . , n} formam uma base em Mat (C, n), mostrando que Mat (C, n) e um espa co vetorial de dimens ao n2 . Isto posto, temos que concluir que qualquer conjunto de mais de n2 matrizes n ao-nulas em Mat (C, n) e linearmente dependente. Se uma das matrizes Ak , k = 1, . . . , n2 , for nula, digamos Aq = , ent ao o polin omio p(x) = xq tem a propriedade k que p(A) = 0, que e o que desejamos provar. Se, por outro lado, as matrizes A , k = 1, . . . , n2 , s ao todas n ao-nulas, 2 e linearmente dependente, pois possui n2 + 1 elementos. Portanto, existem ent ao o conjunto {, A, A2 , . . . , An } constantes c0 , . . . , cn2 , nem todas nulas, tais que c0 + c1 A + c2 A2 + + cn2 An = . Como o lado esquerdo e um polin omio em A, ca provada nossa arma ca o que toda matriz possui um polin omio que a anula. Chegamos ` as seguintes deni co es: omio p : R C de grau n e dito ser um polin omio m onico se for da forma Deni c ao Polin omio M onico.. Um polin p(x) = xn + an1 xn1 + + a1 x + a0 , ou seja, se o coeciente do mon omio de maior grau (no caso, xn ) for igual a 1. Note-se que polin omios m onicos nunca s ao identicamente nulos. omio m nimo de A eo Deni c ao Polin omio M nimo de uma Matriz.. Dada uma matriz A Mat (C, n), o polin polin omio m onico de menor grau que e anulado em A, ou seja, e o polin omio n ao-nulo de menor grau da forma M (x) = xm + am1 xm1 + + a1 x + a0
2

para o qual M (A) = .

As considera co es acima mostram que um tal polin omio sempre existe e que tem grau no m aximo igual a n2 . Essa e, no entanto, uma estimativa exagerada para o grau do polin omio m nimo de uma matriz A Mat (C, n) pois, como veremos abaixo, o polin omio m nimo de uma matriz A Mat (C, n) tem, na verdade, grau menor ou igual a n. Isso e um corol ario de um teorema conhecido como Teorema de Hamilton-Cayley , que demonstraremos abaixo (Teorema 8.3, p agina 315). Finalizamos com um teorema b asico que garante a unicidade do polin omio m nimo e estabelece sua rela ca o com outros polin omios que anulam A. Teorema 8.2 O polin omio m nimo M de uma matriz A Mat (C, n) e u nico. Fora isso se P e um polin omio n aoao P e divis vel por M , ou seja, existe um identicamente nulo que tamb em se anula em A, ou seja, P (A) = , ent polin omio F tal que P (x) = F (x)M (x) para todo x C.

JCABarata. Curso de F sica-Matem atica

Vers ao de 19 de junho de 2011.

Cap tulo 8

315/1943

Demonstra c ao. Dada uma matriz A Mat (C, n), o polin omio m nimo de A e o polin omio de menor grau da forma para o qual M (A) = . Vamos supor que haja outro polin omio N da forma amos o polin omio para o qual N (A) = . Subtraindo um do outro ter M (x) = xm + am1 xm1 + + a1 x + a0 N (x) = xm + bm1 xm1 + + b1 x + b0

que tem grau menor ou igual a m 1 e para o qual vale (M N )(A) = M (A) N (A) = = . Como, por hip otese, n ao h a polin omios n ao-nulos com grau menor que o de M que anulam A, isso e uma contradi ca o, a menos que M = N . Isso prova a unicidade. Seja P um polin omio n ao identicamente nulo para o qual valha P (A) = . Se p e o grau de P , deve-se ter p m, onde m e o grau do polin omio m nimo de A. Logo, pelos bem conhecidos fatos sobre divis ao de polin omios, podemos encontrar dois polin omios F e R, cujos graus s ao, respectivamente p m e r com 0 r < m, tais que P (x) = F (x)M (x) + R(x) , para todo x C. Ora, isso diz que P (A) = F (A)M (A) + R(A) .

(M N )(x) = (am1 bm1 )xm1 + + (a1 b1 )x + (a0 b0 ) ,

Como P (A) = e M (A) = , isso implica R(A) = . Como, por em, o grau de R e menor que m, tem-se que R deve ser identicamente nulo. Isso completa a prova.

8.3.1

O Teorema de Hamilton-Cayley

Vamos aqui demonstrar um teorema sobre matrizes que ser a usado mais adiante de v arias formas, em particular no Teorema Espectral, o chamado Teorema de Hamilton3 -Cayley4 , o qual arma que toda matriz de Mat (C, n) anula seu pr oprio polin omio caracter stico. Esse teorema fornece tamb em, como veremos, um m etodo eciente para o c alculo da inversa de matrizes. Cayley e Hamilton demonstraram casos particulares do teorema para matrizes 2 2, 3 3 (Cayley) e 4 4 (Hamilton). A primeira demonstra ca o geral e devida a Frobenius5 . Cayley, Hamilton e Sylvester6 est ao entre os 7 fundadores modernos da teoria das matrizes . Teorema 8.3 (Teorema de Hamilton-Cayley) Seja A Mat (C, n) e seja pA (x) = det(x A) o polin omio caracter stico de A (e que tem grau n). Ent ao, pA (A) = .

aveis o Teorema 8.3 pode ser provado elementarmente usando o Teorema Espectral, Coment ario. No caso particular de matrizes diagonaliz como indicado no Exerc cio E. 8.20, p agina 324.

onde 1 , , n s ao os autovalores de A. Mas a pr opria rela ca o Ay = 0 indica que um dos autovalores e igual a zero. Logo pA (A)y = 0. Mais genericamente, se y = 0 e {y, Ay } n ao for um conjunto de vetores linearmente independentes, ent ao Ay e y s ao proporcionais, ou seja, existe um autovalor, digamos, n tal que Ay = n y . Nesse caso tamb em tem-se
n1

Prova do Teorema 8.3. Desejamos mostrar que para todo vetor y Cn vale pA (A)y = 0. Se y = 0 isso e trivial. Se y = 0 mas com Ay = 0 ent ao pA (A)y = (1)n 1 n y ,

pA (A)y =
i=1
3 Sir 4 Arthur

(A i ) (A n )y = 0 ,

William Rowan Hamilton (18051865). Cayley (18211895). 5 Ferdinand Georg Frobenius (18491917) 6 James Joseph Sylvester (18141897). 7 Muitos certamente se surpreender ao muit ssimo em saber que, apesar de suas diversas e importantes contribui co es ` a Matem atica, Cayley e Sylvester eram originalmente advogados.

JCABarata. Curso de F sica-Matem atica

Vers ao de 19 de junho de 2011.

Cap tulo 8

316/1943

Seja ent ao y daqui por diante um vetor xado, n ao-nulo e tal que {y, Ay } e um conjunto de dois vetores n ao-nulos e linearmente independentes. Como o espa co Cn tem dimens ao n, nem todos os conjuntos de vetores da forma {y, Ay, A2 y, . . . , Aj y } s ao formados por vetores n ao-nulos linearmente independentes. Por exemplo, se j n, o conjunto {y, Ay, A2 y, . . . , Aj y } n ao pode ser formado por vetores n ao-nulos linearmente independentes pois seu n umero excede a dimens ao do espa co. Seja k o maior n umero tal que {y, Ay, A2 y, . . . Ak1 y } e um conjunto de vetores n ao-nulos e linearmente indepen claro que 1 < k n. dentes. E claro tamb E em, pela deni ca o de k , que Ak y = hk y + hk1 Ay + + h1 Ak1 y , para constantes h1 , . . . , hk . Vamos denominar z1 = Ak1 y , z2 = Ak2 y , . . . , zk = y , ou seja, zj = Akj y , j = 1, . . . , k , todos n ao-nulos por hip otese. Caso k < n, escolhamos ainda vetores zk+1 , . . . , zn de modo que o conjunto {z1 , . . . , zn } forme uma base em Cn . Coloquemo-nos agora a seguinte quest ao: qual e a forma da matriz A nessa base? No subespa co gerado pelos vetores {z1 , . . . , zk } tem-se o seguinte: para i = 2, . . . , k vale Azi = zi1 . Al em disso, por (8.35), Az1 = h1 z1 + h2 z2 + + hk zk . Isso mostra que o subespa co gerado pelos vetores {z1 , . . . , zk } e invariante pela a ca o de A e o operador linear A, no mesmo subespa co, tem a forma h1 h 2 . . . h k 2 h k 1 hk 1 0 ... .. .. .. . . . 0 0 . . . 0 0 0 1 .. . 0 .. 1 0 0 . 0 0 . . . . 0 1 0 (8.35)

pois (A n )y = Ay n y = 0.

(8.36)

0 0 0

... ...

E. 8.15 Exerc cio. Justique isso. Se designarmos por P o operador que realiza essa mudan ca de base, o operador linear A na base {z1 , . . . , zn } tem, portanto, a forma A = P 1 AP , onde onde A1 e a matriz k k denida em (8.36), A2 e uma matriz (n k ) k e A3 e uma matriz (n k ) (n k ). N ao nos ser a necess ario especicar os elementos das matrizes A2 e A3 . Outros segundos (minutos?) de medita ca o, usando a Proposi ca o 8.3 da p agina 304, nos levam a concluir que o polin omio caracter stico pA pode ser escrito como pA (x) = det(x A ) = det(x A1 ) det(x A3 ) . O estudante deve recordar-se que as matrizes A e A , por serem similares, t em o mesmo polin omio caracter stico (Proposi ca o 8.5, p agina 306). A1 A = A2

k, nk
A3

JCABarata. Curso de F sica-Matem atica

Vers ao de 19 de junho de 2011.

Cap tulo 8

317/1943

ao ser a necess ario, Vamos denominar qk (x) = det(x A1 ) e rk (x) = det(x A3 ). Claramente, pA (x) = qk (x)rk (x). N no que segue, calcular rk , mas precisaremos calcular qk . Como esse pequeno resultado tem interesse independente, vamos formul a-lo como um lema, para futura refer encia. Lema 8.1 Para h1 , . . . , hk C, tem-se x h1 h 2 . . . qk (x) := det h k 2 h k 1 hk

1 x

0 1 .. .

... .. .. .. . . .

0 0 0

0 0 0

... ...

0 0 0 . .. . . . = xk (h1 xk1 + + hk1 x + hk ) . 1 0 x 1 0 x

(8.37)

Prova. A prova e feita por indu ca o. Para k = 2 vale

Para k > 2, tem-se, pelas bem conhecidas regras de c alculo de determinantes, qk (x) x h1 h 2 . . = x det . h k 2 hk 1 1 x 0 .. . . 0 .. . .. 0 0 ... 0 0 1 x

x h1 q2 (x) = det h2

1 = x2 h1 x h2 . x

x 0

(k1)(k1)

x h1 h 2 . . + 1 det . h k 2 hk

1 x

0 .. .. . . 0 .. .

0 0 ...

x 0

0 0 1 0

(k1)(k1)

0 1 0 . . . 0 x 1 . . . 0 0 . . . . k1+1 . . . . = xqk1 (x) + (1) (hk ) det . . . . .. 0 . 1 0 0 0 0 . . . x 1 = xqk1 (x) + (1)k+1 hk (1)k2 = xqk1 (x) hk .

(k2)(k2)

(8.38)

JCABarata. Curso de F sica-Matem atica

Vers ao de 19 de junho de 2011.

Cap tulo 8

318/1943

E. 8.16 Exerc cio. Complete os detalhes. Assim, se pela hip otese indutiva qk1 e da forma qk1 (x) = xk1 (h1 xk2 + + hk2 x + hk1 ) , segue de (8.38) que qk (x) = x(xk1 (h1 xk2 + + hk2 x + hk1 )) hk = xk (h1 xk1 + + hk2 x2 + hk1 x + hk ) , como quer amos provar. Retomando, temos que pA (A)y = qk (A)rk (A)y = rk (A)qk (A)y . Sucede, por em, que qk (A)y = 0. De fato, pelo c omputo acima, qk (A)y = Ak y h1 Ak1 y hk2 A2 y hk1 Ay hk y , (8.39)

que e igual a zero por (8.35). Logo pA (A)y = 0. Como y foi escolhido arbitr ario, segue que pA (A) = , demonstrando o Teorema de Hamilton-Cayley, Teorema 8.3.

O Teorema de Hamilton-Cayley e a inversa de matrizes

O Teorema de Hamilton-Cayley fornece-nos um m etodo de calcular a inversa de matrizes n ao-singulares. De fato, se pA (x) = xn + an1 xn1 + + a1 x + a0 e o polin omio caracter stico de uma matriz n ao-singular A, ent ao o Teorema de Hamilton-Cayley arma que An + an1 An1 + + a1 A + a0 = , ou seja, Isso tem por implica ca o A1 = Nota. A An1 + an1 An2 + + a2 A + a1 = a0 . (8.40)

1 An1 + an1 An2 + + a2 A + a1 . a0

Vide (8.149), p agina 382, para uma express ao mais expl cita.
Usando a deni ca o de polin omio caracter stico pA (x) = det(x A), e evidente (tomando-se x = 0) que a0 = (1)n det(A). Assim, a0 = 0 se e somente se A for n ao-singular.

Em muitos casos a f ormula (8.40) e bastante eciente para calcular A1 , pois a mesma envolve poucas opera co es alg ebricas em compara ca o com outros m etodos, o que e uma vantagem para valores grandes de n. Compare, por exemplo, com a regra de Laplace, express ao (8.19), p agina 300, para o c alculo de A1 , que envolve o c omputo de n2 + 1 determinantes de sub-matrizes de ordem n 1 de A. etodo para calcular a inversa das suas matrizes n ao-singulares favoritas. E. 8.17 Exerc cio. Use esse m De volta ao polin omio m nimo

O Teorema 8.2, p agina 314, e o Teorema de Hamilton-Cayley, juntos, permitem-nos precisar algo a respeito da forma geral do polin omio m nimo de uma matriz. Se A Mat (C, n) tem r autovalores distintos 1 , . . . , r , cada qual com multiplicidade alg ebrica a1 , . . . , ar , respectivamente, ent ao seu polin omio caracter stico pA e da forma
r

pA (x) =
k=1

(x k )ak .

JCABarata. Curso de F sica-Matem atica

Vers ao de 19 de junho de 2011.

Cap tulo 8

319/1943

Pelo Teorema de Hamilton-Cayley, pA (A) = 0 e, portanto, pelo Teorema 8.2, M , o polin omio m nimo de A, divide q . Logo, M deve ser da forma
s

M (x) =
l=1

(x kl )bl ,

(8.41)

em, vm = 0 um onde s r, {k1 , . . . , ks } {1 , . . . , r } e onde 0 < bl akl para todo 1 l s. Seja agora, por autovetor de A com autovalor m Segue do fato que M (A) = 0 que
s s

0 = M (A)vm =
l=1 s l=1

(A kl )bl vm =

l=1

(m kl )bl vm .

Logo, (m kl )bl = 0 e isso implica que m {k1 , . . . , ks }. Como isso vale para todo 1 m r, segue que ao e resumida no seguinte: {1 , . . . , r } {k1 , . . . , ks } e, portanto, {1 , . . . , r } = {k1 , . . . , ks }. Nossa conclus Proposi c ao 8.15 Seja A Mat (C, n) com r autovalores distintos 1 , . . . , r C, cada qual com multiplicidade alg ebrica a1 , , . . . , ar , sendo 1 r n. Ent ao, M , o polin omio m nimo de A, e da forma
r

M (x) =
k=1

(x k )bk ,

(8.42)

x C, onde 0 < bl al para todo 1 l r. Em particular, se A Mat (C, n) tiver exatamente n autovalores distintos, teremos que bl = al = 1 para todo 1 l n, e
n

M (x) = pA (x) =
k=1

(x k ) ,

x C.

8.4

Matrizes Diagonaliz aveis e o Teorema Espectral

Vamos agora apresentar uma no ca o intimamente ligada ` a de matriz simples introduzida acima (p agina 310), mas de import ancia maior. e dita ser uma matriz diagonaliz avel se existir uma matriz invers vel P Deni c ao. Uma matriz A Mat (C, n) Mat (C, n) tal que P 1 AP e uma matriz diagonal, ou seja, d1 . 1 . P AP = D = diag (d1 , . . . , dn ) = . 0 .. . 0 . . . . dn

Matrizes diagonaliz aveis

f E acil de se ver que os elementos da diagonal de D s ao os autovalores de A. De fato, se A e diagonaliz avel por P , vale para seu polin omio caracter stico p() = det( A) = det(P 1 ( A)P ) = det( P 1 AP ) = det( D) d1 . . = det . 0 .. . 0 . . . dn = ( d1 ) ( dn ) ,

JCABarata. Curso de F sica-Matem atica

Vers ao de 19 de junho de 2011.

Cap tulo 8

320/1943

o que mostra que os di s ao as ra zes do polin omio caracter stico de A e, portanto, seus autovalores. E. 8.18 Exerc cio. Justique todas as passagens acima. Diagonaliza c ao de matrizes

O pr oximo teorema e fundamental no estudo de matrizes diagonaliz aveis.

Teorema 8.4 Uma matriz A Mat (C, n) e diagonaliz avel se e somente se possuir um conjunto de n autovetores linearmente independentes, ou seja, se e somente se o subespa co gerado pela cole c ao de todos os autovetores de A possuir dimens ao n.

Prova. Vamos primeiro provar que se A Mat (C, n) possui um conjunto de n autovetores linearmente independentes ent ao A e diagonaliz avel. Para tal vamos construir a matriz P que diagonaliza A. Seja {v 1 , . . . , v n } um conjunto de n autovetores linearmente independentes de A, cujos autovalores s ao {d1 , . . . , dn }, i respectivamente. Vamos denotar as componentes de v i na base can onica por vj , j = 1, . . . , n. Seja a matriz P denida por P = v 1 , . . . , v n , ou seja,
1 v1

Como se v e pela constru ca o, a a- esima coluna de P e formada pelas componentes do vetor v a . Por (8.12), segue que AP = Por (8.14) vale, por em, que
1 v1 . . = . 1 vn

. . P = . 1 vn

.. .

n v1

. . . . n vn

Av 1 , . . . , Av n

d1 v 1 , . . . , dn v n .

d1 v 1 , . . . , dn v n

.. .

n v1 d1 . . . . . . n vn 0

.. .

0 . . . = PD . dn

E. 8.19 Exerc cio. Verique. Portanto, AP = P D. Como, por hip otese, as colunas de P s ao formadas por vetores linearmente independentes, tem-se que det(P ) = 0 (por que?). Logo, P e invers vel e, portanto, P 1 AP = D, como quer amos demonstrar. Vamos provar agora a arma ca o rec proca que se A e diagonaliz avel, ent ao possui n autovetores linearmente independentes. Suponha que exista P tal que d1 . 1 . P AP = D = . 0 .. . 0 . . . . dn

JCABarata. Curso de F sica-Matem atica

Vers ao de 19 de junho de 2011.

Cap tulo 8

321/1943

evidente que os vetores da base can E onica e1

s ao autovetores de D com Dea = da ea . Logo, v a = P ea s ao autovetores de A, pois

1 0 = 0 , . . . 0

e2

0 1 = 0 , . . . 0

...,

en

0 0 . . = . 0 1

Av a = AP ea = P Dea = P (da ea ) = da P ea = da v a . Para provar que os vetores v a s ao linearmente independentes, suponha que existam n umeros complexos 1 , . . . , n tais que 1 v 1 + + n v n = 0. Multiplicando-se ` a esquerda por P 1 ter amos 1 e1 + + n en = 0. Como os ea s ao obviamente linearmente independentes, segue que 1 = = n = 0. Matrizes diagonaliz aveis e matrizes simples

Vamos agora discutir a rela ca o entre os conceitos de matriz diagonaliz avel e o de matriz simples, conceito esse introduzido ` a p agina 310. Tem-se a saber o seguinte fato: Proposi c ao 8.16 Uma matriz A Mat (C, n) e diagonaliz avel se e somente se for simples, ou seja, se e somente se a multiplicidade alg ebrica de cada um dos seus autovalores coincidir com sua multiplicidade geom etrica. Prova. Se A e diagonaliz avel existe P tal que P 1 AP = D, diagonal. Como toda matriz diagonal, D e simples. Escrevamos D na forma Um conjunto de n-autovetores de D linearmente independentes e fornecido pelos vetores da base can onica: e1 1 0 = 0 , . . . 0 e2 0 1 = 0 , . . . 0 ..., en 0 0 . . = . . 0 1 D = diag 1 , . . . , 1 , . . . , r , . . . , r , .
a1 vezes ar vezes

co de autovetores com autovalor 1 de D etc. Os vetores e1 , . . . , ea1 geram o subespa

claro que a co de autovetores com autovalor 1 etc. E Para a matriz A, os vetores P e1 , . . . , P ea1 geram o subespa ao linearmente independentes, j a que os vetores da base can onica dimens ao desse subespa co e a1 , pois P e1 , . . . , P ea1 s ao. Como isso tamb em vale para os demais autovalores conclu mos que A e simples. e1 , . . . , ea1 o s Resta-nos agora mostrar que se A Mat (C, n) e simples ent ao A e diagonaliz avel. Como antes, sejam 1 , . . . , r , ebrica a1 , . . . , ar , respectivamente, e seja E(i ) 1 r n, seus autovalores distintos, cada qual com multiplicidade alg o subespa co gerado pelos autovetores com autovalor i . Como A e simples, tem-se que a dimens ao de E(i ) e ai . J a observamos (p agina 308) que subespa cos E(i ) associados a autovalores distintos t em em comum apenas o vetor nulo.

JCABarata. Curso de F sica-Matem atica

Vers ao de 19 de junho de 2011.

Cap tulo 8

322/1943

Assim, se em cada E(i ) escolhermos ai vetores independentes, teremos ao todo um conjunto de i=1 ai = n autovetores (vide (8.28)) linearmente independentes de A. Pelo Teorema 8.4, A e diagonaliz avel, completando a prova.

Projetores

Uma matriz E Mat (C, n) e dita ser um projetor se satiszer E2 = E .

Projetores s ao tamb em denominados matrizes idempotentes. Discutiremos v arias propriedades importantes de projetores adiante, especialmente de uma classe especial de projetores denominados projetores ortogonais. Por ora, vamos mostrar duas propriedades que usaremos logo abaixo quando discutirmos o teorema espectral. A primeira propriedade e a arma ca o que se e um autovalor de um projetor E ent ao ou e igual a zero ou a um. De fato se v e um autovetor associado a um autovalor de E , tem-se que Ev = v e E 2 v = 2 v . Como E 2 = E , segue que 2 v = v . Logo ( 1) = 0 e, portanto, = 0 ou = 1.

A segunda propriedade e uma conseq u encia da primeira: o tra co de um projetor E Mat (C, n) e um n umero inteiro positivo ou nulo, mas menor ou igual a n. De fato, pela deni ca o, o tra co de um projetor E e a soma de seus autovalores. Como os mesmos valem zero ou um a soma e um inteiro positivo ou nulo. Como h a no m aximo n autovalores a soma n ao pode exceder n. Na verdade, o u nico projetor cujo tra co vale exatamente n e a identidade e o u nico projetor cujo tra co vale exatamente 0 e a matriz nula (por que?). Essas observa co es t em a seguinte conseq u encia que usaremos adiante. Se E1 , . . . , Er s ao r projetores n ao-nulos com a propriedade que
r

=
a=1

Ea

ent ao r n. Para ver isso, basta tomar o tra co de ambos os lados dessa express ao:
r

Tr() =
a=1

Tr(Ea ) .

(8.43)

O lado esquerdo vale n enquanto que o lado direito e uma soma de r inteiros positivos. Obviamente isso s o e poss vel se r n.

ao E + E e Uma outra observa ca o u til e a seguinte: se E e E s ao dois projetores satisfazendo EE = E E = , ent igualmente um projetor, como facilmente se constata. O Teorema Espectral

O chamado Teorema Espectral e um dos mais importantes teoremas de toda a Algebra Linear e, em verdade, de toda An alise Funcional, j a que o mesmo possui generaliza co es para operadores limitados e n ao-limitados (auto-adjuntos) agindo em espa cos de Hilbert. Dessas generaliza co es trataremos na Se ca o 35.8.2, p agina 1833, para o caso dos chamados operadores compactos e na Se ca o 35.9, p agina 1839, para o caso geral de operadores limitados auto-adjuntos. Nessa vers ao mais geral o teorema espectral e de import ancia fundamental para a interpreta ca o probabil stica da F sica Qu antica. Vide discuss ao da Se ca o 35.9.5, p agina 1855. Teorema 8.5 (Teorema Espectral para Matrizes) Uma matriz A Mat (C, n) e diagonaliz avel se e somente se existirem r N, 1 r n, escalares distintos 1 , . . . , r e projetores n ao-nulos distintos E1 , . . . , Er Mat (C, n) tais que
r

A =
a=1

a Ea ,
r

(8.44)

=
a=1

Ea

(8.45)

e Ei Ej = i, j Ej .

JCABarata. Curso de F sica-Matem atica

Vers ao de 19 de junho de 2011.

Cap tulo 8

323/1943

Os escalares 1 , . . . , r v em a ser os autovalores distintos de A. Adiante demonstraremos uma vers ao um pouco mais detalhada desse importante teorema (Teorema 8.7, abaixo). Os projetores Ea que surgem em (8.44) s ao denominados projetores espectrais de A. A decomposi ca o (8.44) e freq uentemente denominada decomposi c ao espectral de A. Na Proposi ca o 8.18, p agina 325 mostraremos como os projetores espectrais Ea de A podem ser expressos em termos de polin omios em A. Na Proposi ca o 8.19, p agina 325, provaremos a unicidade da decomposi ca o espectral de uma matriz diagonaliz avel. Prova do Teorema 8.5. Se A Mat (C, n) e diagonaliz avel existe P Mat (C, n) tal que P 1 AP = D = diag (1 , . . . , n ), onde 1 , . . . , n s ao os autovalores de A. Como pode haver autovalores repetidos, vamos denotar por {1 , . . . , r }, 1 r n, o conjunto de autovalores distintos de A. bem claro que podemos escrever E
r

D =
a=1

a Ka ,

onde as matrizes Ka s ao todas matrizes diagonais, cujos elementos diagonais s ao ou 0 ou 1 e tais que
r

Ka = .
a=1

(8.46)

Por exemplo, se

As matrizes Ka s ao simplesmente denidas de modo a terem elementos de matriz iguais a 1 nas posi co es da diagonal ocupadas pelo autovalor a em D e zero nos demais. Formalmente, 1, se i = j e (D)ii = a (Ka )ij = 0, se i = j e (D)ii = a . 0, se i = j 2 0 D = 0 0 0 3 0 0 0 0 0 0 2 0 0 4 teremos 1 0 D = 2 0 0 0 0 0 0 0 0 0 0 0 0 +3 0 1 0 0 0 0 0 1 0 0 0 0 0 0 0 0 +4 0 0 0 0 0 0 0 0 0 0 0 0 0 0 . 0 0 0 1 (8.47)

f E acil constatar que as matrizes Ka t em a seguinte propriedade:

Ka Kb = a, b Ka .

De fato, e evidente que (Ka )2 = Ka para todo a, pois Ka e diagonal com zeros ou uns na diagonal. Analogamente, se a = b Ka Kb = 0, pois os zeros ou uns aparecem em lugares distintos das diagonais das duas matrizes. Como A = P DP 1 , tem-se que A =
a=1 r r

a Ea ,

f acil agora provar que = onde Ea := P Ka P 1 . E


a=1 r r

Ea e que Ei Ej = i, j Ej . De fato, por (8.46),


r

Ea =
a=1 a=1

P K a P 1 = P
a=1

Ka

P 1 = P P 1 = .

JCABarata. Curso de F sica-Matem atica

Vers ao de 19 de junho de 2011.

Cap tulo 8

324/1943

Analogamente, tem-se por (8.47), Ea Eb = P Ka P 1 P Kb P 1 = P Ka Kb P 1 = a, b P Ka P 1 = a, b Ea . Vamos agora provar a rec proca. Vamos supor que A possua a representa ca o (8.44), onde os Ea s satisfazem as propriedades enunciadas. Notemos primeiramente que para todo vetor x, os vetores Ek x ou s ao nulos ou s ao autovetores de A. De fato, por (8.44)
r

AEk x =
j =1

j Ej Ek x = k Ek x .

Logo ou Ek x = 0 ou Ek x e autovetor de A. Como h a no m aximo n autovetores, o espa co por eles gerado tem dimens ao menor ou igual a n. Por (8.45), por em, vale para todo vetor x que
r

x = x =
k=1

Ek x .

Para x n ao-nulo, alguns dos Ek x, acima, devem ser n ao-nulos e, portanto, autovetores de A. Assim, todo vetor x pode ser escrito como uma combina ca o linear de autovetores de A, o que signica que o espa co gerado por esses autovetores tem dimens ao exatamente igual a n. Pelo Teorema 8.4, p agina 320, A e diagonaliz avel. Isso completa a demonstra ca o.

No Teorema 8.7, p agina 328, apresentaremos uma segunda demonstra ca o do Teorema Espectral para Matrizes, a qual lan ca luz sobre outras condi co es de diagonalizabilidade de matrizes. Antes, exploremos algumas das conseq u encias do Teorema Espectral. O C alculo Funcional para matrizes diagonaliz aveis

O Teorema Espectral tem o seguinte corol ario, muitas vezes conhecido como c alculo funcional.

Teorema 8.6 (C alculo Funcional) Seja A Mat (C, n) uma matriz diagonaliz avel e seja
r

A =
a=1

a Ea

sua decomposi c ao espectral, de acordo com o Teorema Espectral, o Teorema 8.5. Ent ao, para qualquer polin omio p vale
r

p(A) =
a=1

p(a )Ea .

(8.48)

Prova. Tem-se, pelas propriedades dos Ea s, A2 =


a, b=1 r

a b Ea Eb =
a, b=1

a b a, b Ea =

(a )2 Ea . Analogamente,
a=1

mostra-se que Am =

a=1

(a )m Ea , para qualquer m N. O resto da prova e trivial.

E. 8.20 Exerc cio. Usando (8.48) demonstre novamente o Teorema de Hamilton-Cayley (Teorema 8.3, p agina 315), agora apenas para o caso particular de matrizes diagonaliz aveis. Por simples constata ca o verica-se tamb em facilmente a validade do seguinte resultado, que usaremos diversas vezes:

JCABarata. Curso de F sica-Matem atica

Vers ao de 19 de junho de 2011.

Cap tulo 8

325/1943

Proposi c ao 8.17 Seja A Mat (C, n) uma matriz diagonaliz avel e invers vel e seja A = espectral, de acordo com o Teorema Espectral, o Teorema 8.5. Ent ao, A1 = 1 Ea . a=1 a
r

a Ea sua decomposi c ao
a=1

O C alculo Funcional para matrizes, Teorema 8.6, tem diversas conseq u encias pr aticas, uma delas sendo a seguinte proposi ca o, que permite expressar os projetores espectrais de uma matriz A diretamente em termos de A e seus autovalores. Proposi c ao 8.18 Seja A Mat (C, n), n ao-nula e diagonaliz avel, e seja A = 1 E1 + + r Er , com os k s distintos, sua representa c ao espectral, descrita no Teorema 8.5. Sejam os polin omios pj , j = 1, . . . , r, denidos por
r

Obtendo os projetores espectrais

pj (x) :=
l=1 l= j

x l j l

(8.49)

Ent ao,

para todo j = 1, . . . , r.

Ej = pj (A) =

k=1 k =j

1 A l j k l=1
l= j

(8.50)

Prova. Pela deni ca o dos polin omios pj , e evidente que pj (k ) = j, k . Logo, pelo C alculo Funcional para matrizes,
r

pj (A) =
k=1

pj (k )Ek = Ej .

O Teorema Espectral para matrizes. Unicidade Proposi c ao 8.19 A representa c ao espectral de uma matriz diagonaliz avel A Mat (C, n) descrita no Teorema 8.5 e u nica.

Demonstra c ao. Seja A Mat (C, n) diagonaliz avel e seja A =

k Ek a representa ca o espectral de A descrita no


k=1 r k Ek uma segunda k=1

Teorema 8.5, onde k , k = 1, . . . , r, com 1 r n s ao os autovalores distintos de A, Seja A =


r r k=1 r

representa ca o espectral para A, onde os ao distintos e onde os Ek s s ao n ao-nulos e satisfazem Ej El = j, l El e k s s

=
k=1

Ek . Por essa u ltima propriedade segue que para um dado vetor x = 0 vale x =

Ek x, de modo que nem todos

os vetores Ek x s ao nulos. Seja Ek x um desses vetores n ao-nulos. Tem-se que AEk x = k=1 k Ek Ek0 x = k0 Ek0 x. Isso 0 0 mostra que k0 e um dos autovalores de A e, portanto, {1 , . . . , r } {1 , . . . , r }. Isso, em particular ensina-nos que r r. Podemos sem perda de generalidade considerar que os dois conjuntos sejam ordenados de modo que k = k para todo 1 k r . Assim, r r

A =
k=1

k Ek =
k=1

k Ek .

(8.51)

JCABarata. Curso de F sica-Matem atica

Vers ao de 19 de junho de 2011.

Cap tulo 8

326/1943

Sejam agora os polin omios pj , j = 1, . . . , r, denidos em (8.49), os quais satisfazem pj (j ) = 1 e pj (k ) = 0 para todo k = j . Pelo C alculo Funcional descrito acima, segue de (8.51) que, com 1 j r ,
r r

pj (A) =
k=1

pj (k )Ek =
k=1 =E j

, pj (k )Ek
=E j

Ej = Ej .

segue do fato que os Ek s satisfazem as mesmas rela co es alg ebricas que os Ek s e, (A igualdade pj (A) = k=1 pj (k )Ek portanto, para a representa ca o espectral de A em termos dos Ek s vale tamb em o C alculo Funcional). Lembrando que a r r r

igualdade Ej =

Ej

vale para todo 1 j r , segue que =

Ek =
k=1 k=1

Ek . A u ltima igualdade implica


k=r +1

Ek = .

Multiplicando por El com r + 1 l r, segue que El = para todo r + 1 l r. Isso s o e poss vel se r = r , pois os E k s s ao n ao-nulos. Isso completa a demonstra ca o.

Algumas outras identidades decorrentes do Teorema Espectral Proposi c ao 8.20 Seja A Mat (C, n) uma matriz diagonaliz avel e invert vel, cujos autovalores distintos sejam {1 , . . . , r }, para algum 1 r n. Ent ao, vale a identidade
r

para cada k {1, . . . , r}.

j =1 j =k

1 k j

l=1 l= k

A l

j =1 j =k

1 1 1 k j

l=1 l= k

1 A1 l ,

(8.52)

Observe-se tamb em que (8.52) implica


r 1 1 k l r

l=1 l= k

A l
r

=
l=1 l= k

k l
r

1 A1 l

(8.53)

2 r k

(1)r1
r j =1

l=1 l= k

A l

=
l=1 l= k

1 A1 l .

(8.54)

Prova da Proposi c ao 8.20. Pelo Teorema Espectral e por (8.50) podemos escrever A em sua representa ca o espectral:
r r

A =

k=1

Se A e tamb em invert vel, a Proposi ca o 8.17, p agina 325, informa-nos que


r

j =1 j =k

1 k j

l=1 l= k

A l .

(8.55)

A1 =

k=1

1 1 Por outro lado, se A e invert vel, A1 e diagonaliz avel (justique!), e seus autovalores distintos s ao { 1 , . . . , r } 1 (justique!). Logo, a representa ca o espectral de A e r r

1 1 k j=1 k j
j =k

l=1 l= k

A l .

A1 =

k=1

1 k

j =1 j =k

1 1 1 k j

l=1 l= k

1 A1 l ,

JCABarata. Curso de F sica-Matem atica

Vers ao de 19 de junho de 2011.

Cap tulo 8

327/1943

onde as matrizes

j =1 j =k

1 1 1 k j

l=1 l= k

1 ao os projetores espectrais de A1 . Aplicando novamente a A1 l s

Proposi ca o 8.17, obtemos


r

A =
k=1

Comparando (8.55) a (8.56) e evocando a unicidade da representa ca o espectral de A, conclu mos pela validade de (8.52) para cada k {1, . . . , r}. E. 8.21 Exerc cio. Seja A Mat (C, n) uma matriz diagonaliz avel e invert vel com apenas dois autovalores distintos, 1 e 2 . Usando (8.52) ou (8.54) mostre que A1 = 1 1 2 1 + 2 A . (8.57)

j =1 j =k

1 1 1 j k

r 1 A1 l .

(8.56)

l=1 l= k

Essa rela c ao n ao e geralmente v alida para matrizes n ao-diagonaliz aveis e invert veis com apenas dois autovalores distintos. A 11 0 matriz 0 1 0 tem autovalores +1 e 1, e invert vel, n ao e diagonaliz avel e n ao satizfaz (8.57). Verique! Prove (8.57) 0 0 1 diretamente do Teorema Espectral. O Teorema Espectral para matrizes. Uma segunda visita

O Teorema Espectral, Teorema 8.5, pode ser formulado de um modo mais detalhado (Teorema 8.7). A principal utilidade dessa outra formula ca o e a de fornecer mais informa co es sobre os projetores espectrais Ea (vide express ao (8.60), abaixo). Obt em-se tamb em nessa nova formula ca o mais condi co es necess arias e sucientes ` a diagonalizabilidade e que podem ser u teis, como veremos, por exemplo, no Teorema 8.22 provado adiante (p agina 331). No teorema a seguir e em sua demonstra ca o seguimos parcialmente [62]. Teorema 8.7 (Teorema Espectral para Matrizes. Vers ao Detalhada) Seja A Mat (C, n). S ao equivalentes as seguintes arma c oes: 1. A possui n autovetores linearmente independentes, ou seja, o subespa co gerado pelos autovetores de A tem dimens ao n. 2. A e diagonaliz avel, ou seja, existe uma matriz P Mat (C, n) invers vel tal que P 1 AP e uma matriz diagonal diag (d1 , . . . , dn ), onde os di s s ao autovalores de A. 3. Para todo vetor x Cn e todo escalar C tais que (A )2 x = 0, vale que (A )x = 0.

4. Se x e um vetor n ao-nulo tal que (A )x = 0 para algum C ent ao n ao existe nenhum vetor y com a propriedade que (A )y = x. 5. Todas as ra zes do polin omio m nimo de A t em multiplicidade 1. 6. Existem r N, escalares distintos 1 , . . . , r e projetores distintos E1 , . . . , Er Mat (C, n), denominados projetores espectrais de A, tais que
r

A =
a=1

a Ea .

Al em disso, as matrizes Ea satisfazem

=
a=1

Ea

(8.58)

e Ei Ej = i, j Ej . (8.59)

JCABarata. Curso de F sica-Matem atica

Vers ao de 19 de junho de 2011.

Cap tulo 8

328/1943

Os projetores espectrais Ek do item 6, acima, podem ser expressos em termos de polin omios da matriz A: Ek = 1 mk (A) , mk (k ) (8.60)

para todo k , 1 k r, onde os polin omios mk s ao denidos por M (x) = (x k )mk (x) , M sendo o polin omio m nimo de A.

Demonstra c ao. A prova da equival encia ser a feita demonstrando-se sucessivamente as seguintes implica co es: 1 2, 2 3, 3 4, 4 5, 5 6, 6 1. Que 1 implica 2 j a foi demonstrado no Teorema 8.4, p agina 320. 2 3. Seja D = P 1 AP diagonal. D = diag (d1 , . . . , dn ). Seja (A )2 x = 0. Segue que P 1 (A )2 P y = 0 onde y = P 1 x. Logo, (D )2 y = 0 ,

y1 yn

ou seja, (dj )2 yj = 0, j = 1, . . . , n, onde yj s ao as componentes de y : y = (da )2 ya = 0 ent ao (da )ya = 0. Logo (D )y = 0 .

. . .

. Agora, e evidente que se

Usando-se y = P 1 x e multiplicando-se ` a direita por P , conclu mos que 0 = P (D )P 1 x = (P DP 1 )x = (A )x , que e o que quer amos provar. 3 4. A prova e feita por contradi ca o. Vamos supor que para algum vetor x = 0 exista C tal que (A )x = 0. amos Suponhamos tamb em que exista vetor y tal que (A )y = x. Ter (A )2 y = (A )x = 0 . ca o. Pelo item 3 isso implica (A )y = 0. Mas isso diz que x = 0, uma contradi 4 5. Seja M o polin omio m nimo de A, ou seja, o polin omio m onico8 de menor grau tal que M (A) = 0. Vamos mostrar que todas as ra zes de M t em multiplicidade 1. Vamos, por contradi ca o, supor que haja uma raiz, 0 , com multiplicidade maior ou igual a 2. Ter amos, para x C, M (x) = p(x)(x 0 )2 . e, por deni ca o, o polin omio de menor grau que zera em A, segue Assim, M (A) = p(A)(A 0 )2 = 0. Como M que p(A)(A 0 ) = 0 .

Assim, existe pelo menos um vetor z tal que p(A)(A 0 )z = 0. Vamos denir um vetor x por x := p(A)(A 0 )z . Ent ao, (A 0 )x = (A 0 )p(A)(A 0 )z = p(A)(A 0 )2 z = M (A)z = 0 , pois M (A) = 0. Agora, pela deni ca o, x = (A 0 )y ,

onde y = p(A)z . Pelo item 4, por em, isso e imposs vel.


8A

deni ca o de polin omio m onico est a` a p agina 314.

JCABarata. Curso de F sica-Matem atica

Vers ao de 19 de junho de 2011.

Cap tulo 8

329/1943

5 6. Pela hip otese que as ra zes de M s ao simples segue da express ao (8.42) da Proposi ca o 8.15, p agina 319, que para x C,
r

M (x) =

j =1

(x j ) ,

onde j s ao as ra zes de M e que coincidem com os r autovalores distintos de A. Para k = 1, . . . , r dena-se os polin omios mk por M (x) =: (x k )mk (x) , ou seja, mk (x) :=
j =1 j =k

(x j ) .

Vamos agora denir mais um polin omio, g , da seguinte forma:


r

claro que mk (j ) = 0 j = k (por que?). E

g (x) = 1

k=1

1 mk (x) . mk (k )

Como os polin omios mk t em grau r 1, o polin omio g tem grau menor ou igual a r 1. Por em, observe-se que, para todos os j , j = 1, . . . , r, vale
r

g (j ) = 1

k=1

mj (j ) 1 mk (j ) = 1 = 0. mk (k ) mj (j )

Assim, g tem pelo menos r ra zes distintas! O u nico polin omio de grau menor ou igual a r 1 que tem r ra zes distintas e o polin omio nulo. Logo, conclu mos que
r

g (x) = 1

k=1

1 mk (x) 0 mk (k )

para todo x C. Isso signica que todos os coecientes de g s ao nulos. Assim, para qualquer matriz B tem-se g (B ) = 0. Para a matriz A isso diz que r 1 = mk (A) . mk (k )
k=1

Denindo-se Ek := conclu mos que

1 mk (A) , mk (k )
r

(8.61)

=
k=1

Ek .

(8.62)

ca o de Ek isso signica Para todo k vale 0 = M (A) = (A k )mk (A), ou seja, Amk (A) = k mk (A). Pela deni AEk = k Ek . Assim, multiplicando-se ambos os lados de (8.62) por A, segue que
r

A =
k=1

k Ek .

Para completar a demonstra ca o de 6, resta-nos provar que Ei Ej = i, j Ej .

JCABarata. Curso de F sica-Matem atica

Vers ao de 19 de junho de 2011.

Cap tulo 8

330/1943

Para i = j tem-se pela deni ca o dos Ek s que Ei Ej = 1 mi (A)mj (A) mi (i )mj (j ) 1 (A k ) (A l ) mi (i )mj (j ) k=1 l=1
k =i l= j

1 mi (i )mj (j ) 1 mi (i )mj (j ) 0,

k=1 k=i, k=j

(A k )

r l=1

(A l )

k=1 k=i, k=j

(A k ) M (A)

2 pois M (A) = 0. Resta-nos provar que Ej = Ej para todo j . Multiplicando-se ambos os lados de (8.62) por Ej teremos r

Ej =
k=1

Ej Ek = Ej Ej ,

j a que Ej Ek = 0 quando j = k . Isso completa a demonstra ca o do item 6. 6 1. Notemos primeiramente que para todo vetor x, os vetores Ek x ou s ao nulos ou s ao autovetores de A. De fato, por 6,
r

AEk x =
j =1

j Ej Ek x = k Ek x .

Logo, ou Ek x = 0 ou Ek x e autovetor de A. O espa co gerado pelos autovetores de A obviamente tem dimens ao menor ou igual a n. Por (8.62), por em, vale para todo vetor x que
r

x = x =
k=1

Ek x .

Assim, todo vetor x pode ser escrito como uma combina ca o linear de autovetores de A, o que signica que o espa co gerado pelos autovetores tem dimens ao exatamente igual a n. Isso completa a demonstra ca o do Teorema 8.7. Destacamos ao leitor o fato de que a express ao (8.60) permite representar os projetores espectrais diretamente em termos da matriz diagonaliz avel A. Diagonalizabilidade de projetores

A proposi ca o abaixo e uma aplica ca o simples do Teorema 8.7 a projetores. A mesma ser a usada abaixo quando falarmos de diagonaliza ca o simult anea de matrizes. Proposi c ao 8.21 Seja E Mat (C, n) um projetor, ou seja, tal que E 2 = E . Ent ao, E e diagonaliz avel. Prova. Seja E Mat (C, n) um projetor. Denamos E1 = E e E2 = E . Ent ao, E2 e tamb em um projetor, pois (E2 )2 = ( E )2 = 2E + E 2 = 2E + E = E = E2 .

JCABarata. Curso de F sica-Matem atica

Vers ao de 19 de junho de 2011.

Cap tulo 8

331/1943

e obvio que = E1 + E2 e Tem-se tamb em que E1 E2 = 0, pois E1 E2 = E ( E ) = E E 2 = E E = 0. Fora isso, que E = 1 E1 + 2 E2 , com 1 = 1 e 2 = 0. Ora, isso tudo diz que E satisfaz precisamente todas as condi co es do item 6 do Teorema 8.7. Portanto, pelo mesmo teorema, E e diagonaliz avel.

Uma condi c ao suciente para diagonalizabilidade

At e agora estudamos condi co es necess arias e sucientes para que uma matriz seja diagonaliz avel. Vimos que uma matriz A Mat (C, n) e diagonaliz avel se e somente se for simples ou se e somente se tiver n autovetores linearmente independentes ou se e somente se puder ser representada na forma espectral, como em (8.44). Nem sempre, por em, e imediato vericar essas hip oteses, de modo que eu til saber de condi co es mais facilmente veric aveis e que sejam pelo menos sucientes para garantir diagonalizabilidade. Veremos abaixo que e, por exemplo, suciente que uma matriz seja auto-adjunta ou normal para garantir que ela seja diagonaliz avel. Uma outra condi ca o u til e aquela contida na seguinte proposi ca o. Proposi c ao 8.22 Se A Mat (C, n) tem n autovalores distintos ent ao A e diagonaliz avel. Prova. Isso e imediato pelas Proposi co es 8.9 e 8.16, das p aginas 310 e 321, respectivamente.

Observa c ao.

A condi ca o mencionada na u ltima proposi ca o e apenas suciente, pois h a obviamente matrizes diagonaliz aveis que n ao t em autovalores todos distintos.

Outra forma de provar a Proposi ca o 8.22 e a seguinte. Seja {1 , . . . , n } o conjunto dos n autovalores de A, todos distintos. O polin omio caracter stico de A e q (x) = (x 1 ) (x n ). Como as ra zes de q t em, nesse caso, multiplicidade 1, segue pela Proposi ca o 8.15, p agina 319, que o polin omio m nimo de A, M , coincide com o polin omio caracter stico de A: q (x) = M (x), x C. Logo, o polin omio m nimo M de A tem tamb em ra zes com multiplicidade 1. Assim, pelo item 5 do Teorema 8.7, p agina 328, A e diagonaliz avel. E. 8.22 Exerc cio. Demonstre a seguinte arma c ao: se os autovalores de uma matriz A s ao todos iguais, ent ao A e diagonaliz avel se e somente se for um m ultiplo de . Sugest ao: use o Teorema Espectral ou a forma geral do polin omio m nimo (8.42). Segue da armativa desse exerc cio que matrizes triangulares superiores com diagonal principal constante, ou seja, da forma 0 . . A = . 0 0 A12 . . . A1(n1) . . . A2(n1) .. 0 0 . 0 A1n A2n . , . . A(n1)n

... ...

s o s ao diagonaliz aveis se todos os elementos acima da diagonal principal forem nulos, ou seja, se Aij = 0, j > i. Naturalmente, a mesma armativa e v alida para matrizes da forma AT , triangulares inferiores com diagonal principal constante.

8.4.1

Diagonaliza c ao Simult anea de Matrizes

Uma matriz A Mat (C, n) e dita ser diagonalizada por uma matriz P Mat (C, n) se P 1 AP for uma matriz diagonal. Uma quest ao muito importante e saber quando duas matrizes diagonaliz aveis podem ser diagonalizadas por uma mesma matriz P . A resposta e fornecida no pr oximo teorema.

JCABarata. Curso de F sica-Matem atica

Vers ao de 19 de junho de 2011.

Cap tulo 8

332/1943

Teorema 8.8 (Diagonaliza c ao Simult anea de Matrizes) Duas matrizes diagonaliz aveis A e B Mat (C, n) podem ser diagonalizadas pela mesma matriz P Mat (C, n) se e somente se AB = BA, ou seja, se e somente se comutarem entre si.

Prova. A parte f acil da demonstra ca o e provar que se A e B podem ser diagonalizadas pela mesma matriz P ent ao A e B comutam entre si. De fato P 1 (AB BA)P = (P 1 AP )(P 1 BP ) (P 1 BP )(P 1 AP ) = 0, pois P 1 AP e P 1 BP s ao ambas diagonais e matrizes diagonais sempre comutam entre si (por que?). Assim, P 1 (AB BA)P = 0 e, portanto, AB = BA. Vamos agora passar a mostrar que se AB = BA ent ao ambas s ao diagonaliz aveis por uma mesma matriz P . Sejam 1 , . . . , r os r autovalores distintos de A e 1 , . . . , s os s autovalores distintos de B . Evocando o teorema espectral, A e B podem ser escritos de acordo com suas decomposi co es espectrais como
r s A i Ei i=1

A = onde, de acordo com (8.60),

B =
j =1

B j Ej ,

A Ei

e
B Ej

A B Como A e B comutam entre si e como Ei e Ej , dados em (8.63)(8.64), s ao polin omios em A e B , respectivamente, A B segue que Ei e Ej tamb em comutam entre si para todo i e todo j .

1 s (j k ) = (B k ) , k=1 k=1
s
k =j k =j

1 r (i k ) = (A k ) , k=1 k=1
r
k =i k =i

i = 1, . . . , r

(8.63)

j = 1, . . . , s .

(8.64)

Com isso, vamos denir


A B B A Qi, j = Ei Ej = Ej Ei

para i = 1, . . . , r e j = 1, . . . , s. Note-se que os Qi, j s s ao projetores pois


A 2 B 2 A B A B A B Q2 i, j = (Ei Ej )(Ei Ej ) = (Ei ) (Ej ) = Ei Ej = Qi, j .

Fora isso, e f acil ver que, Qi, j Qk, l = i, k j, l Qi, j . E. 8.23 Exerc cio. Mostre isso. Note-se tamb em que (8.65)

=
i=1 j =1

Qi, j ,

(8.66)

pois
r s r s A B Ei Ej = i=1 j =1 i=1 r A Ei

s j =1

Qi, j =
i=1 j =1

Armamos que podemos escrever A =

B = = . Ej

s A i, j Qi, j

(8.67)

i=1 j =1

JCABarata. Curso de F sica-Matem atica

Vers ao de 19 de junho de 2011.

Cap tulo 8

333/1943

e B =

s B i, j Qi, j ,

(8.68)

i=1 j =1 A B onde i, co es, j = i e i, j = j . De fato, com essas deni r s A i, j Qi, j = i=1 j =1 i=1 j =1 r s A B i Ei Ej = i=1 r A i Ei

s j =1

Para B a demonstra ca o e an aloga.

B Ej = A = A .

Nas rela co es (8.67) e (8.68) e poss vel fazer simplica co es em fun ca o do fato de que nem todos os projetores Qi, j s ao n ao-nulos. Seja Q1 . . . , Qt a lista dos projetores Qi, j n ao-nulos, ou seja, {Q1 . . . , Qt } = {Qi, j | Qi, j = 0, i = 1, . . . , r e j = 1, . . . , s} . evidente por (8.65) que os Qk s s E ao projetores e que Qk Ql = k, l Qk . Por (8.66), tem-se
t

=
k=1

Qk

(8.69)

e por (8.67) e (8.68)


t

A =
k=1 t

A k Qk B k Qk
k=1

(8.70)

B =

(8.71)

B A B onde as constantes A ao relacionadas de modo obvio com i, j e i, j , respectivamente. k e k est

Em (8.70) e (8.71) vemos que A e B , por serem diagonaliz aveis e por comutarem entre si, t em decomposi co es espectrais com os mesmos projetores espectrais. Note-se tamb em que, pela observa ca o feita no t opico Projetores, ` a p agina 322 (vide equa ca o (8.43)), tem-se 1 t n. Vamos agora completar a demonstra ca o que A e B podem ser diagonalizados por uma mesma matriz invers vel P . Seja Ek o subespa co dos autovetores de Qk com autovalor 1. Subespa cos Ek s diferentes t em em comum apenas o vetor nulo. De fato, se k = l e w e um vetor tal que Qk w = w e Ql w = w ent ao, como Qk Ql = 0 segue que 0 = (Qk Ql )w = Qk (Ql w) = Qk w = w .
dk Seja dk a dimens ao do subespa co Ek e seja u1 k , . . . , uk um conjunto de dk vetores linearmente independentes em Ek . Notemos que dk coincide com a multiplicidade alg ebrica do autovalor 1 de Qk , pois, conforme diz a Proposi ca o t 8.21, o projetor Qk e diagonaliz avel e, portanto, e uma matriz simples (Proposi ca o 8.16). Como = k=1 Qk , tem-se, t tomando-se o tra co, que n = k=1 dk . Pelas deni co es, temos que a Ql u a k = k, l uk , a a a a pois Qk ua k = uk e, portanto, Ql uk = Ql (Qk uk ) = (Ql Qk )uk = 0 para k = l .

(8.72)

Armamos que o conjunto de vetores


d1 d2 dt 1 1 u1 1 , . . . , u1 , u2 , . . . , u2 , . . . ut , . . . , ut

(8.73)

e formado por n vetores linearmente independentes. De fato, suponha que existam constantes ck, j tais que
t dk

ck, j uj k = 0 .
k=i j =1

JCABarata. Curso de F sica-Matem atica

Vers ao de 19 de junho de 2011.

Cap tulo 8

334/1943

dl l Aplicando-se ` a direita Ql ter amos j =1 cl, j uj o e poss vel se cl, j = 0 para todo j pois u1 l , . . . , ul , foram l = 0, o que s escolhidos linearmente independentes. Como l e arbitr ario, conclu mos que cl, j = 0 para todo l e todo j , o que mostra que o conjunto de vetores em (8.73) e linearmente independente.

Seja ent ao a matriz P Mat (C, n) denida por P =


d1 d2 dt 1 1 . u1 1 , . . . , u1 , u2 , . . . , u2 , . . . ut , . . . , ut

P e invers vel pois o conjunto (8.73) e linearmente independente (e, portanto, det(P ) = 0). Tem-se, AP = Escrevendo A =
t l=1 d1 d2 dt 1 1 Au1 . 1 , . . . , Au1 , Au2 , . . . , Au2 , . . . , Aut , . . . , Aut

A l Ql (8.70) e usando (8.72), temos


t

Aua k =
l=1

a A a A l Ql u k = k u k .

Assim, AP = onde
1 A d1 A 1 A d1 A 1 A dt A 1 u 1 , . . . , 1 u 1 , 2 u 1 , . . . , 2 u 1 , . . . , t u t , . . . , t u t

= P DA ,

Portanto, P

AP = DA . Analogamente, BP =
t

A A A A A DA = diag A 1 , . . . , 1 , 2 , . . . , 2 , . . . , t , . . . , t .
d1 vezes d2 vezes dt vezes d1 d2 dt 1 1 Bu1 . 1 , . . . , Bu1 , Bu2 , . . . , Bu2 , . . . But , . . . , But

Escrevendo B =
l=1

B l Ql (8.71) temos, BP =
1 B d1 B d2 B 1 B 1 B dt B 1 u 1 , . . . , 1 u 1 , 2 u 2 , . . . , 2 u 2 , . . . , t u t , . . . , t u t

= P DB ,

onde

Portanto, P BP = DB . Isso provou que A e B s ao diagonaliz aveis pela mesma matriz invers vel P . A demonstra ca o do Teorema 8.8 est a completa.

B B B B B DB = diag B 1 , . . . , 1 , 2 , . . . , 2 , . . . , t , . . . , t .
d1 vezes d2 vezes dt vezes

8.5

Matrizes Auto-Adjuntas, Normais e Unit arias

Seja V um espa co vetorial dotado de um produto escalar , e seja A : V V um operador linear. Um operador linear A que para todos u, v V satisfa ca u, Av = A u, v e dito ser o operador adjunto de A. Em espa cos vetoriais gerais n ao e obvio (e nem sempre verdadeiro!) que sempre exista o adjunto de um operador linear A dado. H a muitos casos, por em, nos quais isso pode ser garantido9. Aqui trataremos do caso dos espa cos V = Cn com o produto escalar usual.
9 Tal e o caso dos chamados operadores lineares limitados agindo em espa cos de Hilbert, para os quais sempre e poss vel garantir a exist encia do adjunto.

A adjunta de uma matriz

JCABarata. Curso de F sica-Matem atica

Vers ao de 19 de junho de 2011.

Cap tulo 8

335/1943

Sejam u = (u1 , . . . , un ) e v = (v1 , . . . , vn ) dois vetores de Cn para os quais dene-se o produto escalar usual
n

u, v =
k=1

uk vk .

Um operador linear A e representado (na base can onica) por uma matriz cujos elementos de matriz s ao Aij , com i, j {1, . . . , n}. um exerc E cio simples (fa ca!) vericar que o operador adjunto A de A e representado (na base can onica) por uma e obtida (na matriz cujos elementos de matriz s ao (A )ij = Aji , com i, j {1, . . . , n}. Ou seja, a matriz adjunta de A base can onica!) transpondo-se A e tomando-se o complexo conjugado de seus elementos. Os seguintes fatos s ao importantes: Proposi c ao 8.23 Se A e B s ao dois operadores lineares agindo em Cn ent ao (A + B ) = A + B para todos , C. Fora isso, Por m, vale para todo A que (A ) = A. (AB ) = B A .

Deixamos a demonstra ca o como exerc cio para o leitor. A opera ca o Mat (C, n) A A Mat (C, n) e denominada opera c ao de adjun c ao de matrizes. Como vimos na Proposi ca o 8.23, a opera ca o de adjun ca o e anti-linear e e um anti-homomorsmo alg ebrico. Os espectro e a opera c ao de adjun c ao

Seja A Mat (C, n). Como j a vimos, o espectro de A, (A), e o conjunto de ra zes de seu polin omio caracter stico, denido por pA (z ) = det(z A), z C. Como para toda B Mat (C, n) vale det(B ) = det(B ) (por qu e?), segue que pA (z ) = det(z A) = det(z A ) = pA (z ), ou seja, pA (z ) = pA (z ). Com isso, provamos a seguinte arma ca o:

e um autovalor de A Proposi c ao 8.24 Seja A Mat (C, n). Ent ao, (A) se e somente se (A ), ou seja, se e somente se e um um autovalor de A . Em s mbolos, as arma co es acima s ao expressas pela igualdade (A) = (A ). Matrizes Hermitianas, normais e unit arias

Vamos agora a algumas deni co es muito importantes.

Deni c ao. Um operador linear em Cn e dito ser sim etrico, Hermitiano ou auto-adjunto se A = A , ou seja, se para todos u, v V satiszer u, Av = Au, v . cos vetoriais de dimens ao nita as no co es de operador sim etrico, Hermitiano ou auto-adjunto s ao Advert encia. Em espa sin onimas. Em espa cos vetoriais de dimens ao innita, por em, h a uma distin ca o entre essas no co es relativa a problemas com o dom nio de deni ca o de operadores. Deni c ao. Um operador linear em Cn e dito ser normal se AA = A A. Ou seja, A e normal se comuta com seu adjunto. claro que todo operador unit e dito ser unit ario se A A = AA = . E ario e Deni c ao. Um operador linear em Cn n 1 normal e que um operador e unit ario em C se e somente se A = A . Note que se A e unit ario ent ao, para todos u, v V , tem-se Au, Av = u, v .

JCABarata. Curso de F sica-Matem atica

Vers ao de 19 de junho de 2011.

Cap tulo 8

336/1943

e um operador linear em Cn dene-se a parte real de A por Deni c ao. Se A Re (A) = e a parte imagin aria de A por Im (A) = 1 (A + A ) 2 1 (A A ). 2i

claro que essas deni E co es foram inspiradas nas rela co es an alogas para n umeros complexos. Note tamb em que A = Re (A) + iIm (A) . e? E. 8.24 Exerc cio. Por qu importante notar que para qualquer operador linear A em Cn sua parte real e imagin E aria s ao ambas operadores Hermitianos: (Re (A)) = Re (A) e (Im (A)) = Im (A). E. 8.25 Exerc cio. Mostre isso. Para operadores normais tem-se a seguinte proposi ca o, que ser au til adiante e serve como caracteriza ca o alternativa do conceito de operador normal. Proposi c ao 8.25 Um operador linear agindo em Cn e normal se e somente se sua parte real comuta com sua parte imagin aria. Deixamos a demonstra ca o (elementar) como exerc cio para o leitor. A import ancia das deni co es acima reside no seguinte fato, que demonstraremos adiante: matrizes Hermitianas e matrizes normais s ao diagonaliz aveis. Antes de tratarmos disso, vamos discutir algumas propriedades do espectro de matrizes Hermitianas e de matrizes unit arias. Os autovalores de matrizes Hermitianas e de matrizes unit arias

Os seguintes teoremas t em import ancia fundamental para o estudo de propriedades de matrizes Hermitianas e de matrizes unit arias. Teorema 8.9 Os autovalores de uma matriz Hermitiana s ao sempre n umeros reais. Prova. Seja A Hermitiana, um autovalor de A e v = 0 um autovetor de A com autovalor . Como A e Hermitiana tem-se v, Av = Av, v . Como v e um autovetor, o lado esquerdo vale v, v e o lado direito vale v, v . Logo, ( ) v, v = 0. Como v = 0 isso implica = , ou seja, e real. 2 1 tem autovalores reais (2 e 3) mas n ao e Hermitiana. Note-se que a rec proca desse teorema e falsa. A matriz 0 3 Para matrizes unit arias temos Teorema 8.10 Os autovalores de uma matriz unit aria s ao sempre n umeros complexos de m odulo 1. Prova. Seja A unit aria, um autovalor de A e v = 0 um autovetor de A com autovalor . Como A e unit aria tem-se Av, Av = v, v . Como v e um autovetor, o lado esquerdo vale v, v . Assim, (||2 1) v, v = 0. Como v = 0 isso implica || = 1.

JCABarata. Curso de F sica-Matem atica

Vers ao de 19 de junho de 2011.

Cap tulo 8

337/1943

Operadores sim etricos e unit arios. Ortogonalidade de autovetores Teorema 8.11 Os autovetores associados a autovalores distintos de uma matriz sim etrica s ao ortogonais entre si.

Prova. Seja A sim etrica e 1 , 2 dois de seus autovalores, que suporemos distintos. Seja v1 autovetor de A com autovalor 1 e v2 autovetor de A com autovalor 2 . Temos, por A ser sim etrico, v1 , Av2 = Av1 , v2 . O lado esquerdo vale 2 v1 , v2 e o lado direito 1 v1 , v2 (lembre-se que 1 e real). Assim (2 1 ) v1 , v2 = 0. Como 2 = 1 , segue que v1 , v2 = 0, que e o que se queria provar. Teorema 8.12 Os autovetores associados a autovalores distintos de uma matriz unit aria s ao ortogonais entre si.

Prova. Seja U unit aria e sejam 1 , 2 dois de seus autovalores, sendo que suporemos 1 = 2 . Seja v1 autovetor de U com autovalor 1 e v2 autovetor de U com autovalor 2 . Temos, por U ser unit ario, U v1 , U v2 = v1 , U U v2 = v1 , v2 . 2 e um n umero complexo de m odulo 1 e, portanto O lado esquerdo vale 2 1 v1 , v2 = 1 v1 , v2 (lembre-se que 1 1 1 = 1 ). Assim 2 1 v1 , v2 = 0 . 1 Como 2 = 1 , segue que v1 , v2 = 0, que e o que se queria provar.

Projetores ortogonais

Um operador linear E agindo em Cn e dito ser um projetor ortogonal se E 2 = E e se E = E .

Projetores ortogonais s ao importantes na decomposi ca o espectral de matrizes auto-adjuntas, como veremos.

1 0 . (E = E ). O mesmo vale para E = 2 0

1 0 2 ao e ortogonal Note-se que nem todo projetor e ortogonal. Por exemplo E = e um projetor (E = E ) mas n 1 0

Um exemplo importante de projetor ortogonal e representado por projetores sobre subespa cos uni-dimensionais gerados por vetores. Seja v um vetor cuja norma assumiremos ser 1, ou seja, v = v, v = 1. Denimos o projetor Pv sobre o subespa co gerado por v por Pv u := v, u v , (8.74) para todo vetor u. Provemos que Pv e um projetor ortogonal. Por um lado, tem-se
2 Pv u = v, u Pv v = v, u

v, v v = v, u v = Pv u ,

2 o que mostra que Pv = Pv . Por outro lado, para quaisquer vetores a e b, usando as propriedades de linearidade, anti-linearidade e conjuga ca o complexa do produto escalar, tem-se

a, Pv b = a, v, b v = v, b

a, v =

a, v v, b =

v, a v, b = Pv a, b ,

provando que Pv = Pv . Isso mostra que Pv e um projetor ortogonal.

Um fato crucial sobre projetores como Pv e o seguinte. Se u e v s ao dois vetores ortogonais, ou seja, se u, v = 0 ent ao Pu Pv = Pv Pu = 0. Para provar isso notemos que para qualquer vetor a vale Pu (Pv a) = Pu ( v, a v ) = v, a Pu v = v, a O mesmo se passa para Pv (Pu a). u, v u = 0 .

JCABarata. Curso de F sica-Matem atica

Vers ao de 19 de junho de 2011.

Cap tulo 8

338/1943

Vamos aqui demonstrar a seguinte arma ca o importante: toda matriz auto-adjunta e diagonaliz avel. Uma outra demonstra ca o (eventualmente mais simples) dessa arma ca o pode ser encontrada na Se ca o 8.8.3, p agina 365. Vide Teorema 8.27, p agina 367. Teorema 8.13 (Teorema Espectral para matrizes auto-adjuntas) Se A Mat (C, n) e auto-adjunta, ent ao A possui n autovetores mutuamente ortonormais v1 , . . . , vn , com autovalores reais 1 , . . . , n , respectivamente, e pode ser representada na forma espectral (8.75) A = 1 Pv1 + + n Pvn .
n k=1

Matrizes auto-adjuntas e diagonalizabilidade

Os projetores Pvk satisfazem Pv = Pvk para todo k e valem tamb em Pvj Pvk = jkPvk , sendo que k

Pvk = .

Portanto, se A e auto-adjunta, ent ao A e diagonaliz avel, sendo que e poss vel encontrar uma matriz unit aria P que diagonaliza A, ou seja, tal que P 1 AP e diagonal e P 1 = P . Note-se que se 1 , . . . , r com 1 r n s ao os autovalores distintos de A, ent ao (8.75) pode ser reescrita como A = 1 P1 + + r Pr , onde cada Pk e o projetor ortogonal dado pela soma dos Pvj s de mesmo autovalor k . A Proposi c ao 8.19, p agina 325, garante a unicidade dessa representa c ao para A. Prova do Teorema 8.13. A demonstra ca o que A e diagonaliz avel ser a feita construindo-se a representa ca o espectral (8.75) para A. Seja 1 um autovalor de A e v1 um autovetor de A com autovalor 1 normalizado de tal forma que v1 = 1. Vamos denir um operador A1 por A1 = A 1 Pv1 . ao auto-adjuntos e 1 e real, segue que A1 e igualmente auto-adjunto. Como A e Pv1 s Armamos que A1 v1 = 0 e que [v1 ] e um subespa co invariante por A1 . De fato, A1 v1 = Av1 1 Pv1 v1 = 1 v1 1 v1 = 0 . Fora isso, se w [v1 ] tem-se A1 w, v1 = w, A1 v1 = 0 ,

mostrando que A1 w e tamb em elemento de [v1 ] . O operador A1 restrito a [v1 ] e tamb em auto-adjunto (por que?). Seja 2 um de seus autovalores com autovetor v2 [v1 ] , que escolhemos com norma 1. Seja A2 = A1 2 Pv2 = A 1 Pv1 2 Pv2 . Como 2 tamb em e real A2 e igualmente auto-adjunto. Fora isso armamos que A2 anula os vetores do subespa co [v1 , v2 ] e mantem [v1 , v2 ] invariante. De fato, A2 v1 = Av1 1 Pv1 v1 2 Pv2 v1 = 1 v1 1 v1 2 v2 , v1 v2 = 0 , pois v2 , v1 = 0. Analogamente, A2 v2 = A1 v2 2 Pv2 v2 = 2 v2 2 v2 = 0 . Por m, para quaisquer , C e w [v1 , v2 ] tem-se A2 w, (v1 + v2 ) = w, A2 (v1 + v2 ) = 0 , que e o que quer amos provar. Prosseguindo indutivamente, construiremos um conjunto de vetores v1 , . . . , vn , todos com norma 1 e com va [v1 , . . . , va1 ] e um conjunto de n umeros reais 1 , . . . , n tais que An = A 1 Pv1 n Pvn anula-se no subespa co [v1 , . . . , vn ]. Ora, como estamos em um espa co de dimens ao n e os vetores vk s ao mutuamente ortogonais, segue que [v1 , . . . , vn ] deve ser o espa co todo, ou seja, An = 0. Provamos ent ao que A = 1 Pv1 + + n Pvn . (8.76)

JCABarata. Curso de F sica-Matem atica

Vers ao de 19 de junho de 2011.

Cap tulo 8

339/1943

Vamos provar agora que essa e a representa ca o espectral de A. Como os vk s s ao mutuamente ortogonais, e evidente que Pvk Pvl = k, l Pvk . Resta-nos provar que Pv1 + + Pvn = . Como v1 , . . . , vn formam uma base, todo vetor x pode ser escrito como uma combina ca o linear x = 1 v1 + + n vn . (8.77)

Tomando-se o produto escalar com va , e usando o fato que os vk s s ao mutuamente ortogonais, tem-se a = va , x . E. 8.26 Exerc cio. Verique. Assim, (8.77) pode ser escrita como x = v1 , x v1 + + vn , x vn = Pv1 x + + Pvn x = (Pv1 + + Pvn ) x . Como isso vale para todo vetor x, segue que Pv1 + + Pvn = . Assim, A possui uma representa ca o espectral como (8.44). Pelo Teorema Espectral 8.5, A e diagonaliz avel. Por (8.76), vemos que Ava = a va (verique!). Logo os a s s ao autovalores de A e os va s seus autovetores. Assim, se A e auto-adjunto, podemos encontrar n autovetores de A mutuamente ortogonais, mesmo que sejam autovetores com o mesmo autovalor. Isso generaliza o Teorema 8.11. f acil vericar, por em, Pelo que j a vimos A e diagonalizada por P 1 AP , onde podemos escolher P = v 1 , . . . , v n . E que P e unit aria. De fato, e um exerc cio simples (fa ca!) mostrar que v1 , v1 . . P P = . vn , v1 .. .

e unit aria. Como va , vb = a, b , a matriz do lado direito e igual a , mostrando que P P = P P = e que, portanto, P

v1 , vn . . . . vn , vn

Para concluir essa discuss ao, temos: Proposi c ao 8.26 Uma matriz A Mat (C, n) e auto-adjunta, se e somente se for diagonaliz avel por uma transforma c ao de similaridade unit aria e se seus autovalores forem reais.

Prova. Se A Mat (C, n) e diagonaliz avel por uma transforma ca o de similaridade unit aria e seus autovalores s ao reais, ou seja, existe P unit aria e D diagonal real com P AP = D, ent ao A = P DP e A = P D P . Como D e diagonal e real, vale D = D e, portanto, A = P DP = A, provando que A e auto-adjunta. A rec proca j a foi provada acima.

Matrizes normais e diagonalizabilidade

O teorema que arma que toda matriz sim etrica e diagonaliz avel tem a seguinte conseq u encia:

Teorema 8.14 Se A Mat (C, n) e normal ent ao A e diagonaliz avel. Prova. J a vimos que toda matriz A pode ser escrita na forma A = Re (A) + iIm (A) onde Re (A) e Im (A) s ao autoadjuntas. Vimos tamb em que se A e normal Re (A) e Im (A) comutam entre si (Proposi ca o 8.25). Pelo Teorema 8.8, Re (A) e Im (A) podem ser simultaneamente diagonalizados. Observa c ao. Como no caso auto-adjunto, o operador que faz a diagonaliza ca o pode ser escolhido unit ario. De fato, vale uma armativa ainda mais forte.

JCABarata. Curso de F sica-Matem atica

Vers ao de 19 de junho de 2011.

Cap tulo 8

340/1943

Teorema 8.15 Uma matriz A Mat (C, n) e normal se e somente se for diagonaliz avel por um operador unit ario. Prova. Resta provar apenas que se A e diagonaliz avel por um operador unit ario P ent ao A e normal. Seja D = P AP . Tem-se D = P A P (por que?). Assim, A A AA = P D P P DP P DP P D P = P (D D DD )P = 0 , j a que D e D comutam por serem diagonais (duas matrizes diagonais quaisquer sempre comutam. Por qu e?). Isso completa a prova que A e normal. Uma outra demonstra ca o (eventualmente mais simples) dessa arma ca o pode ser encontrada na Se ca o 8.8.3, p agina 365. Vide Teorema 8.28, p agina 367.

8.5.1

Matrizes Positivas

Uma matriz A Mat (C, n) e dita ser uma matriz positiva se w, Aw 0 para todo vetor w Cn . A seguinte 10 proposi ca o e relevante : Proposi c ao 8.27 Se A Mat (C, n) e positiva, ent ao A e Hermitiana e tem autovalores n ao-negativos. Reciprocamente, se A e Hermitiana e tem autovalores n ao-negativos, ent ao A e positiva. Prova. A express ao (u, v ) := u, Av , u, v Cn , dene uma forma sesquilinear que, por hip otese, e positiva, ou seja, satisfaz (u, u) 0 para todo u Cn . Pelo Teorema 3.1, p agina 180, e Hermitiana, ou seja, (u, v ) = (v, u) , para todos os vetores u e v . Mas isso signica que u, Av = v, Au , ou seja, u, Av = Au, v para todos os vetores u e v e assim provou-se que A = A . Uma outra forma de demonstrar isso usa a identidade de polariza ca o. Se A e positiva ent ao, para quaisquer vetores u, v Cn vale (u + in v ), A(u + in v ) 0 para todo n Z e, portanto, (u + in v ), A(u + in v ) e um n umero real. Usando a identidade de polariza ca o, eqs. (3.32)(3.33), p agina 190, vale, para quaisquer vetores u, v Cn , Av, u = u, Av
(3.32)

1 in (u + in v ), A(u + in v ) 4 n=0
3

1 in (u + in v ), A(u + in v ) 4 n=0

1 in in in (u + in v ), A(u + in v ) 4 n=0 1 in in (u + in v ), Ain (u + in v ) 4 n=0 1 in (v + in u), A((1)n v + in u) 4 n=0 1 (1)n in (v + in u), A(v + in u) 4 n=0 1 in (v + in u), A(v + in u) 4 n=0
3 (3.33) 3 3 3

sesquilin.

v, Au .

10 V arios dos resultados que seguem podem ser generalizados para operadores lineares positivos agindo em espa cos de Hilbert. Vide Teorema 35.30, p agina 1819.

JCABarata. Curso de F sica-Matem atica

Vers ao de 19 de junho de 2011.

Cap tulo 8

341/1943

Assim, Av, u = v, Au para todos u, v Cn , o que signica que A e Hermitiana. Portanto, por (8.75), podemos ao autovetores mutuamente ortonormais de A com autovalores escrever A = 1 Pv1 + + n Pvn , onde v1 , . . . , vn s 1 , . . . , n , respectivamente. Disso segue que vj , Avj = j para todo j = 1, . . . , n. Como o lado esquerdo e 0, por hip otese, segue que j 0 para todo j = 1, . . . , n. Se, reciprocamente, A for auto-adjunta com autovalores n ao-negativos, segue de (8.75) e da deni ca o de Pvj em (8.74)
n j =1

que w, Aw =

j | w, vj |2 0, para todo w Cn , provando que A e positiva.

O seguinte corol ario e imediato. Corol ario 8.4 Uma matriz A Mat (C, n) positiva se somente se existe uma matriz positiva B (un voca!) tal que A = B 2 . As matrizes A e B comutam: AB = BA.

Demonstra c ao. Se A = B 2 com B positiva, ent ao, como B e auto-adjunta (pela Proposi ca o 8.27), segue que para todo n w C vale w, Aw = w, B 2 w = Bw, Bw = Bw 2 0, provando que A e positiva. Provemos agora a rec proca.

Se A e positiva ent ao, como comentamos na demonstra ca o da Proposi ca o 8.27, A e auto-adjunta com representa ca o ao autovetores mutuamente ortonormais de A com autovalores espectral A = 1 Pv1 + + n Pvn , onde v1 , . . . , vn s 1 , . . . , n , respectivamente, todos n ao-negativos. Dena-se a matriz B := 1 Pv1 + + n Pvn . (8.78)

e f acil ver que B 2 = 1 Pv1 + + n Pvn = A. A unicidade Como, pela ortonormalidade dos vj s, vale Pvj Pvk = j, k Pvj , de B segue da unicidade da decomposi ca o espectral, Proposi ca o 8.19, p agina 325. A igualdade (B 2 )B = B (B )2 signica AB = BA, provando que A e B comutam. Deni c ao. Se A e uma matriz positiva, a ( unica!) matriz positiva B satisfazendo B 2 = A e freq uentemente denotada por A e denominada raiz quadrada da matriz A. Como vimos, A A = AA. Lema 8.2 Se A Mat (C, n) e uma matriz positiva e C Mat (C, n) satisfaz CA = AC ent ao C A = AC . Prova. Se C comuta com A, ent ao C comuta com qualquer polin omio em A. Vimos na Proposi ca o 8.18, p agina 325, que os projetores espectrais de A podem ser escritos como polin o mios em A . Assim, C comuta com os projetores espectrais de A e, portanto, com A, devido a (8.78). Uma conseq u encia interessante das considera co es acima e a seguinte proposi ca o: Proposi c ao 8.28 Toda matriz Hermitiana pode ser escrita como combina c ao linear de at e duas matrizes unit arias. Toda matriz pode ser escrita como combina c ao linear de at e quatro matrizes unit arias.

Demonstra c ao. Seja A Mat (C, n). Se A e Hermitiana (vamos supor que A = , pois de outra forma n ao h a o que se provar), ent ao, para todo w Cn , o produto escalar w A2 w e um n umero real e, pela desigualdade de Cauchy-Schwarz, 2 2 2 2 2 e positiva, pois | w A2 w | A2 w 2 w 2 w 2 Cn . Assim, A Cn w, A w A Cn Logo, a matriz A / A 2 2 2 2 2 2 2 2 e uentemente, A / A2 existe e w, ( A / A )w = w Cn w, A w / A w Cn w Cn = 0. Conseq positiva e Hermitiana. Trivialmente, podemos escrever A = A2 2 A A2 +i

A2 A2

A2 2

A A2

A2 A2

(8.79)

Agora, as matrizes A 2 i
A

A2 A2

s ao unit arias. Para ver isso, notemos que A2 2 A

A +i A2

A i A2

A2 A2

JCABarata. Curso de F sica-Matem atica

Vers ao de 19 de junho de 2011.

Cap tulo 8

342/1943

e que A A2 +i

A2 A2

A A2

A2 A2

= .

Para provar a u ltima igualdade basta expandir o produto e notar que, pelo Lema 8.2, A e A e
A A2
2

A2 A2

comutam, j a que

comutam.

Assim, vemos de (8.79) que uma matriz Hermitiana A e combina ca o linear de at e duas unit arias, provando a primeira parte da Proposi ca o 8.28. Para provar a segunda parte, basta notar que se M Mat (C, n) e uma matriz qualquer, podemos escrever M + M M M M = +i . 2 2i Ambas as matrizes entre par enteses s ao Hermitianas e, portanto, podem cada uma ser escritas como combina ca o linear de at e duas unit arias, totalizando at e quatro unit arias para M . A Proposi ca o 8.28 e v alida n ao apenas para algebras de matrizes. Vide Proposi ca o 35.43, p agina 1772.

8.5.2

O Teorema de In ercia de Sylvester. Superf cies Quadr aticas

Seja M Mat (C, n). Se P Mat (C, n) e invers vel, a transforma ca o M P M P e dita ser uma transforma c ao de congru encia. Uma transforma ca o de congru encia representa a transforma ca o de uma matriz por uma mudan ca de base (justique essa arma ca o!). Se M for auto-adjunta, P M P e tamb em auto-adjunta e, portanto, ambas t em auto-valores reais. Em geral, o conjunto dos auto-valores de M e distinto do conjunto dos auto-valores de P M P (exceto, por exemplo, se P for unit aria). Por em, um teorema devido a Sylvester, frequ entemente denominado Lei de In ercia de Sylvester, arma que uma propriedade do conjunto dos auto-valores e preservada em uma transforma ca o de congru encia, a saber, o n umero de autovalores, positivos, de autovalores negativos e de autovalores nulos (contando-se as multiplicidades). Enunciaremos e demonstraremos esse teorema logo adiante. Dada uma matriz auto-adjunta M Mat (C, n), a tripla de n umeros (m, m , m0 ), onde m e o n umero de autovalores positivos de M , m e o n umero de autovalores negativos de M , m0 e o n umero de autovalores nulos de M , (em todos os casos contando-se as multiplicidades) e denominada (por raz oes hist oricas obscuras) a in ercia da matriz M . Naturalmente, vale m + m + m0 = n. A Lei de In ercia de Sylvester arma, portanto, que a in ercia de uma matriz e preservada por transforma co es de congru encia. Dizemos que duas matrizes A e B Mat (C, n) s ao congruentes se existir P Mat (C, n) invers vel tal que muito f A = P BP . E acil provar que a rela ca o de congru encia e uma rela ca o de equival encia. c ao! E. 8.27 Exerc cio. Demonstre essa arma Dessa forma, a Lei de In ercia de Sylvester arma que a in ercia de matrizes e constante nas classes de equival encia (pela rela ca o de congru encia). Assim, e leg timo perguntar se as classes de equival encia s ao univocamente determinadas pela in ercia de seus elementos. A resposta e negativa (exceto no caso trivial n = 1), como mostra a argumenta ca o do par agrafo que segue. Se A Mat (C, n), com n > 1, e uma matriz positiva, A e da forma P P (Corol ario 8.4, p agina 341). Assim, 2 det A = | det P | e conclu mos que A e invers vel se e somente se P o for. Conclu -se disso que a classe de equival encia (por rela co es de congru encia) que cont em a matriz identidade cont em todas as matrizes positivas e invers veis. Pela Proposi ca o 8.27, p agina 340, esse conjunto coincide com o conjunto de todas as matrizes auto-adjuntas com autovalores positivos, ou seja, que possuem in ercia (n, 0, 0). Entretanto, existem tamb em matrizes n ao-auto-adjuntas com in ercia (n, 0, 0) (por exemplo, matrizes triangulares superiores11 com elementos positivos na diagonal e alguns elementos n aonulos acima da diagonal). Como tais matrizes n ao podem ser equivalentes ` a identidade (toda matriz da forma P P e
11 Para

Transforma co es de congru encia em Mat (C, n)

a deni ca o, vide p agina 346

JCABarata. Curso de F sica-Matem atica

Vers ao de 19 de junho de 2011.

Cap tulo 8

343/1943

auto-adjunta), conclu mos que as classes de equival encia n ao s ao determinadas univocamente pela in ercia das matrizes que as comp oe. A Lei de In ercia de Sylvester

A Lei de In ercia de Sylvester e importante para a classica ca o de formas quadr aticas e sua relev ancia estende-se at e a classica ` ca o de equa co es diferenciais parciais de segunda ordem. Tratemos de seu enunciado e demonstra ca o. Teorema 8.16 (Lei de In ercia de Sylvester) Sejam A e B Mat (C, n) duas matrizes auto-adjuntas. Denotemos por A+ , A , A0 os subespa cos gerados, respectivamente, pelos auto-vetores com autovalores positivos, negativos e nulos de A (e analogamente para B ). Suponhamos que exista P Mat (C, n), invers vel, tal que A = P BP . Ent ao, dim A+ = dim B+ , dim A = dim B e dim A0 = dim B0 , onde dim C denota a dimens ao de um subespa co C Cn . Assim, conclu mos tamb em que A e B t em o mesmo n umero de autovalores positivos, o mesmo n umero de autovalores negativos e o mesmo n umero de autovalores nulos (em todos os casos, contando-se as multiplicidades). Prova. Sejam 1 , . . . , a os auto-valores positivos (n ao necessariamente distintos) e a+1 , . . . , a+a os auto-valores negativos (n ao necessariamente distintos) de A. Analogamente, sejam 1 , . . . , b os auto-valores positivos (n ao necesao necessariamente distintos) de B . Naturalmente, sariamente distintos) e b+1 , . . . , b+b os auto-valores negativos (n valem 0 a + a n e 0 b + b n. Se A e B forem nulos n ao h a o que demonstrar, de modo que podemos supor que ambos t em pelo menos um auto-valor n ao-nulo. Nesse caso, podemos sempre, sem perder em generalidade, supor que A tem pelo menos um autovalor positivo, pois se tal n ao for verdade para A ser a verdadeiro para A. O Teorema Espectral, Teorema 8.5, p agina 322, permite-nos escrever
a a+a

A =
k=1

k Ak

l=a+1 b+b

|l |Al

e
b

B =
k=1

k Bk

l=b+1

| l |B l ,

(8.80)

onde Aj e Bj s ao os projetores espectrais de A e B , respectivamente. Dena-se


a a+a

A+ :=
k=1

Ak ,

A :=
l=a+1

Al

A0 := A+ A

e, analogamente,
b b+b

B+ :=
k=1

Bk ,

B :=
l=b+1

Bl

B0 := B+ B .

A+ , A e A0 s ao, respectivamente, o projetor sobre o subespa co de autovetores com auto-valores positivos, negativos e nulos de A. Analogamente para B . Esses subespa cos s ao A = A Cn , A0 = A0 Cn , B = B Cn , B0 = B0 Cn .

Seja x um vetor n ao-nulo de A+ . Tem-se que Al x = 0 para todo l > a e Ak x = 0 para pelo menos um k = 1, . . . , a. Logo, como k > 0 para todo k = 1, . . . , a, segue que
a a a

x, Ax

=
k=1

k x, Ak x

=
k=1

k Ak x, Ak x

=
k=1

k Ak x

>0.

(8.81)

Por em, para um tal x vale tamb em x, Ax = x, P BP x

= P x, BP x

(8.80) C

b+b

k Bk P x
k=1

l=b+1

| k | B k P x

JCABarata. Curso de F sica-Matem atica

Vers ao de 19 de junho de 2011.

Cap tulo 8

344/1943

Vamos agora supor que B+ < dim A+ (ou seja, que b < a). Armamos que podemos encontrar ao menos um x+ A+ , n ao-nulo, tal que Bk P x+ = 0 para todo k = 1, . . . , b. Se assim n ao fosse, n ao existiria x A+ n ao-nulo satisfazendo B+ P x = 0, ou seja, valeria B+ P x = 0 para todo x A+ com x = 0. Logo, (P A+ ) (B+ ) = {0}, o que implica que P A+ B+ . Isso, por sua vez, signica que dimens ao do subespa c o P A+ e menor ou igual ` a dimens ao de B+ e, como P e invers vel, isso implica, dim A+ dim B+ , uma contradi ca o. Assim, para um tal x+ ter amos
b+b

x+ , Ax+

l=b+1

|k | Bk P x+

0,

contradizendo (8.81). Conclu mos disso que dim B+ dim A+ . Como B = (P )1 AP 1 , um racioc nio an alogo trocando 1 A e B e trocando P P implica que dim A+ dim B+ . Assim, dim B+ = dim A+ . Tamb em de forma totalmente an aloga prova-se que dim B = dim A (isso tamb em pode ser visto imediatamente trocando A A e B B ). Isso implica ainda que dim B0 = dim A0 , completando a demonstra ca o.

Para matrizes reais agindo no espa co Rn valem arma co es an alogas ` as obtidas acima. Seja M Mat (R, n). Se P Mat (R, n) e invers vel, a transforma ca o M P T M P e dita ser uma transforma c ao de congru encia real, ou simplesmente transforma c ao de congru encia. Uma transforma ca o de congru encia representa a transforma ca o de uma matriz por uma mudan ca de base (justique essa arma ca o!). Para transforma co es de congru encia reais vale tamb em a Lei de In ercia de Sylvester: se A Mat (R, n) e sim etrica (ou seja, se A = AT ) sua in ercia e preservada por transforma co es de congru encia A P T AP com P Mat (R, n) invers vel. Como essa arma ca o e um mero caso particular do anterior, omitimos a demonstra ca o e convidamos o estudante a complet a-la. Classica c ao de matrizes sim etricas em Rn

Transforma co es de congru encia em Mat (R, n)

Matrizes sim etricas em Rn podem ser classicadas de acordo com o tipo de in ercia que possuem, classica ca o essa invariante por transforma co es de congru encia. Uma matriz sim etrica A Mat (R, n), n > 1, e dita ser 1. Parab olica, se ao menos um dos seus autovalores for nulo, ou seja, se sua in ercia for da forma (a, a , a0 ) com a0 1; 2. El ptica, se todos os seus autovalores forem positivos ou se todos forem negativos, ou seja, se sua in ercia for da forma (a, a , 0) com a 1 e a = 0 ou com a 1 e a = 0; 3. Hiperb olica, se um de seus autovalores for positivo e os demais negativos, ou o oposto: se um de seus autovalores for negativo e os demais positivos, ou seja, se sua in ercia for da forma (1, a , 0) com a 1 (a, 1, 0) com a 1; 4. Ultra-Hiperb olica, se ao menos dois de seus autovalores forem positivos e ao menos dois forem negativos, nenhum sendo nulo, ou seja, se sua in ercia for da forma (a, a , 0) com a 2 e a 2. Esse caso s o se d a se n 4. Essa nomenclatura que classica as matrizes em parab olicas, el pticas, hiperb olicas e ultra-hiperb olicas tem uma motiva ca o geom etrica relacionada ` a classica ca o de superf cies quadr aticas em Rn , assunto que ilustraremos abaixo. Superf cies quadr aticas Rn

Sejam x1 , . . . , xn s ao n vari aveis reais. A forma mais geral de um polin omio real de segundo grau nessas vari aveis e
n n n

p(x) =
i=1 j =1

Aij xi xj +
k=1

ck xk + d , + c, x + d, onde,

onde Aij R, ck R e d R. A express ao acima para p pode ser escrita como p(x) = x, Ax
c1 x1

naturalmente, A e a matriz cujos elementos s ao Aij , c =

cn

. . .

ex=

xn

. . .

. A matriz A pode ser sempre, sem perda

JCABarata. Curso de F sica-Matem atica

Vers ao de 19 de junho de 2011.

Cap tulo 8

345/1943

de generalidade, escolhida como sim etrica. Para ver isso, notemos que, A pode sempre ser escrita como soma de uma 1 T matriz sim etrica e uma anti-sim etrica: A = 2 (A + AT ) + 1 2 (A A ). Contudo,
n n

x, (A AT )x

=
i=1 j =1

(Aij Aji ) xi xj = 0
R,

1 como facilmente se constata. Assim, a parte anti-sim etrica de A, ou seja, 2 (A AT ), n ao contribui em x, Ax 1 T a doravante considerada sim etrica. a parte sim etrica 2 (A + A ). Portanto, A ser

apenas

Estamos agora interessados em classicar as superf cies em Rn denidas por p(x) = , com constante. H a primeiramente dois casos a considerar: 1) A e invers vel e 2) A n ao e invers vel. 1. Se A e invers vel, podemos escrever p(x) = x, Ax
R

+ c, x

+d =

1 1 x + A1 c , A x + A1 c 2 2

1 c, A1 c 4

+d.

1 1 1 1 Verique! Assim, a equa ca o p(x) = ca x + 2 e a constante + A c , A x+ 2 A c R = , onde 1 1 d . A matriz sim e trica A pode ser diagonalizada por uma matriz ortogonal, ou seja, podemos c, A c 4 R T escrever A = O DO, com D = diag (1 , . . . , n ), com k sendo os autovalores de A e O sendo ortogonal. Podemos sempre escolher O de sorte que os primeiros m autovalores 1 , . . . , m s ao positivos e os demais m+1 , . . . , n s ao negativos (n ao h a autovalores nulos, pois A foi suposta invers vel). 1 1 1 c , A x+ 2 A c Com isso, x + 1 2A n 2 ent ao, k=1 k yk = ou seja, R m k=1 2 k yk

= y, Dy

R, n

1 1 onde y = O x + 2 A c . A equa ca o p(x) = ca,

l=m+1

2 |l | yl = .

(8.82)

Temos os seguintes sub-casos a tratar: (a) Se todos os autovalores de A s ao positivos e > 0, a equa ca o (8.82) descreve um elips oide em Rn (se < 0 n n ao h a solu co es e se = 0 a equa ca o descreve apenas o ponto y = 0 em R ). O mesmo vale, reciprocamente, se todos os autovalores de A forem negativos e < 0 (se > 0 n ao h a solu co es e se = 0 a equa ca o descreve apenas o ponto y = 0 em Rn ). (b) Se um dos autovalores de A e positivo e os demais n 1 s ao negativos, ou se ocorre o oposto, ou seja, se um dos autovalores de A e negativo e os demais n 1 s ao positivos, ent ao a equa ca o (8.82) descreve um hiperbol oide (n 1)-dimensional em Rn no caso = 0. Se > 0 o hiperbol oide tem duas folhas (i.e., possui duas componentes conexas) e no caso < 0 apenas uma. A Figura 8.1, p agina 347, exibe hiperbol oides com uma e duas folhas em R3 . Devido a sua estabilidade, hiperbol oides de uma folha s ao freq uentemente encontrados em estruturas arquitet onicas. A bem conhecida catedral de Bras lia, de Niemeyer12 , e um exemplo. A estabilidade estrutural desse formato decorre do fato que por qualquer ponto de um hiperbol oide de uma folha passam duas linhas retas inteiramente contidas dentro do mesmo (prove isso!). Se = 0 a equa ca o (8.82) descreve um cone (n 1)-dimensional em Rn . (c) Este caso ocorre apenas se n 4. Se ao menos dois autovalores de A e positivo e ao menos dois s ao positivos a equa ca o (8.82) descreve, no caso = 0, uma superf cie (n 1)-dimensional em Rn denominada ultra-hiperbol oide. Se = 0 a equa ca o (8.82) descreve uma (n 1)-dimensional em Rn denominada ultra-cone.

2. Se A n ao e invers vel temos que proceder de modo ligeiramente diferente. Como antes, a matriz sim etrica A pode ser diagonalizada por uma matriz ortogonal, ou seja, podemos escrever A = OT DO, com D = diag (1 , . . . , n ), com k sendo os autovalores de A e O sendo ortogonal. Como A n ao tem inversa, alguns de seus autovalores s ao nulos. Podemos sempre escolher O de sorte que os primeiros m autovalores 1 , . . . , m s ao positivos, os m ao negativos e os demais m+m +1 , . . . , n s ao nulos. Naturalmente, autovalores seguintes m+1 , . . . , m+m s 0 m + m < n. Podemos, ent ao, escrever p(x) = x, Ax R + c, x R + d = y, Dy R + Oc, y R + d onde y = Ox.
12 Oscar

Niemeyer Soares Filho (1907).

JCABarata. Curso de F sica-Matem atica

Vers ao de 19 de junho de 2011.

Cap tulo 8

346/1943

Assim, se c = 0 a equa ca o p(x) = ca yOc m +m 1 2 = + |l | yl c


l=m+1 m k=1 2 , k yk

(8.83)

onde = ( d)/ c e yOc e a proje ca o de y na dire ca o do vetor Oc. Se a dimens ao do subespa co dos autovalores nulos A0 for maior que 1 a equa ca o (8.83) descrever a cilindros de diversos tipos, dependendo do n umero de autovalores positivos e negativos e de Oc ter uma proje ca o ou n ao em A0 . N ao descreveremos os todos os detalhes aqui, mas um exemplo de interesse se d a em R3 , se A0 tiver dimens ao 2 e Oc for um vetor n ao-nulo de A0 . Nesse caso equa ca o (8.83) descreve um cilindro parab olico. Vide Figura 8.3, p agina 348. Para o caso em que A0 tem dimens ao 1 e Oc e um elemento n ao-nulo desse subespa co, a equa ca o (8.83) descreve diversos tipos de parabol oides (n 1)-dimensionais. Temos os seguintes casos: (a) a equa ca o (8.83) descreve um parabol oide el ptico (n 1)-dimensional caso todos os autovalores n ao-nulos de A forem positivos ou se todos os autovalores n ao-nulos de A forem negativos. Vide Figura 8.2, p agina 8.2. (b) A equa ca o (8.83) descreve um parabol oide hiperb olico (n 1)-dimensional caso um autovalor de A seja negativo e os demais autovalores n ao-nulos de A sejam positivos (ou o contr ario: caso um autovalor de A seja positivo e os demais autovalores n ao-nulos de A sejam negativos). Vide Figura 8.2, p agina 8.2. (c) A equa ca o (8.83) descreve um parabol oide ultra-hiperb olico (n 1)-dimensional caso pelo menos dois dos autovalores n ao-nulos de A sejam positivos e pelo menos dois dos autovalores n ao-nulos de A sejam negativos. Esse caso s o pode ocorrer se n 5. Para c = 0 diversas situa co es acima podem tamb em descrever cilindros, por exemplo, se Oc encontra-se no subespa co dos autovetores com autovalores n ao-nulos. Se c = 0 e dim A0 1, equa ca o p(x) = ca
m 2 k yk k=1 m +m

l=m+1

2 |l | yl = ,

(8.84)

com = d. A equa ca o (8.84) descreve diversos tipo de cilindros (n 1)-dimensionais. (a) Caso c = 0 a equa ca o (8.84) descreve um cilindro el ptico (n 1)-dimensional caso todos os autovalores n aonulos de A forem positivos ou se todos os autovalores n ao-nulos de A forem negativos. Vide Figura 8.3, p agina 348. (b) Caso c = 0 a equa ca o (8.84) descreve um cilindro hiperb olico (n 1)-dimensional caso um autovalor de A seja negativo e os demais autovalores n ao-nulos de A sejam positivos (ou o contr ario: caso um autovalor de A seja positivo e os demais autovalores n ao-nulos de A sejam negativos). Vide Figura 8.3, p agina 348. (c) Caso c = 0 a equa ca o (8.84) descreve um cilindro ultra-hiperb olico (n 1)-dimensional caso pelo menos dois dos autovalores n ao-nulos de A sejam positivos e pelo menos dois dos autovalores n ao-nulos de A sejam negativos. Esse caso s o pode ocorrer se n 5 (lembrar que pelo menos um dos autovalores de A e nulo).

8.6

Matrizes Triangulares

Uma matriz S Mat (C, n) e dita ser uma matriz triangular superior se forem nulos os elementos abaixo da diagonal principal, ou seja, se Sij = 0 sempre que i > j . Note que esses n ao precisam ser necessariamente os u nicos elementos nulos de S . Uma matriz I Mat (C, n) e dita ser uma matriz triangular inferior se forem nulos os elementos acima da diagonal principal, ou seja, se Iij = 0 sempre que i < j . Note que esses n ao precisam ser necessariamente os u nicos elementos nulos de I . Proposi c ao 8.29 Matrizes triangulares superiores possuem as seguintes propriedades:

JCABarata. Curso de F sica-Matem atica

Vers ao de 19 de junho de 2011.

Cap tulo 8

347/1943

Figura 8.1: Hiperbol oides com uma e duas folhas em R3 .

e uma matriz triangular superior. 1. A matriz identidade 2. O produto de duas matrizes triangulares superiores e novamente uma matriz triangular superior. 3. O determinante de uma matriz triangular superior e o produto dos elementos da sua diagonal. Assim, uma matriz triangular superior e invers vel se e somente se n ao tiver zeros na diagonal. 4. Se uma matriz triangular superior e invers vel, sua inversa e novamente uma matriz triangular superior. As arma c oes acima permanecem verdadeiras trocando matriz triangular superior por matriz triangular inferior.

Prova. Os tr es primeiros itens s ao elementares. Para provar o item 4, usa-se a regra de Laplace, express ao (8.19), p agina 300. Como e f acil de se ver, Cof(S )ji = 0 se i > j . Logo, S 1 e triangular superior, se existir. As propriedades acima atestam que o conjunto das matrizes n n triangulares superiores invers veis forma um grupo, denominado por alguns autores Grupo de Borel13 de ordem n e denotado por GBn (C). O seguinte resultado sobre matrizes triangulares superiores ser a usado diversas vezes adiante. Lema 8.3 Uma matriz triangular superior S Mat (C, n) e normal (ou seja, satisfaz SS = S S ) se e somente se for diagonal. Prova. Se S e diagonal, S e obviamente normal pois S e tamb em diagonal e matrizes diagonais sempre comutam entre si. Provaremos a rec proca, o que ser a feito por indu ca o. Para n = 1 n ao h a o que provar. Se n = 2, S e da forma b ), com a, b, c C. A condi S = (a c a o SS = S S signica 0 c | a |2 + | b |2 cb bc | a |2 = |c |2 ab ba | b | 2 + | c |2 ,

13 Armand

Borel (19232003).

JCABarata. Curso de F sica-Matem atica

Vers ao de 19 de junho de 2011.

Cap tulo 8

348/1943

Figura 8.2: Um parabol oide el ptico (esquerda) e um parabol oide hiperb olico (direita) em R3 .

Figura 8.3: Um cilindro el ptico (esquerda), um cilindro hiperb olico (centro) e um cilindro parab olico (direita) em R3 .

ca o SS = S S signica ambas b e com n 1 linhas, sendo C uma matriz (n 1) (n 1) triangular superior. A condi | a |2 + b T b Cb bT C |a|2 = ab CC abT B + CC ,

o que implica b = 0, provando que S e diagonal. Procedemos agora por indu ca o, supondo n > 2 e que o lema seja v alido para matrizes (n 1) (n 1) triangulares superiores normais. Se S Mat (C, n) e triangular superior, S e da forma b1 0 . a bT . . . S= , sendo a C , b = . , = . , C bn1 0

mos que bT b = 0, ou seja, |b1 |2 + + |bn1 |2 = 0 e, sendo B a matriz cujos elementos s ao Bij = bi bj . Disso extra portanto, b = . Com isso, camos com CC = C C , ou seja, C e normal. Como C e triangular superior ent ao, pela hip otese indutiva, C e diagonal. Isso, mais o fato provado que b e nulo, implica que S e diagonal, provando o lema.

JCABarata. Curso de F sica-Matem atica

Vers ao de 19 de junho de 2011.

Cap tulo 8

349/1943

8.7

O Teorema de Decomposi c ao de Jordan e a Forma Can onica de Matrizes

Nas se co es anteriores demonstramos condi co es que permitem diagonalizar certas matrizes. Nem todas as matrizes, por em, podem ser diagonalizadas. Podemos nos perguntar, no entanto, qu ao pr oximo podemos chegar de uma matriz diagonal. Mostraremos nesta se ca o que toda matriz A pode ser levada (por uma transforma ca o de similaridade) ` a uma forma pr oxima ` a diagonal, denominada forma can onica de Jordan14 . Resumidamente (a arma ca o precisa ser a apresentada mais adiante), mostraremos que existe uma matriz P tal que P 1 AP tem a seguinte forma: 1 0 0 0 . . . 0 0 1 0 0 0 2 0 0 . . . 0 0 2 3 0 . . . 0 0 0 3 4 . . . 0 0 .. . . .. 0 0 0 .. . 0 0 0 0 , . . . n1 n

(8.85)

n1 0

onde 1 , . . . , n s ao os autovalores de A e onde os i valem 1 ou 0, mas que forma que a matriz diagonal 1 0 0 0 . . . 0 0 0 0 0 0 2 0 0 . . . 0 0 0 3 0 . . . 0 0 0 0 4 . . . 0 0 .. .. . . 0 0 0 .. . 0 0 0 0 , . . . 0 n

(8.86)

n1 0

14 Marie Ennemond Camille Jordan (18381922). A forma can onica de matrizes foi originalmente descoberta por Weierstrass (Karl Theodor Wilhelm Weierstrass (18151897)) e redescoberta por Jordan em 1870.

JCABarata. Curso de F sica-Matem atica

Vers ao de 19 de junho de 2011.

Cap tulo 8

350/1943

e a matriz supra-diagonal

comutam entre si.

0 0 0 0 . . . 0 0

1 0 0 0 . . . 0 0

0 2 0 0 . . . 0 0

0 0 3 0 . . . 0 0

.. .. . .

0 0 0 0 .. 0 0 .

0 0 0 0 , . . . n1 0

(8.87)

O resultado central que provaremos, e do qual as armativas feitas acima seguir ao, diz que toda matriz A pode ser levada por uma transforma ca o do tipo P 1 AP a uma matriz da forma D + N , onde D e diagonal e N e nilpotente (ou seja, tal que N q = 0 para algum q ) e tais que D e N comutam: DN = N D. Essa e a armativa principal do c elebre Teorema da Decomposi ca o de Jordan, que demonstraremos nas p aginas que seguem. Esse Teorema da Decomposi ca o de Jordan generaliza os teoremas sobre diagonalizabilidade de matrizes: para matrizes diagonaliz aveis tem-se simplesmente N = 0 para um P conveniente. Antes de nos dedicarmos ` a demonstra ca o desses fatos precisaremos de alguma prepara ca o.

8.7.1

Resultados Preparat orios

Somas diretas de subespa cos

Seja V um espa co vetorial e V1 e V2 dois de seus subespa cos. Dizemos que V e a soma direta de V1 e V2 se todo vetor v de V puder ser escrito de modo u nico da forma v = v1 + v2 com v1 V1 e v2 V2 . Se V e a soma direta de V1 e V2 escrevemos V = V1 V2 .

Subespa cos invariantes

Se V = V1 V2 e tanto V1 quanto V2 s ao invariantes pela a ca o de A, escrevemos A = A1 A2 onde Ai e A restrita a Vi . Se escolhermos uma base em V da forma {v1 , . . . , vm , vm+1 , . . . , vn }, onde {v1 , . . . , vm } e uma base em V1 e {vm+1 , . . . , vn } e uma base em V2 , ent ao nessa base A ter a a forma A = A1

Um subespa co E de Cn e dito ser invariante pela a c ao de uma matriz A, se Av E para todo v E.

nm, m

m, nm
A2

onde A1 Mat (C, m) e A2 Mat (C, n m). E. 8.28 Exerc cio. Justique a forma (8.88).

(8.88)

A representa ca o (8.88) e dita ser uma representa c ao em blocos diagonais de A, os blocos sendo as sub-matrizes A1 e A2 . Um fato relevante que decorre imediatamente de (8.88) e da Proposi ca o 8.3, p agina 304, e que usaremos freq uentemente adiante, e que se A = A1 A2 ent ao det(A) = det(A1 ) det(A2 ) .

JCABarata. Curso de F sica-Matem atica

Vers ao de 19 de junho de 2011.

Cap tulo 8

351/1943

Operadores nilpotentes

Seja V um espa co vetorial e N : V V um operador linear agindo em V . O operador N e dito ser um operador nilpotente se existir um inteiro positivo q tal que N q = 0. O menor q para o qual N q = 0 e dito ser o ndice de N . Vamos a alguns exemplos. 0 E. 8.29 Exerc cio. Verique que N = 0 0 0 E. 8.30 Exerc cio. Verique que N = 0 0 0 E. 8.31 Exerc cio. Verique que N = 0 0 1 0 e uma matriz nilpotente de ndice 3. 0 1 0 0 a 0 0 c e uma matriz nilpotente de ndice 3. b com a = 0 e b = 0 0 0 ao matrizes nilpotentes de ndice 2. 0 s 0

0 0 0 1 0 0 e N = 0 1 0 0 0 0

O seguinte fato sobre os autovalores de operadores nilpotentes ser a usado adiante. Proposi c ao 8.30 Se N Mat (C, n) e nilpotente ent ao seus autovalores s ao todos nulos. Isso implica que seu polin omio caracter stico e qN (x) = xn , x C. Se o ndice de N e q ent ao o polin omio m nimo de N e mN (x) = xq , x C. No Corol ario 8.5, p agina 356, demonstraremos que uma matriz e nilpotente se e somente se seus autovalores forem todos nulos. Prova da Proposi c ao 8.30. Se N = 0 o ndice e q = 1 e tudo e trivial. Seja N = 0 com ndice q > 1. Seja v = 0 um claro autovetor de N com autovalor : N v = v . Isso diz que 0 = N q v = q v . Logo q = 0 e, obviamente, = 0. E n q ent ao que qN (x) = x . Que o polin omio m nimo e mN (x) = x segue do fato que mN (x) deve ser um divisor de qn (x) (isso segue do Teorema 8.2 junto com o Teorema de Hamilton-Cayley, Teorema 8.3), p agina 315). Logo mN (x) e da forma xk para algum k n. Mas o menor k tal que mN (N ) = N k = 0 e, por deni ca o, igual a q . Isso completa a prova. Mais sobre matrizes nilpotentes ser a estudado na Se ca o 8.7.3 onde, em particular, discutiremos a chamada forma can onica de matrizes nilpotentes. O n ucleo e a imagem de um operador linear

Seja V um espa co vetorial e A : V V um operador linear agindo em V . N(A) := {x V | Ax = 0} .

O n ucleo de A e denido como o conjunto de todos os vetores que s ao anulados por A:

A imagem de A e denida por R(A) := {x V | y V tal que x = Ay } .

JCABarata. Curso de F sica-Matem atica

Vers ao de 19 de junho de 2011.

Cap tulo 8

352/1943

Armamos que N(A) e R(A) s ao dois subespa cos de V . Note-se primeiramente que 0 N(A) e 0 R(A) (por que?). Fora isso, se x e y N(A) ent ao, para quaisquer escalares e , A(x + y ) = Ax + Ay = 0 , provando que combina co es lineares x + x tamb em pertencem a N(A). Analogamente se x e x R(A) ent ao existem y e y V com x = Ay , x = Ay . Logo x + x = A(y + y ) , provando que combina co es lineares x + y tamb em pertencem a R(A). Para um operador A xado, e k N, vamos denir Nk = N(Ak ) e Rk = R(Ak ) . Esses subespa cos Nk e Rk s ao invariantes por A. De fato, se x Nk , ent ao Ak (Ax) = A(Ak x) = A0 = 0, mostrando que k Ax Nk . Analogamente, se x Rk ent ao x = A y para algum vetor y . Logo, Ax = A(Ak y ) = Ak (Ay ), mostrando que Ax Rk . Armamos que Nk Nk+1 e que As demonstra co es dessas armativas s ao quase banais. Se x Nk ent ao Ak x = 0. Isso obviamente implica Ak+1 x = 0. Logo x Nk+1 e, portanto, Nk Nk+1 . Analogamente, se x Rk+1 ent ao existe y tal que x = Ak+1 y . Logo x = Ak (Ay ), o que diz que x Rk . Portanto Rk+1 Rk . Isso diz que os conjuntos Nk formam uma cadeia crescente de conjuntos: {0} N1 N2 Nk V , e os Rk formam uma cadeia decrescente de conjuntos: V R1 R2 Rk { 0 } . (8.91) (8.90) Rk Rk+1 . (8.89)

Consideremos a cadeia crescente (8.90). Como os conjuntos Nk s ao subespa cos de V , e claro que a cadeia n ao pode ser estritamente crescente se V for um espa co de dimens ao nita, ou seja, deve haver um inteiro positivo p tal que Np = Np+1 . Seja p o menor n umero inteiro para o qual isso acontece. Armamos que para todo k 1 vale Np = Np+k .

Vamos provar isso. Se x Np+k ent ao Ap+k x = 0, ou seja, Ap+1 (Ak1 x) = 0. Logo, Ak1 x Np+1 . Dado que k 1 Np = Np+1 , isso diz que A x Np , ou seja, Ap (Ak1 x) = 0. Isso, por sua vez, arma que x Np+k1 . O que zemos ent ao foi partir de x Np+k e concluir que x Np+k1 . Se repetirmos a argumenta ca o k vezes concluiremos que x Np . Logo, Np+k Np . Por (8.89) tem-se, por em, que Np Np+k e, assim, Np+k = Np . Assim, a cadeia (8.90) tem, no caso de V ter dimens ao nita, a forma {0} N1 N2 Np = Np+1 = = Np+k = V . (8.92)

Como dissemos, p ser a daqui por diante o menor inteiro para o qual Np = Np+1 . O lema e o teorema que seguem t em grande import ancia na demonstra ca o do Teorema de Decomposi ca o de Jordan. Lema 8.4 Com as deni c oes acima, Np Rp = {0}, ou seja, os subespa cos Np e Rp t em em comum apenas o vetor nulo.

Demonstra c ao. Seja x tal que x Np e x Rp . Isso signica que Ap x = 0 e que existe y tal que x = Ap y . Logo, 2p p A y = A x = 0, ou seja, y N2p . Pela deni ca o de p tem-se que N2p = Np . Assim, y Np . Logo Ap y = 0. Mas, pela p pr opria deni ca o de y valia que A y = x. Logo x = 0. Esse lema tem a seguinte conseq u encia importante.

JCABarata. Curso de F sica-Matem atica

Vers ao de 19 de junho de 2011.

Cap tulo 8

353/1943

Teorema 8.17 Com as deni c oes acima vale que V = Np Rp , ou seja, cada x V pode ser escrito de modo u nico na forma x = xn + xr , onde xn Np e xr Rp . Demonstra c ao. Seja m a dimens ao de Np e seja {u1 , . . . , um } uma base em Np . Vamos estender essa base, incluindo vetores {vm+1 , . . . , vn } de modo que {u1 , . . . , um , vm+1 , . . . , vn } seja uma base em V . Armamos que {Ap vm+1 , . . . , Ap vn } e uma base em Rp . Seja x Rp e seja y V tal que x = Ap y . Como todo vetor de V , y pode ser escrito como combina ca o linear de elementos da base {u1 , . . . , um , vm+1 , . . . , vn }:
m n

y =
i=1

i ui +
i=m+1

i vi .

Logo, x =

i A ui +
i=1 i=m+1

i A vi =
i=m+1

i Ap vi .

(8.93)

Os vetores {Ap vm+1 , . . . , Ap vn } s ao linearmente independentes. Isso se mostra com o seguinte argumento. Se existirem
n n n

escalares m+1 , . . . , n tais que


i=m+1

i Ap vi = 0, ent ao ter amos Ap


n m i=m+1

i vi

= 0, ou seja,
i=m+1

i vi Np . Isso

implica que existem constantes 1 , . . . , m tais que


i=m+1

i vi =
i=1

i ui , pois os vetores {u1 , . . . , um } s ao uma base

em Np . Ora, como {u1 , . . . , um , vm+1 , . . . , vn } s ao linearmente independentes, segue que os i s e os j s s ao todos nulos. Isso prova que {Ap vm+1 , . . . , Ap vn } s ao linearmente independentes e, portanto, por (8.93), formam uma base em Rp . Isso incidentalmente provou que a dimens ao de Rp e n m. Temos, portanto, que dim (Np ) + dim (Rp ) = dim (V ). Para i = m + 1, . . . , n dena-se ui = Ap vi . Armamos que o conjunto de vetores {u1 , . . . , um , um+1 , . . . , un } = {u1 , . . . , um , Ap vm+1 , . . . , Ap vn } e tamb em linearmente independente e, portanto, forma uma base em V . Suponhamos que haja constantes escalares 1 , . . . , n tais que
n m n

0 =
i=1

i ui =
i=1

i ui + Ap
i=m+1 n

i vi

Isso implica, obviamente,


m i=1

i ui = Ap

i vi
i=m+1

O lado esquerdo dessa igualdade e um elemento de Np (pois u1 , . . . , um s ao uma base em Np ), enquanto que o lado esquerdo e obviamente um elemento da imagem de Ap , ou seja, de Rp . Contudo, j a vimos (Lema 8.4) que o u nico vetor que Np e Rp t em em comum e o vetor nulo. Logo,
m

i ui = 0
i=1

(8.94)

i Ap vi = 0 .
i=m+1

(8.95)

A rela ca o (8.94) implica 1 = = m = 0, pois {u1 , . . . , um } e uma base em Np . A rela ca o (8.95) implica m+1 = = n = 0, pois {Ap v1 , . . . , Ap vm } e uma base em Rp . Assim, todos os i s s ao nulos, provando que {u1 , . . . , um , um+1 , . . . , un } = {u1 , . . . , um , Ap vm+1 , . . . , Ap vn } e um conjunto de n vetores linearmente independentes.

JCABarata. Curso de F sica-Matem atica

Vers ao de 19 de junho de 2011.

Cap tulo 8

354/1943

Conseq uentemente, todo x V pode ser escrito na forma


n m n

x =
i=1

i ui =
i=1

i ui + Ap
i=m+1 x r Rp xn Np

i vi

Provar a unicidade dessa decomposi ca o ca como exerc cio. Isso completa a demonstra ca o. Uma das coisas que o teorema que acabamos de demonstrar diz e que, dado um operador A, o espa co V pode ser decomposto em uma soma direta de dois subespa cos, invariantes por A: um onde A e nilpotente, Np , e outro onde A e invers vel, Rp . A e nilpotente em Np pois Ap x = 0 para todo elemento x de Np . A e invers vel em Rp pois se x Rp e tal que Ax = 0 isso implica x N1 Np . Mas x s o pode pertencer a Np e a Rp se for nulo. Logo, em Rp , Ax = 0 se e somente se x = 0, provando que A e invers vel15. Para refer encia futura formulemos essa armativa na forma de um teorema: Teorema 8.18 Se A e um operador linear n ao-nulo agindo em um espa co vetorial V = Cn ent ao e poss vel decompor V em dois subespa cos invariantes por A, V = S T , de forma que A restrito a S e nilpotente, enquanto que A restrito a T e invers vel. Esse ser a o teorema b asico do qual extrairemos a demonstra ca o do Teorema de Decomposi ca o de Jordan.

8.7.2

O Teorema da Decomposi c ao de Jordan

Chegamos agora ao resultado mais importante desta se ca o, o Teorema da Decomposi ca o de Jordan16, um importante teorema estrutural sobre matrizes de import ancia em v arios campos, por exemplo na teoria das equa co es diferenciais ordin arias. Para tais aplica co es, vide Cap tulo 12, p agina 465. O Teorema da Decomposi ca o de Jordan tamb em tem certa relev ancia na Teoria de Grupos, e o usaremos para provar que toda matriz n n complexa invers vel (ou seja, todo elemento do grupo GL(C, n)) pode ser escrita como exponencial de outra matriz (Proposi ca o 9.11, p agina 400). No Cap tulo 9 usaremos o Teorema da Decomposi ca o de Jordan para provar a identidade u til det(eA ) = eTr(A) , v alida para qualquer matriz n n real ou complexa (Proposi ca o 9.7, p agina 398). Enunciado e demonstra c ao do Teorema da Decomposi c ao de Jordan Teorema 8.19 (Teorema da Decomposi c ao de Jordan) Seja A um operador linear agindo no espa co V = Cn e seja {1 , . . . , r } o conjunto de seus autovalores distintos. Ent ao, existem r subespa cos S1 , . . . , Sr tais que V = S1 . . . Sr e tais que cada Si e invariante por A. Ou seja, A = A1 . . . Ar , onde Ai e A restrita a Si . Fora isso, e a matriz identidade em Si e onde Ni e nilpotente. Por m, a dimens ao cada Ai , e da forma Ai = i i + Ni , onde i si de cada subespa co Si e igual ` a multiplicidade alg ebrica do autovalor i .

Demonstra c ao. Seja {1 , . . . , r } o conjunto dos autovalores distintos de A e seja ni a multiplicidade alg ebrica do autovalor i . Seja A1 = A 1 . Pelo Teorema 8.18, p agina 354, V pode ser escrito como V = S1 T1 , onde S1 e T1 s ao invariantes por A1 , sendo A1 nilpotente em S1 e invers vel em T1 . Assim, A1 e da forma A1 = N1 M1 com N1 nilpotente e M1 invers vel. Logo A = 1 + A1 = (1 S1 + N1 ) (1 T1 + M1 ) , (8.96)

e a matriz identidade em S1 etc. Vamos mostrar que a dimens ao de S1 e igual ` a multiplicidade alg ebrica de 1 . onde S1 Por (8.96) o polin omio caracter stico de A e qA () = det( A) = det(( 1 )S1 N1 ) det(( 1 )T1 M1 ) .
que esse argumento s o funciona em espa cos vetoriais V que tenham dimens ao nita, o que estamos supondo aqui. Camille Jordan (18381922). A forma can onica de matrizes (que ser a discutida mais adiante) foi originalmente descoberta por Weierstrass (Karl Theodor Wilhelm Weierstrass (18151897)) e redescoberta por Jordan em 1870.
16 Marie Ennemond 15 Lembre-se

JCABarata. Curso de F sica-Matem atica

Vers ao de 19 de junho de 2011.

Cap tulo 8

355/1943

omio caracter stico de N1 , tem-se Se qN1 denota o polin det(( 1 )S1 N1 ) = qN1 ( 1 ) = ( 1 )s1 , onde, na u ltima igualdade, usamos a Proposi ca o 8.30, p agina 351, sobre a forma do polin omio caracter stico de uma omio caracter stico de M1 . Como matriz nilpotente. Da , segue que qA () = ( 1 )s1 qM1 ( 1 ), sendo qM1 o polin M1 e invers vel, M1 n ao tem o zero como autovalor. Logo, qM1 (0) = 0. Portanto s1 e igual ` a multiplicidade de 1 como raiz de qA , ou seja, e igual a n1 , a multiplicidade alg ebrica de 1 . A id eia agora e prosseguir decompondo agora o operador 1 T1 + M1 que aparece em (8.96) da mesma maneira como zermos acima com A. Evocando novamente o Teorema 8.18, p agina 354, T1 pode ser escrito como T1 = S2 T2 , onde S2 e T2 s ao invariantes por A2 , sendo A2 nilpotente em S2 e invers vel em T2 . Assim, V = S1 S2 T2 . Agindo em T1 = S2 T2 , A2 e da forma A2 = N2 M2 com N2 nilpotente e M2 invers vel. Logo A = 2 T1 + A2 = (2 S2 + N2 ) (2 T2 + M2 ) . Vamos, como acima, mostrar que a dimens ao de S2 e igual ` a multiplicidade alg ebrica de 2 . Pela deni ca o, Logo, A = (1 S1 + N1 ) A = (1 S1 + N1 ) (2 S2 + N2 ) (2 T2 + M2 ) . qA () = det (( 1 )S1 N1 ) det (( 2 )S2 N2 ) det (( 2 )T2 M2 ) . qA () = ( 1 )n1 ( 2 )s2 qM2 ( 2 ) . e assim um subespa co de Como M2 e invers vel, M2 n ao tem autovalor zero e, assim, qM2 (0) = 0. Logo, s2 = n2 . T2 dimens a o n n1 n2 . (8.97) Seja A = 1 T1 + M1 e que age em T1 , que e um espa co de dimens ao n n1 . Denimos A2 = A 2 T1 .

Portanto, pelos mesmos argumentos usados acima,

Prosseguindo nas mesmas linhas, ap os r passos chegaremos a um subespa co Tr de dimens a o n n1 nr = 0 (por (8.28), p agina 306). A , teremos V = S1 Sr , onde cada Si tem dimens a o ni e A = (1 S1 + N1 ) (r Sr + Nr ) , onde os Ni s s ao todos nilpotentes. Isso completa a demonstra ca o. Um corol ario importante do Teorema de Decomposi ca o de Jordan e o seguinte: Teorema 8.20 Para toda matriz A Mat (C, n) existe uma matriz invers vel P Mat (C, n) tal que P 1 AP = D + N , onde D e uma matriz diagonal formada pelos autovalores de A e N e uma matriz nilpotente e de tal forma que D e N comutam: DN = N D. Conseq uentemente, toda matriz A Mat (C, n) pode ser escrita na forma A = Ad + An com Ad An = An Ad , sendo Ad diagonaliz avel e An nilpotente, a saber, Ad = P DP 1 e An = P N P 1 , com D e N dados acima.

Demonstra c ao do Teorema 8.20. O Teorema 8.19 est a dizendo que, numa base conveniente, A tem a forma de blocos

JCABarata. Curso de F sica-Matem atica

Vers ao de 19 de junho de 2011.

Cap tulo 8

356/1943

diagonais

A1 0 A = . . . 0

0 A2 . . . 0

.. .

ou seja, onde

1 s1 + N1 0 0 0 = . . . . . . Ar 0 A = D+N ,

2 s2 + N2

..

. . .

. . .

r sr + Nr

(8.98)

1 s1 0 D = . . . 0

0 2 s2 . . . 0

.. .

r sr

0 0 = diag 1 , . . . , 1 , . . . , r , . . . , r . . . s1 vezes sr vezes 0 N2 . . . 0 .. . 0 0 . . . . Nr

Acima si e a dimens ao do subespa co Si . f E acil de se ver que N e uma matriz nilpotente, pois se o ki e o ndice de Ni (ou seja, ki e o menor inteiro positivo ao para k := max (k1 , . . . , kr ) tem-se para o qual Niki = 0), ent Nk (N1 )k 0 = . . . 0 0 .. 0 (N2 )k . . . 0 . 0 . . . (Nr )k = 0.

N1 0 N = . . . 0

(8.99)

Em verdade, k = max (k1 , . . . , kr ) e o ndice de N (por que?).

ca o. Por m, como cada Ni comuta com i si , ca claro que D e N comutam. Isso completa a demonstra Corol ario 8.5 Uma matriz M Mat (C, n) e nilpotente se e somente se todos os seus autovalores forem nulos.

JCABarata. Curso de F sica-Matem atica

Vers ao de 19 de junho de 2011.

Cap tulo 8

357/1943

Prova. A Proposi ca o 8.30, p agina 351, arma que se M e nilpotente todos os seus autovalores s ao nulos. O Teorema 8.20, p agina 355, arma que se os autovalores de M s ao nulos, ent ao existe P tal que P 1 M P = N , nilpotente. Isso implica que M e nilpotente.

8.7.3

Matrizes Nilpotentes e sua Representa c ao Can onica

Os teoremas que estudamos acima nesta se ca o revelam a import ancia de matrizes nilpotentes. Um fato relevante e que elas podem ser representadas de uma forma especial, denominada forma can onica, da qual traremos logo abaixo. Antes, alguma prepara ca o se faz necess aria. Seja N Mat (C, n) uma matriz nilpotente de ndice q , ou seja, N q = 0, mas N q1 = 0. Para uso futuro, provemos o seguinte lema: Lema 8.5 Seja N uma matriz nilpotente de ndice q . Est ao existe um vetor v = 0 tal que os q vetores v, N v, N 2 v, ..., N q 1 v , (8.100)

s ao linearmente independentes. Fora isso, o subespa co q -dimensional Jv, q := v, N v, N 2 v, . . . , N q1 v de V gerado por esses q vetores e invariante por N . Prova. Se q = 1, ent ao N = 0 e n ao h a nada a provar, pois a arma ca o e trivialmente verdadeira para qualquer v = 0. Seja ent ao q > 1 (em cujo caso N = 0, trivialmente). Sabemos, por hip otese, que a matriz N q1 e n ao-nula. Isso q 1 imediato que os vetores signica que existe pelo menos um vetor v = 0 tal que N v = 0. Fixemos um tal vetor. E N v, N 2 v, . . . , N q1 v s ao todos n ao-nulos pois, se tiv essemos N j v = 0 para algum 1 j < q 1, ent ao, aplicando-se q1j q 1 N a esquerda, ter ` amos N v = 0, uma contradi ca o. Sejam agora 1 , . . . , q escalares tais que 1 v + 2 N v + 3 N 2 v + + q N q1 v = 0 . (8.101)

Aplicando-se N q1 nessa igualdade e lembrando que N q = 0, conclu mos que 1 N q1 v = 0. Como N q1 v = 0, segue que 1 = 0 e, com isso, (8.101) ca 2 N v + 3 N 2 v + + q N q1 v = 0 . (8.102) Aplicando agora N q2 nessa igualdade conclu mos que 2 = 0. Prosseguindo, conclu mos depois de q passos que todos os escalares j s ao nulos. Isso prova que os q vetores de (8.100) s ao linearmente independentes. Que o subespa co Jv, q denido acima e invariante por N e evidente pois, para quaisquer escalares 1 , . . . , q , tem-se N 1 v + 2 N v + + q N q 1 v = 1 N v + 2 N 2 v + + q1 N q1 v Jv, q .

O seguinte teorema e central para o que segue. Teorema 8.21 Se N e uma matriz nilpotente de ndice q agindo em V e v um vetor com a propriedade que N q1 v = 0, ent ao existe um subespa co K de V tal que Jv, q K = {0}, tal que V = Jv, q K e tal que K e tamb em invariante por N. Prova.17 A prova e feita por indu ca o em q . Note-se que se q = 1, ent ao N = 0 e a armativa e trivial, pois podemos tomar como v qualquer vetor n ao-nulo, Jv, q seria o subespa co gerado por esse v e K o subespa co complementar a v , que e trivialmente invariante por N , pois N = 0. Vamos supor ent ao que a arma ca o seja v alida para matrizes nilpotentes de ndice q 1 e provar que a mesma e v alida para matrizes nilpotentes de ndice q . O que desejamos e construir um subespa co K com as propriedades desejadas, ou seja, tal que V = Jv, q K , sendo K invariante por N .
17 Extra da,

com modica co es, de [82].

JCABarata. Curso de F sica-Matem atica

Vers ao de 19 de junho de 2011.

Cap tulo 8

358/1943

Seja V0 = R(N ) o conjunto imagem de N . Sabemos que V0 e um subespa co de V e que e invariante por N . Fora isso, N e nilpotente de ndice q 1 agindo em V0 (por que?) claro que N q2 v0 = N q1 v = 0. Assim, pelo Lema 8.5, o subespa Seja v0 = N v V0 . E co (q 1)-dimensional Jv0 , q1 = v0 , N v0 , . . . , N q2 v0 = N v, N 2 v, . . . , N q1 v = JN v, q1 , que e um subespa co de V0 , e invariante por N e, da hip otese indutiva, conclu mos que existe um subespa co K0 de V0 que e invariante por N tal que JN v, q1 K0 = {0} e tal que V0 = JN v, q1 K0 . ca o: Seja agora K1 := {x V | N x K0 }. Vamos provar a seguinte arma

I. Todo vetor x de V pode ser escrito na forma x = y + z onde y Jv, q e z K1 .

Para provar isso, notemos que para qualquer x V vale certamente que N x V0 . Portanto, como pela hip otese indutiva V0 = JN v, q1 K0 , podemos escrever N x = y + z , com y JN v, q1 e z K0 . Como y JN v, q1 , y e da forma de uma combina ca o linear y = 1 N v + + q1 N q1 v = N y , onde y := 1 v + 2 N v + + q1 N q2 v e um elemento de Jv, q . Logo, z = N (x y ). Como z K0 , segue que z := x y K1 . Assim, x = y + z , com y Jv, q e z K1 . Isso provou I.

Note que a arma ca o feita em I n ao signica que V = Jv, q K1 , pois os subespa cos Jv, q e K1 podem ter uma intersec ca o n ao-trivial. Tem-se, por em, o seguinte: II. Jv, q K0 = {0}.

Provemos essa arma ca o. Seja x Jv, q K0 . Como x Jv, q , x e da forma x = 1 v + 2 N v + + q N q1 v . 2 q 1 Logo N x = 1 N v + 2 N v + + q1 N v JN v, q1 . Agora, como x K0 e, por hip otese, K0 e invariante por N , segue que N x K0 . Logo, N x JN v, q1 K0 . Todavia, mencionamos acima que JN v, q1 K0 = {0}. Logo, N x = 0, ou seja, 0 = N x = 1 N v + 2 N 2 v + + q1 N q1 v . Como os vetores N v, . . . , N q1 v s ao linearmente independentes, conclu mos que 1 = q1 = 0. Logo, x = q N q1 v . Isso signica que x JN v, q1 . Demonstramos, ent ao, que se x Jv, q K0 ent ao x JN v, q1 K0 mas, como JN v, q1 K0 = {0}, segue que x = 0. Isso conclui a prova de II.

III. K0 e Jv, q K1 , s ao dois subespa cos disjuntos de K1 . evidente que Jv, q K1 A demonstra ca o e muito simples. E e subespa co de K1 . Como K0 e invariante pela a ca o de N , segue que se x K0 ent ao N x K0 . Pela deni ca o, isso diz que x K1 e conclu mos que K0 e um subespa co e K1 . Que K0 e Jv, q K1 s ao subespa cos disjuntos, segue do fato que K0 (Jv, q K1 ) = K1 (Jv, q K0 ) = K1 {0} = {0} .
A arma ca o III implica que K1 = (Jv, q K1 ) K0 K0 para algum subespa co K 0 de K1 (n ao necessariamente u nico). Seja agora K := K0 K0 . Note que K1 = (Jv, q K1 ) K e, portanto, II

(Jv, q K1 ) K = {0} .

(8.103)

Provaremos que esse K possui as propriedades desejadas, ou seja, que V = Jv, q K , sendo K invariante por N . Isso e feito em tr es passos. 1. Jv, q e K s ao subespa cos disjuntos, ou seja, Jv, q K = {0}, pois, como K K1 , segue que K = K K1 e, portanto, Jv, q K = Jv, q (K K1 ) = (Jv, q K1 ) K
(8.103)

{0} .

2. Jv, q K cont em os vetores de Jv, q e de (Jv, q K1 ) K = K1 . Por I, isso implica que Jv, q K = V . 3. K e invariante por N , pois o fato que K K1 , implica, pela deni ca o de K1 , que N K N K1 K0 K .

JCABarata. Curso de F sica-Matem atica

Vers ao de 19 de junho de 2011.

Cap tulo 8

359/1943

A prova do Teorema 8.21 est a completa A principal conseq u encia do Teorema 8.21 e a seguinte. Proposi c ao 8.31 Seja N Mat (C, n) uma matriz nilpotente de ndice q . Ent ao, existem 1. um inteiro positivo r, com 1 r n, 2. r n umeros inteiros positivos n q1 q2 qr 1, com q1 + + qr = n, 3. r vetores v1 , . . . , vr satisfazendo N qj vj = 0 mas N qj 1 vj = 0, j = 1, . . . , r, tais que V = Jv1 , q1 Jvr , qr .

Prova. Se q = 1 ent ao N = 0. Basta tomar r = n e escolher v1 , . . . , vn uma base qualquer em V . Os qj s s ao todos iguais a 1. Consideremos ent ao q > 1 com N = 0. Tomemos q1 = q . Pelo Teorema 8.21, existem um vetor v1 = 0 e um subespa co K 1 , invariante por N tais que V = Jv1 , q1 K 1 . Como K 1 e invariante por N , podemos tamb em dizer que a matriz N e nilpotente quando restrita a K 1 (j a que e claro que q2 q = q1 . nilpotente em todo V ). Denotemos por q2 o ndice de N quando restrita a K 1 . E

Assim, podemos aplicar o Teorema 8.21 para a matriz N restrita a K 1 e concluir que existe v2 = 0 em K 1 e um subespa co K 2 de K 1 , invariante por N , tais que K 1 = Jv2 , q2 K 2 . Note que N q2 v2 = 0, pois v2 K 1 . Com isso, temos V = Jv1 , q1 Jv2 , q2 K 2 .

Novamente K 2 e invariante por N e, como K 2 e um subespa co de K 1 . O ndice de N em K 2 ser a q3 q2 q1 .

O espa co V tem dimens ao nita. Assim, a prova se conclu repetindo o procedimento acima um n umero nito r de vezes. Note que N qj vj = 0, pois N q1 v1 = 0, e vj K j 1 para todo j = 2, . . . , r. Pela constru ca o acima, e claro que q1 + + qr = n, a dimens ao de V , e que os n vetores v1 , N v1 , . . . , N q1 1 v1 , v2 , N v2 , . . . , N q2 1 v2 , . . . , vr , N vr , . . . , N qr 1 vr

s ao linearmente independentes e formam uma base em V . Vamos denot a-los (na ordem em que aparecem acima) por b1 , . . . , bn . Note agora que, pela constru ca o, N bj = bj +1 , para j em cada um dos conjuntos {1, . . . , q1 1}, {1 + q1 , . . . , q1 + q2 1}, {1 + q1 + q2 , . . . , q1 + q2 + q3 1} , ... {1 + q1 + + qr1 , . . . , q1 + + qr 1} , (8.104)

com l = 0, . . . , r 1, sendo que N bj = 0 para todo j na forma q1 + + ql , l = 1, . . . , r. ultimas arma co es. E. 8.32 Exerc cio importante para compreender o que segue. Justique as Isso signica que na base b1 , . . . , bn os elementos de matriz de N s ao todos nulos exceto aqueles na forma Nj, j +1 com j em algum dos conjuntos listados em (8.104), em cujo caso Nj, j +1 = 1. Pictoriamente, isso diz-nos que na base b1 , . . . , bn a matriz N assume uma forma genericamente ilustrada na Figura 8.4. Essa e a denominada forma can onica da matriz nilpotente N ou representa c ao can onica da matriz nilpotente N , que descrevemos mais detalhadamente no que segue.

JCABarata. Curso de F sica-Matem atica

Vers ao de 19 de junho de 2011.

Cap tulo 8

360/1943

(q 1) vezes
1

}
N =

(q 1) vezes
2

}
0
8.7.4

0
1 0 0
(q 1) vezes
r

0 0 1

Figura 8.4: Forma can onica t pica de uma matriz nilpotente N . Os elementos da primeira supra-diagonal podem valer 0 ou 1. Todos os demais elementos de matriz s ao nulos.

Os elementos da diagonal principal s ao todos nulos. Os u nicos elementos n ao-nulos da matriz podem estar localizados apenas na diagonal imediatamente acima da principal, ou seja, aquela diagonal formada por elementos de matriz do tipo Nj, j +1 com j = 1, . . . , n 1. Chamaremos essa diagonal de primeira supra-diagonal. Os elementos da primeira supra-diagonal podem ser 0 ou 1, da forma seguinte: a primeira supra-diagonal possuir a r leiras. As primeiras r 1 leiras s ao formadas por qj elementos, j = 1, . . . , n 1, sendo os primeiros qj 1 elementos iguais a 1 e o u ltimo igual a 0. A u ltima leira ter a qr 1 elementos iguais a 1. Assim, se qr = 1, o u ltimo elemento da primeira supra-diagonal ser a nulo, proveniente da (r 1)- esima leira (essa eau nica forma de aparecer um zero no u ltimo elemento da primeira supra-diagonal). Note que zeros consecutivos podem ocorrer, se tivermos alguns qj s iguais a 1. Note tamb em que os elementos da primeira supra-diagonal podem ser todos nulos (o que valer a se r = n, em cujo caso q1 = = rn = 1. Isso s o pode ocorrer se N = 0 e, nesse caso, q = 1) ou todos iguais a 1 (o que valer a se r = 1, em cujo caso q1 = n).

A Forma Can onica de Matrizes

Finalizamos esta se ca o e nossa discuss ao sobre o Teorema da Decomposi ca o de Jordan e suas conseq u encias reunindo o que descobrimos at e aqui. Se A Mat (C, n) o Teorema 8.19, p agina 354 ensinou-nos que numa base conveniente (ou seja, por uma trans-

JCABarata. Curso de F sica-Matem atica

Vers ao de 19 de junho de 2011.

Cap tulo 8

361/1943

1 forma ca o de similaridade P0 AP0 ), toda matriz A tem a forma de blocos diagonais:

1 P0 AP0

A1 0 = . . . 0

0 A2 . . . 0

.. .

sendo 1 , . . . , r os autovalores distintos de A. O j - esimo bloco e de tamanho nj nj , sendo que nj e a multiplicidade alg ebrica do autovalor j . As matrizes Nj s ao nilpotentes.
c Cada matriz Nj pode ser levada ` a sua forma can onica Nj (tal como explicado na Figura 8.4, p agina 360, e no que se lhe segue) em uma base conveniente, ou seja, por uma transforma ca o de similaridade Pj1 Nj Pj . Assim, denindo

1 n1 + N1 0 0 0 = . . . . . . Ar 0

2 n2 + N2

..

. . .

. . .

r nr + Nr

(8.105)

P1 0 P = . . . 0

0 P2 . . . 0

.. .

0 0 , . . . Pr

(8.106)

JCABarata. Curso de F sica-Matem atica

Vers ao de 19 de junho de 2011.

Cap tulo 8

362/1943

1 vemos que P 1 (P0 AP0 )P = (P0 P )1 A(P0 P ), sendo que, por (8.105), 1 P1

(1 n1 + N1 ) P1

1 P 1 (P0 AP0 )P

1 P2 (2 n2 + N2 ) P1

..

. . .

. . .

. . .

1 (r nr + Nr ) Pr Pr

1 n1 + 0 . . . 0

c N1

c 2 n2 + N2

..

. . .

. . .

c r nr + Nr

(8.107)

E. 8.33 Exerc cio. Complete os detalhes. A matriz nal de (8.107) e denominada forma can onica da matriz A, ou forma can onica de Jordan da matriz A. Como dissemos, toda matriz A assume essa forma numa certa base. Devido ao fato de todos as sub-matrizes nilpotentes c Nj terem a forma can onica, os u nicos elementos n ao-nulos da forma can onica da matriz A podem estar ou na diagonal principal (sendo estes os autovalores de A, cada um aparecendo em uma leira de nj elementos), ou na primeira supradiagonal, sendo que estes valem apenas 0 ou 1 e seguem as regras descritas acima. Isso e ilustrado na Figura 8.5, A Figura 8.5, mostra a forma can onica de uma matriz que possui 4 autovalores distintos 1 , 2 , 3 e 4 . A primeira supra-diagonal e formada pela seq u encia de n umeros
1 a 1 b 1 c 1 d 1 , . . . , 1 , 0 , 1 , . . . , 1 , 0 , 1 , . . . , 1 , 0 , 1 , . . . , 1 ,

(8.108)

j sendo que os i assumem apenas os valores 0 ou 1, de acordo com as regras explicadas acima quando discutimos a forma can onica de matrizes nilpotentes. Todos os elementos fora da diagonal principal e da primeira supradiagonal s ao nulos. O primeiro bloco e de dimens ao (a + 1) (a + 1), o segundo bloco e de dimens ao (b + 1) (b + 1) etc., sendo a + 1 a multiplicidade alg ebrica de 1 , b + 1 a multiplicidade alg ebrica de 2 etc. interessante notar que na primeira supra-diagonal, sempre ocorrem zeros nos pontos localizados fora dos blocos, ou E seja, nos pontos onde ocorrem transi co es entre dois autovalores distintos (indicados por setas na Figura 8.5). Esses s ao os zeros que ocorrem explicitamente na lista (8.108).

Por m, comentamos que a forma can onica n ao e exatamente u nica, pois e poss vel ainda fazer transforma co es de similaridade que permutem os blocos de Jordan da matriz. Al em disso, dentro de cada subespa co invariante (onde cada

JCABarata. Curso de F sica-Matem atica

Vers ao de 19 de junho de 2011.

Cap tulo 8

363/1943

bloco age) e poss vel fazer certas permuta co es dos elementos da base, de modo a preservar a diagonal e permutar os i s da primeira supradiagonal.

8.8

Algumas Representa co es Especiais de Matrizes

Nas se co es anteriores apresentamos algumas formas especiais de representar matrizes com determinadas caracter sticas, como aquelas expressas no Teorema Espectral e no Teorema de Jordan. Nesta se ca o apresentaremos outras representa co es, relevantes em certos contextos, como a decomposi ca o polar.

8.8.1

A Decomposi c ao Polar de Matrizes

bem conhecido o fato de que todo n E umero complexo z pode ser escrito na forma polar z = |z |ei , onde |z | 0 e a uma arma ca o an aloga v alida para matrizes A Mat (C, n), [, ). Tem-se que |z | = zz e ei = z |z |1 . H a qual e muito u til, e da qual trataremos nesta se ca o. Antes de enunciarmos esse resultado de forma mais precisa (o Teorema da Decomposi ca o Polar, Teorema 8.22, abaixo), fa camos algumas observa co es preliminares. Seja A Mat (C, n) e seja a matriz A A. Notemos primeiramente que (A A) = A A = A A, ou seja, A A e auto-adjunta. Pelo Teorema 8.13, p agina 338, e poss vel encontrar um conjunto ortonormal {vk , k = 1, . . . , n} de autovetores de A A, com autovalores dk , k = 1, . . . , n, respectivamente, sendo que a matriz P := v1 , . . . , vn (8.109)

Com esses fatos ` a m ao, vamos denir uma matriz diagonal, que denotaremos sugestivamente por D1/2 , por D1/2 := 2 diag ( d1 , . . . , dn ). Tem-se que D1/2 = D, uma propriedade obvia18 . Note-se tamb em que D1/2 = D1/2 , pois e real. Os n umeros positivos d1 , . . . , dn s ao freq uentemente denominados valores singulares de A. cada dk Denamos agora a matriz A A, por A A := P D1/2 P . (8.110) 2 Essa matriz A A e auto-adjunta, pois A A = P D1/2 P = P D1/2 P = A A. Observemos que A A = P (D1/2 )2 P = P DP = A A. Disso segue que det A A
2

(para a nota ca o, vide (8.9)) e unit aria e diagonaliza A A, ou seja, P (A A)P = D, sendo D a matriz diagonal D := diag (d1 , . . . , dn ), cujos elementos da diagonal s ao os autovalores de A A. Os autovalores dk s ao todos maiores ou iguais a zero. De fato, se vk = 0 e um autovetor de A A com autovalor dk , teremos dk vk 2 = dk vk , vk C = vk , Bvk C = vk , A Avk C = Avk , Avk C = Avk 2 . Logo, dk = Avk 2 / vk 2 0.

= det

A A

= det(A A) = det(A ) det(A) = det(A) det(A) = | det(A)|2 .

A A = | det(A)| e, portanto, A A e invers vel se e somente se A o for. Alguns autores denotam a matriz A A por |A|, por analogia com o m odulo de um n umero complexo. Podemos agora formular e demonstrar o resultado que procuramos: Provamos assim que det Teorema 8.22 (Teorema da Decomposi c ao Polar) Seja A Mat (C, n). Ent ao, existe uma matriz unit aria U Mat (C, n) tal que (8.111) A = U A A . Se A e invers vel, ent ao U e univocamente determinada. A representa c ao (8.111) e denominada representa ca o polar de A.
n ao e a u nica matriz com essa propriedades, pois qualquer matriz do tipo diag ( d1 , . . . , dn ), com os sinais escolhidos independentemente uns dos outros, tamb em tem como quadrado a matriz D .
18 Essa

JCABarata. Curso de F sica-Matem atica

Vers ao de 19 de junho de 2011.

Cap tulo 8

364/1943

Prova. Sejam, como acima, dk , k = 1, . . . , n os autovalores de A A com autovetores respectivos vk , k = 1, . . . , n. Sabemos pelo Teorema 8.13, p agina 338 que podemos escolher os vk s de forma que vk , vl C = k l . Como vimos acima, os autovalores dk satisfazem dk 0. Sem perda de generalidade, vamos sup o-los ordenados de forma que dk > 0 para todo k = 1, . . . , r e dk = 0 para todo k = r + 1, . . . , n. Com essa escolha, tem-se que Avk = 0 para todo k = r + 1, . . . , n , pois de A Avk = 0, segue que 0 = vk , A Avk
C

(8.112)

= Avk , Avk

= Avk

Para k = 1, . . . , r, sejam wk os vetores denidos da seguinte forma: 1 wk := Avk , dk f E acil ver que wk , wl
C

k = 1, . . . , r .

(8.113)

1 Avk , Avl = dk dl

1 = A Avk , vl dk dl

dk = vk , vl dk dl

dk = k l = k l , dk dl

para todos k, l = 1, . . . , r. Assim, o conjunto de vetores {wk , k = 1, . . . , r} forma um conjunto ortonormal. A eles podemos acrescentar um novo conjunto {wk , k = r + 1, . . . , n}, escolhido arbitrariamente, de vetores ortonormais pertencentes ao complemento ortogonal do subespa co gerado por {wk , k = 1, . . . , r} e construir assim, um conjunto ortonormal {wk , k = 1, . . . , n}. Sejam agora a matriz P , denida em (8.109) e as seguintes matrizes de Mat (C, n): Q := w1 , . . . , wn , U := QP

(para a nota ca o, vide (8.9)). Como {vk , k = 1, . . . , n} e {wk , k = 1, . . . , n} s ao dois conjuntos ortonormais, segue que P e Q s ao matrizes unit arias (por qu e?) e, portanto, U tamb em e unit aria. 1/2 1/2 f E acil ver que AP = QD , onde D d1 , . . . , dn , De fato, def idiag AP
(8.109)

A v1 , . . . , vn

(8.12)

Av1 , . . . , Avn Av1 , . . . , Avr 0, . . . , 0 d1 w1 , . . . , dr wr 0, . . . , 0

(8.112)

(8.113)

(8.14)

w1 , . . . , wn D1/2 = QD1/2 .
(8.110)

Agora, de AP = QD1/2 , segue que A = QD1/2 P = U P D1/2 P

Para mostrar que U e univocamente determinado se A for invers vel, suponhamos que exista U tal que A = U A A = U e invers vel se e somente se A o for. Logo, se A e invers vel, a igualdade comentamos acima, A A A A. Como ao seja invers vel a arbitrariedade de U reside U A A = U A A implica U = U , estabelecendo a unicidade. Caso A n na escolha dos vetores ortogonais {wk , k = r + 1, . . . , n}. O seguinte corol ario e elementar:

U A A, que e o que quer amos provar.

Teorema 8.23 Seja A Mat (C, n). Ent ao, existe uma matriz unit aria V Mat (C, n) tal que A = AA V . Se A e invers vel, ent ao V e univocamente determinada.

(8.114)

JCABarata. Curso de F sica-Matem atica

Vers ao de 19 de junho de 2011.

Cap tulo 8

365/1943

aria U0 . Como Prova. Para a matriz A , (8.111) diz-nos A = U0 (A ) A = U0 AA para alguma matriz unit que AA e auto-adjunta, segue que A = AA U0 . Identicando V = U0 , obtemos o que desejamos. O Teorema da Decomposi ca o Polar pode ser generalizado para abranger operadores limitados agindo em espa cos de Hilbert (vide Teorema 35.31, p agina 1822) e mesmo para abranger operadores n ao-limitados agindo em espa cos de Hilbert (vide [169]).

8.8.2

A Decomposi c ao em Valores Singulares

O Teorema da Decomposi ca o Polar, Teorema 8.22, p agina 363, tem um corol ario de particular interesse. Teorema 8.24 (Teorema da Decomposi c ao em Valores Singulares) Seja A Mat (C, n). Ent ao, existem matrizes unit arias V e W Mat (C, n) tais que A = V SW , (8.115) onde S Mat (C, n) e uma matriz diagonal cujos elementos diagonais s ao os valores singulares de A, ou seja, os autovalores de A A. Prova. A arma ca o segue imediatamente de (8.111) e de (8.110) tomando V = U P , W = P e S = D1/2 . O Teorema 8.24 pode ser generalizado para matrizes retangulares. No que segue, m, n N e usaremos as deni co es (8.3), (8.7) e a rela ca o (8.8) (vide p agina 296) que permitem mapear injetivamente matrizes retangulares em certas matrizes quadradas. Teorema 8.25 (Teorema da Decomposi c ao em Valores Singulares. Geral) Seja A Mat (C, m, n). Ent ao, existem matrizes unit arias V e W Mat (C, m + n) tais que A = Im, m+n V SW Jm+n, n , (8.116) onde S Mat (C, m + n) e uma matriz diagonal cujos elementos diagonais s ao os valores singulares de A (denida em (8.7)), ou seja, os autovalores de (A ) A . Prova. A matriz A Mat (C, m + n) e uma matriz quadrada e, pelo Teorema 8.24, possui uma decomposi ca o em valores singulares A = V SW com V e W Mat (C, m + n), unit arias, e S Mat (C, m + n) sendo uma matriz diagonal cujos elementos diagonais s ao os valores singulares de A . Com isso, (8.116) segue de (8.8). Na Se ca o 8.9, p agina 370, estudaremos uma aplica ca o do Teorema da Decomposi ca o em Valores Singulares, a saber, ao estudo da chamada Pseudo-Inversa de Moore-Penrose e suas aplica co es em problemas de optimiza ca o linear.

8.8.3

O Teorema da Triangulariza c ao de Schur

O teorema que apresentamos abaixo, devido a Schur19 , e semelhante, mas n ao id entico, ao Teorema de Jordan: toda matriz de Mat (C, n) pode ser levada por uma transforma ca o de similaridade induzida por uma matriz unit aria a uma matriz triangular superior (para a deni ca o, vide Se ca o 8.6, p agina 346). Esse teorema e alternativamente denominado Teorema da Triangulariza c ao de Schur ou Teorema da Decomposi c ao de Schur. Como veremos, esse teorema pode ser usado para fornecer uma outra demonstra ca o (eventualmente mais simples) da diagonalizabilidade de matrizes autoadjuntas e de matrizes normais por matrizes unit arias. Teorema 8.26 (Teorema da Decomposi c ao de Schur) Seja A Mat (C, n). Ent ao, existe U Mat (C, n), unit aria, e S Mat (C, n), triangular superior, tais que A = U SU . Os elementos da diagonal de S s ao os autovalores de A.
19 Issai

Schur (18751941).

JCABarata. Curso de F sica-Matem atica

Vers ao de 19 de junho de 2011.

Cap tulo 8

366/1943

Antes de provarmos esse teorema, mencionemos um corol ario evidente: Corol ario 8.6 Seja A Mat (C, n). Ent ao, existe V Mat (C, n), unit aria, e I Mat (C, n), triangular inferior, tais que A = V IV . Os elementos da diagonal de I s ao os autovalores de A. Prova do Corol ario 8.6. Pelo Teorema 8.26, a matriz A pode ser escrita da forma A = V SV , com V unit aria e S triangular superior. Logo, A = V S V . Por em, S I e triangular inferior.

Tamb em pelo Teorema 8.26, os autovalores de A s ao os elementos diagonais de S , que s ao o complexo conjugado dos elementos diagonais de S I . Mas os autovalores de A s ao o complexo conjugado dos autovalores de A (pela Proposi ca o 8.24, p agina 335) e, portanto, s ao os elementos diagonais de I .

Prova do Teorema 8.26. Comecemos observando que se A = U SU com U unit ario, ent ao A e S t em o mesmo polin omio caracter stico e, portanto, os mesmos autovalores, incluindo a multiplicidade (vide a discuss ao em torno de (8.30), p agina e triangular superior e, 306). Mas o polin omio caracter stico de S e pS (x) = det(x S ) = n k=1 (x Skk ), pois S portanto, os autovalores de S s ao os elementos de sua diagonal. Passemos ` a demonstra ca o da armativa principal, ou seja, que A = U SU com U unit ario e S triangular superior. u1 , . . . , un
(1)

Seja n 2 e v1 um autovetor de A com autovalor 1 e v1 = 1. Seja U (1) uma matriz unit aria da forma U (1) =
(1)

com u1 = v1 , ou seja, cuja primeira coluna e o vetor v1 . Ent ao,

(1)

AU

(1) (8.12)

(1) Au1 ,

...,

Au(1) n

(1) 1 u1 ,

(1) Au2 ,

...,

Au(1) n

para certos bk e akl , k, l = 1, . . . , n 1, onde


n1

(1)

(1)

1 0 (1) = U . . . 0

(1) b1

.. .

(1) bn1

a11 . . .

(1)

a1(n1) . . . a(n1)(n1)
(1)

(1)

a(n1)1

(1)

Auk

(1)

= bk u 1 +
l=1

(1) (1)

alk ul+1 ,

(1) (1)

k = 2, . . . , n .

(8.117)

Para simplicar a nota ca o, denimos b(1) . . = . , (1) bn1


(1) b1

n1

(n1 tendo n 1 linhas) e escrevemos a identidade (8.117) como 1 U (1) AU (1) =

0 . . = . , 0

A(1)

. . = . (1) a(n1)1
(1) T (1)

(1) a11

.. .

(1) a1(n1)

. . . a(n1)(n1)
(1)

n1

(8.118)

Para n = 2 isso demonstra o teorema, pois arma que 1 U (1) AU (1) = 0


(1) b1

(1) a11

JCABarata. Curso de F sica-Matem atica

Vers ao de 19 de junho de 2011.

Cap tulo 8

367/1943

sendo o lado direito uma matriz triangular superior. Para n > 2 procedemos por indu ca o. Supondo a arma ca o v alida para matrizes (n 1) (n 1), ent ao existe uma matriz unit aria V Mat (C, n 1) tal que V A(1) V = S (1) , sendo T 1 S (1) triangular superior. Assim, denindo a matriz unit aria U (2) Mat (C, n) por U (2) := 1 n , teremos por V
n 1

(8.118),

U (1) U (2) AU (1) U (2)

U (2) U (1) AU (1) U (2) 1

n1

T n1
V

(1) T (1)

n1

n1

T n1
V

V b

T (1) T

n1

V A(1) V
T (1) T

que e triangular superior, pois S (1) o e. Como U (1) U (2) e unit aria (pois U (1) e U (2) o s ao), o teorema est a provado. Coment ario.
Toda matriz triangular superior S pode ser escrita na forma D + N , sendo D a matriz diagonal formada pela diagonal de S (ou seja, Dii = Sii para todo i = 1, . . . , n) e N e nilpotente (pois e triangular superior, mas com diagonal nula). Assim, o Teorema 8.26 arma que toda matriz A pode ser levada ` a forma D + N por uma transforma ca o de similaridade unit aria. Por em, o Teorema 8.26 n ao garante (nem e verdade, em geral) que D e N comutem. Assim, o Teorema 8.26 e distinto do Teorema de Jordan, Teorema 8.20, p agina 355.

V b

n1

S (1)

O Teorema 8.26 tem por corol ario o seguinte teorema, j a provado anteriormente por outros meios (Teorema 8.13, p agina 338, e Proposi ca o 8.26, p agina 339). Teorema 8.27 Uma matriz A Mat (C, n) e auto-adjunta, se e somente se for diagonaliz avel por uma transforma c ao de similaridade unit aria e se seus autovalores forem reais. Prova. Pelo Teorema 8.26, existe uma matriz unit aria U tal que U AU = S , sendo S triangular superior cujos elementos diagonais s ao os autovalores de A. Assim, se A = A , segue que S = (U AU ) = U A U = U AU = S . Mas para uma matriz triangular superior S , a igualdade S = S implica que S e diagonal e os elementos da diagonal s ao reais. Reciprocamente, se A Mat (C, n) e diagonaliz avel por uma transforma ca o de similaridade unit aria e seus autovalores s ao reais, ou seja, existe U unit aria e D diagonal real com U AU = D, ent ao A = U DU e A = U D U . Como D e diagonal e real, vale D = D e, portanto, A = U DU = A, provando que A e auto-adjunta. Pelo Teorema 8.26, se A Mat (C, n) e uma matriz normal e U AU = S , com U unit aria e S triangular superior, ent ao S e normal (justique!). Assim, junto com o Lema 8.3, p agina 347, provamos o seguinte: Teorema 8.28 Uma matriz A Mat (C, n) e normal se e somente se for diagonaliz avel por uma transforma c ao de similaridade unit aria.

Essas arma co es foram demonstradas por outros meios no Teorema 8.15, p agina 340.

JCABarata. Curso de F sica-Matem atica

Vers ao de 19 de junho de 2011.

Cap tulo 8

368/1943

8.8.4

A Decomposi c ao QR e a Decomposi c ao de Iwasawa (KAN)

O prop osito desta se ca o e apresentar a chamada decomposi c ao de Iwasawa20 , ou decomposi c ao KAN 21 , de matrizes invers veis, Teorema 8.30. Esse teorema tem rela ca o com a teoria dos grupos de Lie, como discutiremos brevemente ao nal. Os dois primeiros resultados preparat orios abaixo, Proposi ca o 8.32 e Teorema 8.29 (Decomposi ca o QR), t em interesse por si s o. Proposi c ao 8.32 Seja R Mat (C, n) uma matriz triangular superior cujos elementos diagonais s ao n ao-nulos (i.e., R e invers vel). Ent ao, podemos escrever R = AN , onde A Mat (C, n) e a matriz diagonal formada com a diagonal de R: A = diag (R11 , . . . , Rnn ), e N Mat (C, n) e uma matriz triangular superior cujos elementos diagonais s ao iguais a 1. f Prova. E acil constatar que (abaixo m n 1) R11 0 . . R = . 0 0 R12 .. .. .. .. R22 .. . . . . .

Rmm 0

R1n R11 0 R2n . . . . . = . 0 Rmn Rnn 0

0 R22 .. .

.. .. .. . . .

..

Rmm 0

0 1 0 0 . . . . . . 0 0 Rnn 0

R12 R11

.. .. .. . . .

.. 1 0

1 .. .

R2n R22 . . . . Rmn Rmm 1

R1n R11

O estudante deve comparar as arma co es do teorema a seguir com o Teorema da Decomposi ca o Polar, Teorema 8.22, p agina 363, e com o Teorema da Decomposi ca o de Schur, Teorema 8.26, p agina 365. Teorema 8.29 (Teorema da Decomposi c ao QR) Seja M Mat (C, n) uma matriz invers vel. Ent ao, M pode ser escrita na forma M = QR, onde Q Mat (C, n) e unit aria e R Mat (C, n) e triangular superior, sendo que os elementos diagonais de R s ao estritamente positivos. e invers vel, os vetores mk , k = 1, . . . , n, s ao linearmente Prova do Teorema 8.29. Seja M = m1 , . . . , mn . Como M independentes, ou seja, formam uma base em Cn . Podemos, portanto, usar o procedimento de ortogonaliza ca o de GramSchmidt (vide Se ca o 3.3, p agina 192) e construir uma nova base ortonormal de vetores qj , j = 1, . . . , n, a partir dos vetores ml , l = 1, . . . , n. Tais vetores s ao denidos por
j 1

q1 =

m1 , m1

qj =

mj mj

ql , mj
l=1 j 1

ql , j = 2, . . . , n . ql

ql , mj
l=1

Como e f acil vericar, tem-se qi , qj m1 = q1 m1 ,


20 Kenkichi

= i j para todos i, j = 1, . . . , n. As rela co es acima implicam trivialmente


j 1 j 1

mj = qj mj

ql , mj
l=1

ql +
l=1

ql ql , mj

j = 2, . . . , n ,

Iwasawa (19171998). n ao h a uniformidade na literatura quanto ` a denomina ca o dessa decomposi ca o. Vamos cham a-la de decomposi ca o de Iwasawa pois a mesma e um caso particular (para o grupo GL(C, n) das matrizes complexas n n invers veis) de um teorema mais geral da teoria dos grupos de Lie, denominado Teorema da Decomposi ca o de Iwasawa, que arma que todo elemento g de um grupo de Lie semi-simples pode ser escrito como produto de um elemento k de um subgrupo compacto maximal, por um elemento a de um subgrupo Abeliano (real) e por um elemento n de um subgrupo nilpotente (ou seja, cuja algebra de Lie e nilpotente): g = kan. Em Alem ao, as palavras compacto, Abeliano e nilpotente s ao Kompakt, Abelsch e Nilpotent, da a denomina ca o decomposi ca o KAN para essa decomposi ca o, denomina ca o essa encontrada em alguns textos.
21 Infelizmente

JCABarata. Curso de F sica-Matem atica

Vers ao de 19 de junho de 2011.

Cap tulo 8

369/1943

rela co es estas que podem ser escritas em forma matricial como R11 0 . . R , onde R := . 0 0 Rjj = mj

q1 , m2

..

R22

..

m1 , . . . , mn

q1 , . . . , qn

..

..

..

R(n1)(n1)

ql , mj ql

com R11 =

q2 , mn C . , (8.119) . . qn1 , mn C Rnn


C

q1 , mn

j 1

m1 ,

j = 2, . . . , n .

l=1

ca-se da validade da rela c ao (8.119). E. 8.34 Exerc cio. Conven Denindo Q := q1 , . . . , qn , a rela ca o (8.119) diz-nos que M = QR, sendo R triangular superior (como se v e) e Q unit aria (pois os vetores ql , l = 1, . . . , n, s ao ortonormais). Isso completa a prova do Teorema 8.29. Chegamos assim ao importante Teorema da Decomposi ca o de Iwasawa para matrizes invers veis: Teorema 8.30 (Teorema da Decomposi c ao de Iwasawa, ou Decomposi c ao KAN ) Seja M Mat (C, n) uma e uma matriz matriz invers vel. Ent ao, M pode ser escrita de modo u nico na forma M = KAN , onde K Mat (C, n) unit aria, A Mat (C, n) e a uma matriz diagonal, tendo elementos diagonais estritamente positivos, e N Mat (C, n) e uma matriz triangular superior cujos elementos diagonais s ao iguais a 1. Prova. A arma ca o que M pode ser escrita na forma M = KAN , com K , A e N com as propriedades acima segue imediatamente da Proposi ca o 8.32 e do Teorema 8.29, dispensando demonstra ca o. O u nico ponto a se demonstrar ea unicidade dessa decomposi ca o. Vamos ent ao supor que para algum M Mat (C, n) existam K, K0 Mat (C, n), matrizes unit arias, A, A0 Mat (C, n), matrizes diagonais, tendo elementos diagonais estritamente positivos, e N, N0 Mat (C, n) matrizes triangulares superiores cujos elementos diagonais s ao iguais a 1, tais que M = KAN = K0 A0 N0 .
1 Segue imediatamente disso que K0 K = A0 N0 N 1 A1 . O lado esquerdo dessa igualdade e uma matriz unit aria e, portanto, normal. O lado direito e uma matriz triangular superior (pela Proposi ca o 8.29, p agina 346). Pelo Lema 1 8.3, p agina 347, A0 N0 N 1 A1 deve ser uma matriz diagonal D. Assim, temos que K0 K = D e A0 N0 N 1 A1 = D. 1 1 A primeira dessas rela co es diz-nos que D e unit aria. A segunda diz-nos que N0 N = A 0 DA, ou seja, N0 = D0 N , 1 onde D0 := A0 DA e diagonal (por ser o produto de tr es matrizes diagonais). Agora, N e N0 s ao matrizes triangulares superiores cujos elementos diagonais s ao iguais a 1. Portanto, a rela ca o N0 = D0 N com D0 diagonal s o e poss vel se D0 = (de outra forma haveria elementos na diagonal de N ou de N0 diferentes de 1), estabelecendo que N = N0 . 1 1 . Agora, A e A0 s ao diagonais, tendo na diagonal n umeros Provamos, assim, que A 0 DA = , ou seja, D = A0 A reais positivos. Logo, D tamb em e diagonal e tem na diagonal n umeros reais positivos e, portanto, D = D . Como D e unit aria (como observado linhas acima), segue que D2 = . Logo, os elementos Dkk da diagonal de D satisfazem Dkk = 1, para todo k = 1, . . . , n (os sinais podendo ser distintos para k s distintos). Agora, como A0 = DA e

JCABarata. Curso de F sica-Matem atica

Vers ao de 19 de junho de 2011.

Cap tulo 8

370/1943

como A e A0 t em na diagonal n umeros reais positivos, n ao podemos ter Dkk = 1 para algum k e, portanto, D = . Conseq uentemente, K = K0 e A = A0 , estabelecendo a unicidade desejada. Note o leitor que o conjunto das matrizes unit arias de Mat (C, n) forma um subgrupo de GL(C, n) (o grupo das matrizes complexas n n invers veis). O conjunto das matrizes diagonais de Mat (C, n) tendo elementos diagonais estritamente positivos e igualmente um subgrupo de GL(C, n). Por m, o conjunto das matrizes triangulares superiores de Mat (C, n) cujos elementos diagonais s ao iguais a 1 e tamb em um subgrupo de GL(C, n). Assim, o Teorema 8.30 arma que cada elemento de GL(C, n) pode ser escrito de modo u nico como produto de elementos de cada um desses tr es subgrupos. Esse e um caso particular de um teorema da teoria dos grupos de Lie conhecido como Teorema da Decomposi c ao de Iwasawa.

8.9

A Pseudo-Inversa de Moore-Penrose. Optimiza c ao Linear

Na presente se ca o introduziremos uma generaliza ca o especial da no ca o de inversa de matrizes, a qual aplica-se mesmo a matrizes n ao-quadradas. O conceito que descreveremos, a chamada pseudo-inversa de Moore-Penrose, e particularmente u til no tratamento de problemas de optimiza ca o linear, como discutiremos adiante (Se ca o 8.9.2, p agina 378), ou seja, em problemas onde procura-se solu co es optimalmente aproximadas de sistemas de equa co es lineares como Ax = y , onde A e uma matriz m n dada, y um vetor-coluna, dado, com m componentes e x, a inc ognita do problema, e um um vetor-coluna com n componentes. Em tais problemas procura-se vetores x tais que a norma de Ax y seja a menor poss vel e que representem, portanto, n ao necessariamente a solu ca o exata do sistema Ax = y (que pode n ao existir), mas a melhor aproxima ca o em termos de m nimos quadrados ao que seria a solu ca o. Inversas generalizadas, ou pseudo-inversas

Sejam m, n N e seja uma matriz (n ao necessariamente quadrada) A Mat (C, m , n). Uma matriz B Mat (C, n, m) e dita ser uma inversa generalizada, ou pseudo-inversa, de A, se satiszer as seguintes condi co es: 1. ABA = A, 2. BAB = B .

O leitor h a de notar que se A Mat (C, n) e uma matriz quadrada invers vel, sua inversa A1 satisfaz trivialmente as propriedades denidoras da inversa generalizada. Provaremos mais adiante que toda matriz A Mat (C, m , n) possui ao menos uma inversa generalizada, a saber, a pseudo-inversa de Moore-Penrose. Com a generalidade da deni ca o acima, por em, n ao se pode garantir a unicidade da inversa generalizada de A. Com a amplitude da deni ca o acima, a no ca o inversa generalizada n ao e muito u til, mas certos tipos mais espec cos de inversas generalizadas s ao de interesse em certos tipos de problemas. No que segue discutiremos a chamada pseudoinversa de Moore-Penrose e seu emprego em problemas de optimiza ca o linear. Deni c ao da pseudo-inversa de Moore-Penrose de uma matriz

Sejam m, n N e seja uma matriz (n ao necessariamente quadrada) A Mat (C, m , n). Uma matriz A+ Mat (C, n, m) e dita ser uma pseudo-inversa de Moore-Penrose de A se satiszer as seguintes condi co es: 1. AA+ A = A, 2. A+ AA+ = A+ , 3. AA+ Mat (C, m) e A+ A Mat (C, n) s ao auto-adjuntas. O leitor h a de notar que se A Mat (C, n) e uma matriz quadrada invers vel, sua inversa A1 satisfaz trivialmente as propriedades denidoras da pseudo-inversa de Moore-Penrose.

JCABarata. Curso de F sica-Matem atica

Vers ao de 19 de junho de 2011.

Cap tulo 8

371/1943

A no ca o de pseudo-inversa descrita acima foi introduzida por E. H. Moore22 em 1920 e redescoberta por R. Penrose23 em 1955. O conceito de pseudo-inversa de Moore-Penrose eu til para a resolu ca o de problemas de optimiza ca o lineares, ou seja, ` a determina ca o da melhor aproxima ca o em termos de m nimos quadrados ` a solu ca o de sistemas lineares. Trataremos desses aspectos mais adiante (vide Teorema 8.33, p agina 379), ap os demonstrarmos resultados sobre exist encia e unicidade. Outros desenvolvimentos da teoria das pseudo-inversas de Moore-Penrose e suas aplica co es, podem ser encontrados em [20]. Vide tamb em as refer encias originais: E. H. Moore, On the reciprocal of the general algebraic matrix. Bulletin of the American Mathematical Society 26, 394395 (1920); R. Penrose, A generalized inverse for matrices, Proceedings of the Cambridge Philosophical Society 51, 406413 (1955) e R. Penrose, On best approximate solution of linear matrix equations, Proceedings of the Cambridge Philosophical Society 52, 1719 (1956). Nas p aginas que seguem demonstraremos que toda a matriz A Mat (C, m, n) possui uma pseudo-inversa de MoorePenrose, a qual eu nica. Come camos com a quest ao da unicidade para em seguida tratarmos de propriedades gerais e, posteriormente, da quest ao da exist encia. As aplica co es em problemas de optimiza ca o s ao discutidas na Se ca o 8.9.2, p agina 378. A unicidade da pseudo-inversa de Moore-Penrose

Seja A+ Mat (C, n, m) uma pseudo-inversa de Moore-Penrose de A Mat (C, m, n) e seja B Mat (C, n, m) uma outra pseudo-inversa de Moore-Penrose de A, ou seja, tal que ABA = A, BAB = B com AB e BA auto-adjuntas. oteses, M1 e auto-adjunta (por ser a diferen ca de duas Seja M1 := AB AA+ = A(B A+ ) Mat (C, m). Pelas hip matrizes auto-adjuntas) e (M1 )2 = (AB AA+ )A(B A+ ) = (ABA AA+ A)(B A+ ) = (A A)(B A+ ) = 0. Como M1 e auto-adjunta, o fato que (M1 )2 = 0 implica M1 = 0, pois para todo x Cm tem-se M1 x 2 C = M1 x, M1 x C = x, (M1 )2 x C = 0, o que signica que M1 = 0. Isso provou que AB = AA+ . Analogamente, prova-se que BA = A+ A (para tal, considere-se a matriz auto-adjunta M2 := BA A+ A Mat (C, n) e proceda-se como acima). Agora, tudo isso implica que A+ = A+ AA+ = A+ (AA+ ) = A+ AB = (A+ A)B = BAB = B , provando a unicidade. Como j a comentamos, se A Mat (C, n) e uma matriz quadrada invers vel, sua inversa A1 satisfaz trivialmente as propriedades denidoras da pseudo-inversa de Moore-Penrose e, portanto, tem-se nesse caso A+ = A1 , univocamente. tamb E em evidente pela deni ca o que para mn , a matriz m n identicamente nula, vale (mn )+ = nm . Apresentaremos no que seguir a duas demonstra co es da exist encia da pseudo-inversa de Moore-Penrose de matrizes arbitr arias de Mat (C, m, n). Ambas as demonstra co es permitem produzir algoritmos para a determina ca o expl cita da pseudo-inversa de Moore-Penrose. Uma primeira demonstra ca o ser a apresentada na Se ca o 8.9.1.1, p agina 375, (vide o Teorema 8.31, p agina 376, e o Teorema 8.32, p agina 377) e decorrer a de diversos resultados que estabeleceremos a seguir. Destacamos particularmente as express oes (8.141) e (8.142), as quais permitem calcular a pseudo-inversa de Moore-Penrose A+ de uma matriz A Mat (C, m, n) diretamente em termos de A, A e dos autovalores de AA ou de A A (ou seja, dos valores singulares de A). Uma segunda demonstra ca o ser a apresentada na Se ca o 8.9.3, p agina 379, e para a mesma faremos uso da decomposi ca o em valores singulares apresentada no Teorema 8.24, p agina 365. A essa segunda demonstra ca o da Se ca o 8.9.3 o leitor interessado poder a passar sem perdas neste ponto. Os resultados da Se ca o 8.9.3, por em, n ao ser ao usados no que segue. Essa segunda demonstra ca o e a mais freq uentemente apresentada na literatura, mas cremos que as express oes (8.141) e (8.142) fornecem um m etodo algoritmicamente mais simples para a determina ca o da pseudo-inversa de Moore-Penrose de uma matriz geral. Calculando a pseudo-inversa de Moore-Penrose em casos particulares A exist encia da pseudo-inversa de Moore-Penrose

Demonstremos a unicidade da pseudo-inversa de Moore-Penrose de uma matriz A Mat (C, m, n), caso exista.

Se A Mat (C, m, n), ent ao A Mat (C, n, m) e denida como a matriz cujos elementos (A )ij s ao dados por Aji para todos 0 i n e 0 j m. Futuramente obteremos as express oes (8.141) e (8.142), as quais permitem calcular a pseudo-inversa de Moore-Penrose A+ Mat (C, n, m) de uma matriz A Mat (C, m, n) diretamente em termos de A, A e dos autovalores de AA ou de A A. Nos exerc cios que seguem indicaremos situa co es especiais mas u teis nas quais a pseudo-inversa de Moore-Penrose pode ser calculada de modo relativamente simples.
22 Eliakim 23 Sir

Hastings Moore (18621932). Roger Penrose (1931).

JCABarata. Curso de F sica-Matem atica

Vers ao de 19 de junho de 2011.

Cap tulo 8

372/1943

a1

E. 8.35 Exerc cio. Constate que se A Mat (C, m, 1), A =


1 A
2 C

am

. . .

, um vetor-coluna n ao-nulo, ent ao A+ =

1 A

2 C

A =

( a1 , ..., am ), onde A

| a 1 |2 + + | a m |2 .

Observe-se que se z C, podemos considerar z como uma matriz complexa 1 1, ou seja, como elemento de 0, z = 0 + Mat (C, 1, 1) e, com isso, obtemos do exposto acima (z ) = . 1, z = 0 z O resultado do Exerc cio E. 8.35 pode ser generalizado. E. 8.36 Exerc cio. Seja A Mat (C, m, n). Mostre que se (AA )1 existe, ent ao A+ = A (AA )1 . Mostre que se 1 + 1 (A A) existe, ent ao A = (A A) A . Sugest ao: em ambos os casos, verique que o lado direito satisfaz as propriedades denidoras da pseudo-inversa de Moore-Penrose e use a unicidade. Os resultados do Exerc cio E. 8.36 podem ser generalizados para situa co es em que AA ou A A n ao s ao invers veis + + pois, como veremos na Proposi ca o 8.34, p agina 373 valem sempre as rela co es A+ = A AA = A A A . Tamb em o Teorema 8.31, p agina 376, apresentar a uma generaliza ca o dos resultados do Exerc cio E. 8.36, mostrando uma outra forma de proceder quando AA ou A A n ao forem invers veis. Os exerc cios que seguem cont em aplica co es dos resultados do Exerc cio E. 8.36.
0 i E. 8.37 Exerc cio. Seja A = ( 2 0 i 1 ), com A = 2 0 0 i i 1

. Mostre que AA possui inversa, mas que A A n ao possui.


1 9 4 2i 1 5i i 4

Usando o Exerc cio E. 8.36, calcule a pseudo-inversa de Moore-Penrose A+ de A, obtendo A+ = essa A satisfaz de fato as propriedades denidoras da pseudo-inversa de Moore-Penrose. E. 8.38 Exerc cio. Seja A =
12 0 i 03 +

. Verique que

, com A =

1 0 0 2 i 3

. Mostre que AA n ao possui inversa, mas que A A possui.


1 10 10 2i 6 0 i 3

Usando o Exerc cio E. 8.36, calcule a pseudo-inversa de Moore-Penrose A+ de A, obtendo A+ = + que essa A satisfaz de fato as propriedades denidoras da pseudo-inversa de Moore-Penrose.

. Verique

8.9.1

Outras Propriedades da Pseudo-Inversa de Moore-Penrose

As seguintes propriedades da pseudo-inversa de Moore-Penrose seguem das deni co es e da unicidade. Suas demonstra co es s ao elementares e s ao deixadas como exerc cio: para A Mat (C, m, n) valem 1. (A+ )+ = A, 2. (A+ ) = AT
T +

, A+ = A

e, conseq uentemente, (A+ ) = (A )+ ,

3. (zA)+ = z 1 A+ para todo z C n ao-nulo. de se observar, por E em, que se A Mat (C, m, n) e B Mat (C, n, p), nem sempre (AB )+ e dada por B + A+ , ao contr ario do que ocorre com a inversa usual (para o caso m = n = p). Uma exce ca o relevante ser a encontrada na Proposi ca o 8.34, p agina 373. A seguinte proposi ca o lista mais algumas propriedades importantes, algumas das quais usaremos logo adiante:

JCABarata. Curso de F sica-Matem atica

Vers ao de 19 de junho de 2011.

Cap tulo 8

373/1943

Proposi c ao 8.33 A pseudo-inversa de Moore-Penrose satisfaz as seguintes rela c oes A+ = A+ (A+ ) A , A A (A+ ) , A A A+ , A (A+ ) A+ , (A+ ) A A , A+ A A , (8.120) (8.121) (8.122) (8.123) (8.124) (8.125)

A = A A+ = =

A = A v alidas para toda A Mat (C, m, n). =

Das rela co es acima, a mais relevante talvez seja a rela ca o (8.122), pois faremos uso importante dela na demonstra ca o da Proposi ca o 8.33, p agina 379, que trata da aplica ca o da pseudo-inversa de Moore-Penrose a problemas de optimiza ca o linear. Prova da Proposi c ao 8.33. Por AA+ ser auto-adjunta, vale AA+ = (AA+ ) = (A+ ) A . Multiplicando-se ` a esquerda por + + A obtemos A = A+ (A+ ) A , provando (8.120). Substituindo-se A A+ e usando o fato que A = (A+ )+ , obt em-se de (8.120) que A = AA (A+ ) , que e a rela ca o (8.121). Substituindo-se A A e usando o fato que (A )+ = (A+ ) , obt em-se de (8.121) que A = A AA+ que e a rela ca o (8.122). As rela co es (8.123)(8.125) podem ser obtidas analogamente a partir do fato de A+ A ser tamb em auto-adjunta, mas e mais f acil obt e-las substituindo-se A A em (8.120)(8.122) e tomando-se o adjunto das express oes resultantes. Da Proposi ca o 8.33 podem ser obtidos v arios resultados de interesse, alguns dos quais encontram-se reunidos na proposi ca o que segue. Proposi c ao 8.34 Para a pseudo-inversa de Moore-Penrose vale AA para todo A Mat (C, m, n). Disso obt em-se que A+ = A AA tamb em para todo A Mat (C, m, n). A express ao (8.127) generaliza os resultados do Exerc cio E. 8.36, p agina 372 e pode ser empregada para calcular A+ + + desde que AA ou A A sejam previamente conhecidas. Prova da Proposi c ao 8.34. Seja B = A AA
(8.121) + + +

A+

(8.126)

A A

A ,

(8.127)

A+ . Tem-se
(8.125)

A A (A+ ) A

A A (A+ ) A+ A A = (AA )B (AA ) ,

onde usamos tamb em que A B = A


+

= A+ . Tem-se tamb em que = (A+ ) A+ A A+


(8.123)

A+

(8.120)

(A+ ) A+ A A (A+ ) A+ = B A A B .
(8.122)

Observe-se tamb em que A A B = A A (A+ ) A+ = AA+

JCABarata. Curso de F sica-Matem atica

Vers ao de 19 de junho de 2011.

Cap tulo 8

374/1943

que e auto-adjunto, por deni ca o. Analogamente, B A A = (A+ ) A+ A A


(8.124)

(A )+ A

que tamb em e auto-adjunto, por deni ca o. Os fatos expostos nas linhas acima provaram que B e a pseudo-inversa de Moore-Penrose de AA , provando (8.126). Substituindo-se A A em (8.126) obt em-se tamb em A A Observe-se agora que A AA e que A A provando (8.127). A pseudo-inversa de Moore-Penrose, o n ucleo e a imagem de uma matriz
+ + (8.126) +

= A+ A A A A+ A
+

.
(8.123)

(8.128) A+ A+ ,

A+

(8.128)

(8.120)

Denimos o n ucleo e a imagem (range) de uma matriz A Mat (C, m, n) por Ker (A) := {u Cn | Au = 0} e evidente que Ker (A) Ran (A) := {Au, u Cn }, respectivamente. E e um subespa co linear de Cn e que Ran (A) e um m subespa co linear de C . A seguinte proposi ca o ser a usada logo adiante, mas e de interesse por si s o. Proposi c ao 8.35 Seja A Mat (C, m, n) e sejam denidos P1 := n A+ A Mat (C, n) e P2 := m AA+ Mat (C, n). Ent ao, valem as seguintes arma c oes:
1. P1 e P2 s ao projetores ortogonais, ou seja, satisfazem (Pk )2 = Pk e Pk = Pk , k = 1, 2.

2. Ker (A) = Ran (P1 ), Ran (A) = Ker (P2 ), Ker (A+ ) = Ran (P2 ) e Ran (A+ ) = Ker (P1 ). 3. Ran (A) = Ker (A+ ) e Ran (A+ ) = Ker (A) . 4. Ker (A) Ran (A+ ) = Cn e Ker (A+ ) Ran (A) = Cm , ambas somas diretas de subespa cos ortogonais. Prova. Que P1 e P2 s ao auto-adjuntos segue do fato de AA+ e A+ A o serem. Tem-se tamb em que (P1 )2 = 2A+ A + A+ AA+ A = 2A+ A + A+ A = A+ A = P1 e analogamente para P2 . Isso provou o item 1.

Seja x Ker (A). Como Ran (P1 ) e um subespa co linear fechado de Cn , o Teorema do Melhor Aproximante e o Teorema da Decomposi ca o Ortogonal (que neste texto s ao apresentados com toda generalidade no contexto de espa cos de Hilbert, como Cm na forma do Teorema 34.1, p agina 1684, e do Teorema 34.2, p agina 1686, respectivamente) e m nimo. Al em disso, x z0 e ortogonal a garantem-nos a exist encia de um u nico z0 Ran (P1 ) tal que x z0 Cm Ran (P1 ). Assim, existe ao menos um y0 Cm tal que x P1 y0 e ortogonal a todo elemento da forma P1 y , ou seja, x P1 y0 , P1 y C = 0 para todo y Cm , o que implica P1 (x P1 y0 ), y C = 0 para todo y Cm , o que por sua vez implica P1 (x P1 y0 ) = 0. Isso, por em, arma que P1 x = P1 y0 . Como x Ker (A) vale P1 x = x (pela deni ca o de P1 ). Provamos portanto que se x Ker (A) ent ao x Ran (P1 ), estabelecendo que Ker (A) Ran (P1 ). Por outro lado, o fato que AP1 = A( A+ A) = A A = 0 implica que Ran (P1 ) Ker (A), provando que Ran (P1 ) = Ker (A). Se z Ker (P1 ), ent ao z = A+ Az , provando que z Ran (A+ ). Isso provou que Ker (P1 ) Ran (A+ ). Por outro lado, se u Ran (A+ ) ent ao existe v Cm tal que u = A+ v . Logo, P1 u = (n A+ A)A+ v = (A+ A+ AA+ )v = 0, provando que u Ker (P1 ) e que Ran (A+ ) Ker (P1 ). Isso estabeleceu que Ker (P1 ) = Ran (A+ ). P2 e obtida de P1 com a substitui ca o A A+ (lembrando-se que (A+ )+ = A). Logo, os resultados de acima implicam + que Ran (P2 ) = Ker (A ) e que Ker (P2 ) = Ran (A). Isso provou o item 2.

Se M Mat (C, p) (com p N, arbitr ario) e auto-adjunta, ent ao y, M x C = M y, x C para todos x, y Cp . Essa rela ca o torna evidente que Ker (M ) = Ran (M ) (justique!). Com isso o item 3 segue do item 2 tomando-se M = P1 e M = P2 . O item 4 e evidente pelo item 3.

JCABarata. Curso de F sica-Matem atica

Vers ao de 19 de junho de 2011.

Cap tulo 8

375/1943

cio E. 8.37, p agina 372, e para o exemplo do Exerc cio E. E. 8.39 Exerc cio. Calcule P1 e P2 para o exemplo do Exerc 8.38, p agina 372.

8.9.1.1

A Regulariza c ao de Tikhonov. Exist encia

No Exerc cio E. 8.36, p agina 372, vimos que se (AA )1 existe, ent ao A+ = A (AA )1 e que se (A A)1 existe, ent ao + 1 A = (A A) A . No caso de essas inversas n ao existirem h a um procedimento alternativo que tamb em permite obter A+ . Sabemos da Proposi ca o 8.6, p agina 307, que mesmo se (AA )1 n ao existir, a matriz AA + ser a invert vel para todo C n ao-nulo com || pequeno o suciente. Isso permite conjecturar que as express oes A (AA + )1 e (A A + )1 A , que est ao bem denidas para = 0 com || pequeno, convergem a A+ quando tomamos o limite 0. Como veremos no que segue, essa conjectura e correta.

Pelo dito acima, podemos substituir as matrizes AA ou A A, caso sejam singulares, pelas matrizes invers veis AA + ca o (que envolve a substitui ca o provis oria ou A A + com = 0 com || pequeno. Esse procedimento de regulariza de uma express ao singular por outra regular) e denominado regulariza c ao de Tikhonov24 , em honra ao matem atico que desenvolveu essas id eias no contexto de equa co es integrais25.

Nosso primeiro resultado consiste em provar que os limites descritos acima de fato existem e s ao iguais, o que ser a feito nos dois lemas que seguem. veis (i.e., Lema 8.6 Seja A Mat (C, m, n) e seja C tal que AA + m e A A + n sejam invers (AA ) (A A), um conjunto nito). Ent ao, A (AA + m )1 = (A A + n )1 A . Prova. Sejam B := A (AA + m )1 e C := (A A + n )1 A . Temos que Logo, (A A + n )B = A , o que implica B = (A A + n )1 A = C . A AB = A AA (AA +m )1 = A AA +m m (AA +m )1 = A m (AA +m )1 = A B .

Lema 8.7 Para toda A Mat (C, m, n) os limites lim A (AA + m )1 e lim (A A + n )1 A existem e s ao iguais
0 0

(pelo Lema 8.6), denindo um elemento de Mat (C, n, m).

Prova do Lema 8.7. Notemos primeiramente que A e uma matriz identicamente nula se e somente se AA ou A A o forem. De fato, se, por exemplo, A A = 0, valer a para todo vetor x que 0 = x, A Ax C = Ax, Ax C = Ax 2 , provando que A = 0. Como a arma ca o a ser provada e evidente se A for nula, suporemos no que segue que AA e A A n ao s ao nulas. A matriz AA Mat (C, m) e, evidentemente, auto-adjunta. Sejam 1 , . . . , r seus autovalores distintos. Pelo Teorema Espectral para operadores auto-adjuntos (vide Teorema 8.5, p agina 322 e Teorema 8.13, p agina 338) podemos escrever
r

AA =
a=1

a Ea ,
r a=1

(8.129) Ea = m . Logo,

onde Ea s ao os projetores espectrais de AA e satisfazem Ea Eb = ab Ea , Ea = Ea e r

AA + m =
a=1

(a + )Ea

e, portanto, para {1 , . . . , r }, vale pela Proposi ca o 8.17, p agina 325, AA + m


1

1 Ea + a=1 a

A AA + m

1 A Ea . + a a=1

24 Andrei Nikolaevich Tikhonov (19061993). O sobrenome russo Tikhonov e por vezes transliterado como Tykhonov, Tichonov ou ainda Tychono. 25 Para uma refer encia geral, vide [210]. Para os trabalhos originais, vide: Tikhonov, A. N., 1943, On the stability of inverse problems, Dokl. Akad. Nauk. USSR, 39, No. 5, 195198 (1943); Tikhonov, A. N., Solution of incorrectly formulated problems and the regularization method, Soviet Math. Dokl. 4, 10351038 (1963), tradu ca o para o ingl es de Dokl. Akad. Nauk. USSR 151, 501504 (1963).

JCABarata. Curso de F sica-Matem atica

Vers ao de 19 de junho de 2011.

Cap tulo 8

376/1943

H a dois casos a se considerar 1. AA n ao tem auto-valor nulo e 2. AA tem auto-valor nulo. No caso em que AA n ao tem auto-valor nulo e claro pela u ltima express ao que o limite lim A (AA + m )1 existe
0

e vale
0

lim A (AA + m )1 =

1 A Ea . a=1 a

(8.130)

No caso em que AA tem auto-valor nulo, digamos, 1 = 0, o projetor E1 projeta sobre o n ucleo de AA : Ker (AA ) := n {u C | AA u = 0}. Se x Ker (AA ), ent ao A x = 0, pois 0 = x, AA x C = A x, A x C = A x . Portanto, A E1 = 0 e, assim, podemos escrever, A (AA + m )1 = donde obt em-se
0

(8.131)

1 A Ea , + a a=2 1 A Ea . a=2 a
r

lim A (AA + m )1 =

(8.132)

Isso provou que lim A (AA + m )1 sempre existe.


0

em existe e coincide com lim A (AA + m )1 . Pelo Lema 8.6, p agina 375, o limite lim (A A + n )1 A tamb
0 0

A principal conseq u encia e o seguinte resultado: Teorema 8.31 (Regulariza c ao de Tikhonov) Para toda A Mat (C, m, n) valem A+ = lim A AA + m
0 1

(8.133)

A+ = lim A A + n
0

A .

(8.134)

Como a exist encia dos limites acima foi estabelecida para matrizes arbitr arias no Lema 8.7, p agina 375, o Teorema 8.31 cont em uma prova geral de exist encia da pseudo-inversa de Moore-Penrose. Prova do Teorema 8.31. As arma co es a serem provadas s ao evidentes caso A = mn pois, como j a vimos (mn )+ = nm . Assim, assumiremos no que segue que A e n ao nula, o que equivale, pelo exposto no in cio da prova do Lema 8.7, a supor que AA e A A n ao s ao nulas. Pelos Lemas 8.6 e 8.7 e suciente demonstrar (8.133). H a dois casos a se considerar 1. AA n ao tem auto-valor nulo e 2. AA tem auto-valor nulo. No caso 1., vimos em (8.130), na prova do Lema 8.7 (e com a nota ca o l a estabelecida), que r 1 1 = A Ea =: B . lim A AA + m 0 a=1 a Note-se agora que AB = 1 AA Ea = a a=1
r

1 a=1 a

b Eb
b=1

Ea =
a=1 b=1

1 b ab Ea = a

Ea = m ,
a=1

(8.135)

que e auto-adjunta, e que BA =

1 A Ea A , a=1 a

(8.136)

JCABarata. Curso de F sica-Matem atica

Vers ao de 19 de junho de 2011.

Cap tulo 8

377/1943

que e tamb em auto-adjunta, pois a R para todo a (por serem autovalores de uma matriz auto-adjunta) e pelo fato de (A Ea A) = A Ea A para todo a, j a que Ea = Ea . De (8.135) segue que ABA = A. De (8.136) segue que BAB = 1 A Ea A a=1 a
r r r r

b=1

1 A Eb b

=
a=1 b=1

1 A Ea (AA )Eb . a b

Agora, pela decomposi ca o espectral (8.129) de AA , segue que (AA )Eb = b Eb . Logo,
r r

BAB =
a=1 b=1

1 A Ea Eb = a

1 A Ea a=1 a

Eb
b=1

= B.

Isso provou que A = A+ no caso em que AA n ao tem autovalor nulo. Vamos agora supor que AA n ao autovalor nulo, a saber, 1 . Vimos em (8.132), na prova do Lema 8.7, que lim A AA + m
1 r

1 A Ea =: B . a=2 a

Usando o fato que (AA )Ea = a Ea , o qual segue da decomposi ca o espectral (8.129) de AA , obt em-se AB = 1 AA Ea = a a=2
r

1 a Ea = a=2 a
r

r a=2

Ea = m E1 ,

(8.137)

que e auto-adjunta, pois E1 o e. Tem-se tamb em BA = 1 A Ea A , a=2 a (8.138)

que e tamb em auto-adjunta, pelos argumentos j a expostos. De (8.137) segue que ABA = A E1 A. Note-se agora que (E1 A) = A E1 = 0, por (8.131). Isso demonstrou que E1 A = 0 e que ABA = A. De (8.138) segue que BAB = 1 A Ea A a=2 a
r r b=2

1 A Eb b

=
a=2 b=2

1 A Ea (AA )Eb . a b

Usando novamente que (AA )Eb = b Eb , obtemos


r r

BAB =
a=2 b=2

1 A Ea Eb = a

1 A Ea a=2 a

Eb
b=2

= B

1 A Ea E1 = B , a=2 a

m E1

pois Ea E1 = 0 para a = 1. Isso demonstrou que BAB = B . Assim, estabelecemos que A = A+ tamb em no caso em que AA tem autovalor nulo, completando a prova de (8.133).

8.9.1.2

A Pseudo-Inversa de Moore-Penrose e o Teorema Espectral


r

Durante a demonstra ca o do Teorema 8.31 estabelecemos tamb em o seguinte resultado de interesse: c ao espectral de AA , onde Teorema 8.32 Seja A Mat (C, m, n) n ao-nula e seja AA = a=1 a Ea a representa {1 , . . . , r } R e o conjunto dos autovalores distintos de AA e Ea s ao os correspondentes projetores espectrais auto-adjuntos. Ent ao, vale r 1 A+ = A Ea . (8.139) a a=1
a =0

JCABarata. Curso de F sica-Matem atica

Vers ao de 19 de junho de 2011.

Cap tulo 8

378/1943

Analogamente, seja A A = b=1 b Fb a representa c ao espectral de A A, onde {1 , . . . , s } R e o conjunto dos autovalores distintos de A A e Fb os correspondentes projetores espectrais auto-adjuntos. Ent ao, vale tamb em
s

=
b=1 b =0

1 Fb A . b

(8.140)

(Vale mencionar aqui que, pelo Exerc cio E. 8.6, p agina 308, o conjunto de autovalores n ao-nulos de AA coincide com o conjunto de autovalores n ao-nulos de A A: {1 , . . . , r } \ {0} = {1 , . . . , s } \ {0}). De (8.139) e (8.140) segue que para A n ao-nula valem
r

A+

=
a=1 a =0

1
r

a
l=1 l= a

(a l )

A
s

l=1 l= a

AA l m ,

(8.141)

A+

=
b=1 b =0

1
r

b
l=1 l= b

(b l )

l=1 l= b

A A l n A .

(8.142)

As express oes (8.141) e (8.142) fornecem mais um algoritmo geral para o c omputo da pseudo-inversa de MoorePenrose, o qual pode ser de implementa ca o simples, pois requer apenas a determina ca o dos autovalores de AA ou de A A. Prova do Teorema 8.32. A igualdade (8.139) foi provada durante a demonstra ca o do Teorema 8.31 (vide (8.130) e (8.132)). A rela ca o (8.140) pode ser provada analogamente, mas segue mais facilmente do truque j a mencionado de usar (8.139), trocando A A e tomando-se o adjunto da express ao obtida. As rela co es (8.141) e (8.142) seguem da Proposi ca o 8.18, p agina 325, particularmente de (8.50). E. 8.40 Exerc cio. Usando (8.141) ou (8.142) reobtenha as matrizes A+ dos Exerc cios E. 8.35, E. 8.37 e E. 8.38.

8.9.2

A Pseudo-Inversa de Moore-Penrose e Problemas de Optimiza c ao Linear

Tratemos agora de uma das principais aplica co es da no ca o de pseudo-inversa de Moore-Penrose, a saber, no tratamento de problemas de optimiza ca o linear, que motivaremos e deniremos a seguir. Sejam A Mat (C, m, n) e y Cm dados e considere-se o problema de determinar x Cn que satisfa ca a equa ca o linear Ax = y . (8.143) No caso em que m = n e A tem inversa, a solu ca o ( unica) e, naturalmente, x = A1 y . Nos demais casos uma solu ca o pode n ao estar presente ou n ao ser u nica. Podemos considerar o problema alternativo de saber para quais x Cn e a menor poss vel. Tais vetores x Cn seriam, no sentido da norma Euclidiana a norma Euclidiana Ax y Cm nimos quadrados, os melhores aproximantes ao que seria a solu ca o de (8.143). Um tal Cm , ou seja, em termos de m problema e por vezes dito ser um problema de optimiza c ao linear. Esse problema pode ser tratado com o uso da no ca o de pseudo-inversa de Moore-Penrose, a qual permite caracterizar precisamente o conjunto dos vetores x que minimizam Ax y Cm . A isso dedicaremos as linhas que seguem, sendo o principal resultado condensado no seguinte teorema:

JCABarata. Curso de F sica-Matem atica

Vers ao de 19 de junho de 2011.

Cap tulo 8

379/1943

Teorema 8.33 (Optimiza c ao Linear) Sejam A Mat (C, m, n) e y Cm dados. Ent ao, a cole c ao de todos vetores n nimo absoluto coincide com o conjunto de C para os quais a aplica c ao Cn x Ax y Cm [0, ) assume um m A+ y + Ker (A) = A+ y + n A+ A z, z Cn . (8.144)

interessante observar Esse conjunto e dito ser o conjunto minimizante do problema de optimiza c ao linear em quest ao. E que pela Proposi c ao 8.35, p agina 374, tem-se tamb em A+ y + Ker (A) = A+ y + Ran (A+ ) .

Como se v e do enunciado acima, a pseudo-inversa de Moore-Penrose fornece a melhor aproxima ca o em termos de m nimos quadrados ` a solu ca o de sistemas lineares. Observe-se que para os elementos x do conjunto minimizante (8.144) e nulo se e somente se y Ker (P2 ) = Ran (A) (pela Proposi ca o vale Ax y Cm = (AA+ m )y Cm = P2 y Cm , que 8.35, p agina 374), um fato um tanto obvio. Prova do Teorema 8.33. A imagem de A, Ran (A), e um subespa co linear fechado de Cm . O Teorema do Melhor Aproximante e o Teorema da Decomposi ca o Ortogonal (que neste texto s ao apresentados com toda generalidade no contexto de espa cos de Hilbert, como Cm na forma do Teorema 34.1, p agina 1684, e do Teorema 34.2, p agina 1686, respectivamente) garantem-nos a exist encia de um u nico y0 Ran (A) tal que y0 y Cm e m nimo, sendo que esse y0 satisfaz a propriedade de y0 y ser ortogonal a Ran (A).

e m nimo. Tal x0 n ao e necessariamente u nico e, como e Assim, existe ao menos um x0 Cn tal que Ax0 y Cm f acil ver, x1 Cn tem as mesmas propriedades se e somente se x0 x1 Ker (A) (j a que Ax0 = y0 e Ax1 = y0 , pela unicidade de y0 ). Como observamos, Ax0 y e ortogonal a Ran (A), ou seja, (Ax0 y ), Au C = 0 para todo u Cn . Isso signica que (A Ax0 A y ), u C = 0 para todo u Cn e, portanto, x0 satisfaz A Ax0 = A y .
(8.122)

(8.145)

Agora, a rela ca o (8.122) mostra-nos que x0 = A+ y satisfaz (8.145), pois A AA+ y = A y . Assim, conclu mos que nimo e composto por todos os vetores da o conjunto de todos x Cn que satisfazem a condi ca o de Ax y Cm ser m forma A+ y + x1 com x1 Ker (A). Pela Proposi ca o 8.35, p agina 374, x1 e da forma x1 = (n A+ A)z para algum n z C , completando a prova. Os exerc cios que seguem ilustram a aplica ca o da pseudo-inversa de Moore-Penrose no tratamento de problemas de optimiza ca o linear. E. 8.41 Exerc cio. Usando o Exerc cio E. 8.37, p agina 372, determine o conjunto dos melhores aproximantes x C3 ` a 1 0 i) e y = solu c ao da equa c ao linear Ax = y com A = ( 2 . Para tais vetores minimizantes x , calcule Ax y . C 2i 0 i 1 O exerc cio que segue envolve uma situa ca o menos trivial que a do exerc cio anterior, pois trata de um sistema linear sub-determinado e que n ao tem solu ca o. cio E. 8.38, p agina 372, determine o conjunto dos melhores aproximantes x C2 E. 8.42 Exerc cio. Usando o Exerc 1 1 2 ` a solu c ao da equa c ao linear Ax = y com A = 0 i e y = 3 . Para tais vetores minimizantes x, calcule Ax y C . Observe que nesse caso y Ran (A) e, portanto, o sistema Ax = y n ao tem solu c ao.
0 3 2

8.9.3

Exist encia e Decomposi c ao em Valores Singulares

Passemos agora a uma segunda demonstra ca o da exist encia da pseudo-inversa de Moore-Penrose de uma matriz A Mat (C, m, n) geral, fazendo uso aqui do Teorema da Decomposi ca o em Valores Singulares, Teorema 8.24, p agina 365. Trataremos primeiramente de matrizes quadradas para depois passarmos ao caso de matrizes n ao-quadradas. Determinando a pseudo-inversa de Moore-Penrose para matrizes quadradas

Come caremos pelas matrizes diagonais. Se D Mat (C, n) e uma matriz diagonal, a pseudo-inversa de Moore-Penrose

JCABarata. Curso de F sica-Matem atica

Vers ao de 19 de junho de 2011.

Cap tulo 8

380/1943

de D e dada pela matriz diagonal D+ Mat (C, n) cujos elementos diagonais s ao denidos para todo i = 1, . . . , n por 1 , se Dii = 0 , Dii + (D )ii = 0 , se Dii = 0 .

elementar vericar que DD+ D = D, D+ DD+ = D+ e que DD+ e D+ D s E ao auto-adjuntas. Em verdade, vale DD+ = D+ D que e uma matriz diagonal com elementos diagonais iguais a 0 ou a 1: 1 , se Dii = 0 , + + (DD )ii = (D D)ii = 0 , se Dii = 0 .

Passemos agora ` a quest ao da exist encia da pseudo-inversa de Moore-Penrose de uma matriz quadrada geral. Se A Mat (C, n) tem uma decomposi ca o em valores singulares A = V SW (vide Teorema 8.24, p agina 365), ent ao a pseudo-inversa de Moore-Penrose A+ de A e dada por A+ = W S + V . De fato, AA+ A = (V SW )(W S + V )(V SW ) = V SS + SW + = V SW = A e A+ AA+ = (W S + V )(V SW )(W S + V ) = W S + SS + V = W S + V = A+ . Al em disso, AA+ = (V SW )(W S + V ) = V (SS + )V e auto-adjunta, pois SS + e uma + matriz diagonal com elementos diagonais iguais a 0 ou a 1. Analogamente, A A = (W S + V )(V SW ) = W (S + S )W e auto-adjunta. Determinando a pseudo-inversa de Moore-Penrose para matrizes retangulares

Seja A Mat (C, m + n) a matriz quadrada (m + n) (m + n) denida em (8.7), p agina 297. Como A e uma matriz quadrada, estabelecemos acima que ela possui uma pseudo-inversa de Moore-Penrose (A )+ , u nica, satisfazendo 1. A (A )+ A = A , 2. (A )+ A (A )+ = (A )+ , 3. A (A )+ e (A )+ A s ao auto-adjuntas. No que segue demonstraremos que A+ Mat (C, n, m), a pseudo-inversa de Moore-Penrose de A Mat (C, m, n), e dada, seguindo as deni co es (8.3)(8.4), por A+ := (Jm+n, n )T (A )+ (Im, m+n )T , ou seja, A+ = (Jm+n, n )T (Im, m+n )T A(Jm+n, n )T
+

Consideraremos agora matrizes gerais (n ao necessariamente quadradas) A Mat (C, m, n).

(8.146)

(Im, m+n )T .

(8.147)

O ponto de partida e a exist encia da pseudo-inversa de A . A rela ca o A (A )+ A = A signica, usando a deni ca o (8.7), (Im, m+n )T A (Jm+n, n )T (A )+ (Im, m+n )T A(Jm+n, n )T = (Im, m+n )T A(Jm+n, n )T e das rela co es (8.5)(8.6) segue, multiplicando-se a ` esquerda por Im, m+n e ` a direita por Jm+n, n que AA+ A = A, uma das rela co es que desejamos provar. A rela ca o (A )+ A (A )+ = (A )+ signica, usando a deni ca o (8.7), (A )+ (Im, m+n )T A(Jm+n, n )T (A )+ = (A )+ . Multiplicando ` a esquerda por (Jm+n, n )T e ` a direita por (Im, m+n )T , isso estabelece a validade de A+ AA+ = A+ .

JCABarata. Curso de F sica-Matem atica

Vers ao de 19 de junho de 2011.

Cap tulo 8

381/1943

Como A (A )+ e auto-adjunta, segue da deni ca o a deni ca o (8.7), que (Im, m+n )T A(Jm+n, n )T (A )+ e auto-adjunta, ou seja, (Im, m+n )T A(Jm+n, n )T (A )+ = A(Jm+n, n )T (A )+ Im, m+n . Logo, multiplicando-se ` a esquerda por Im, m+n e ` a direita por (Im, m+n )T , segue de (8.5) que A(Jm+n, n )T (A )+ (Im, m+n )T = Im, m+n A(Jm+n, n )T (A )+ provando que AA+ e auto-adjunta. Por m, como (A )+ A e auto-adjunta, segue da deni ca o (8.7) que (A )+ (Im, m+n )T A(Jm+n, n )T e auto-adjunta, ou seja, (A )+ (Im, m+n )T A(Jm+n, n )T = Jm+n, n (A )+ (Im, m+n )T A . Logo, multiplicando-se ` a esquerda por (Jm+n, n )T e ` a direita por Jm+n, n , segue de (8.6) que (Jm+n, n )T (A )+ (Im, m+n )T A = estabelecendo que A+ A e auto-adjunta. Com isso estabelecemos que A+ dada em (8.146) e a pseudo-inversa de Moore-Penrose de A. (A )+ (Im, m+n )T A

T A(Jm+n, n )T (A )+ Im, m +n

Jm+n, n =

T + T Jm +n, n (A ) (Im, m+n ) A

8.10
8.10.1

Propriedades Especiais de Determinantes


Expans ao do Polin omio Caracter stico
n

m Seja A Mat (C, n) e seja pA () = det( A) = omio caracter stico. Desejamos m=0 cm , C, seu polin obter uma f ormula explicita para os coecientes cm em termos de determinantes de sub-matrizes de A (vide abaixo). Vamos designar por ak a k - esima coluna de A, de sorte que, pela nota ca o introduzida em (8.9), p agina 297, valha A = a1 , . . . , an . Recordando a deni ca o de base can onica fornecida em (8.10) e (8.11), p agina 297, ca claro que pA () = det e1 a1 , . . . , en an . Usando a propriedade de multilinearidade do determinante (linearidade em rela ca o a cada coluna), segue que n

pA () =

m=1

e a matriz obtida a partir da matriz A substituindo onde, para 1 j1 < < jm n, a1 , . . . , ej1 . . . , ejm . . . , an sua jl - esima coluna por ejl para cada l = 1, . . . , m. Note que no caso m = n, tem-se for cosamente jl = l para cada l = 1, . . . , n e a1 , . . . , ej1 . . . , ejm . . . , an = e1 , . . . , en = . Com isso, escrevemos
n1

(1)nm m

1j1 <<jm n

det a1 , . . . , ej1 . . . , ejm . . . , an + (1)n det(A) ,

pA () = n +

m=1

(1)nm m

1j1 <<jm n

det a1 , . . . , ej1 . . . , ejm . . . , an + (1)n det(A) .

Como cada vetor-coluna ejl cont em 1 na jl - esima linha, as demais linhas sendo nulas, as bem-conhecidas regras de c alculo de determinantes ensinam-nos que, para todo m = 1, . . . , n 1, det a1 , . . . , ej1 . . . , ejm . . . , an = det Aj1 , ..., jm ,

esimas Aj1 , ..., jm sendo a matriz de Mat (C, n m) (ou seja (n m) (n m)) obtida a partir de A eliminando-lhe as jl - linhas e colunas para todo l = 1, . . . , m. Assim, obtemos
n1

pA () = n +

m=1

onde e poss vel reconhecer os coecientes de pA ().

(1)nm m

1j1 <<jm n

det Aj1 , ..., jm + (1)n det(A) ,

(8.148)

JCABarata. Curso de F sica-Matem atica

Vers ao de 19 de junho de 2011.

Cap tulo 8

382/1943

Pelo Teorema de Hamilton-Cayley, Teorema 8.3, p agina 315, pA (A) = e, portanto,


n1

An +

m=1

Como comentamos em (8.40), p agina 318, se A for invers vel, obt em-se disso n1 1 (1)nm An1 + det Aj1 , ..., jm Am1 . A1 = (1)n+1 det(A) m=1
1j1 <<jm n

(1)nm

1j1 <<jm n

det Aj1 , ..., jm Am + (1)n det(A) = .

(8.149)

8.10.2

A Desigualdade de Hadamard

Vamos nesta se ca o demonstrar uma desigualdade para determinantes de matrizes, a qual e muito u til, a chamada desigualdade de Hadamard26 . Teorema 8.34 (Teorema do Determinante de Hadamard) Seja A Mat (C, n). Ent ao,
n n

| det(A)|

j =1 i=1

|Aij |2 ,

(8.150)

sendo Aij o elemento ij da matriz A. Segue disso que para toda matriz A Mat (C, n) vale
n

| det(A)| nn/2

max |Aij |
ij

(8.151)

O importante na estimativa (8.151) e o tipo de depend encia em n que se tem do lado direito. Ela ser a usada, por exemplo, em estimativas de converg encia da s erie de determinantes de Fredholm na Se ca o 17.2, p agina 789. Prova do Teorema 8.34. A prova de (8.151) e elementar, por (8.150). Passemos ` a prova de (8.150). Seja A Mat (C, n). Se A n ao tem inversa, ent ao det(A) = 0 e a desigualdade (8.150) e trivialmente satisfeita, n ao havendo o que se provar. Vamos ent ao supor que A tenha inversa. Seja A o conjunto de todas as matrizes M de Mat (C, n) com a propriedade que
n i=1 n

|Mij |2 =

i=1

|Aij |2

tamb para todo j = 1, . . . , n. Claro est a que A A. E em claro que A e um subconjunto compacto de Mat (C, n) (visto n2 aqui como C ). A fun ca o | det(M )| e cont nua como fun ca o de M e, portanto, assume ao menos um m aximo absoluto (n ao necessariamente u nico) em A, por este ser compacto (teorema de Weierstrass). Seja T A um desses m aximos. Note-se que | det(T )| | det(A)| > 0 e, portanto, T tem inversa.
n

Para todo i = 1, . . . , n vale por (8.20), p agina 300, que det(T ) =


j =1

Tij Cof(T )ij , onde Cof(T ), chamada de

matriz dos cofatores de T , foi denida no enunciado do Teorema 8.1, p agina 299. Seja xo esse i. Pela desigualdade de Cauchy-Schwarz, vale
n n n n
26 Jacques Salomon Hadamard (18651963). A refer encia ao trabalho de Hadamard e: J. Hadamard, R esolution dune question relativ aux d eterminants, Bull. Sci. Math. 28, 240-246 (1893).

| det(T )|2

j =1

|Tij |2

j =1

|Cof(T )ij |2 =

j =1

|Aij |2

j =1

|Cof(T )ij |2 .

(8.152)

JCABarata. Curso de F sica-Matem atica

Vers ao de 19 de junho de 2011.

Cap tulo 8

383/1943

Au ltima igualdade sendo devida ao fato que T A. Como e bem sabido, para o produto escalar a, b :=

n k=1

ak bk , a desigualdade de Cauchy-Schwarz | a, b | a

b e

uma igualdade se e somente se os vetores a e b forem proporcionais. Assim, tem-se a igualdade em (8.152) se e somente esima linha de T for proporcional ` a i- esima linha se existir i C tal que Tij = i Cof(T )ij para todo j , ou seja, se a i- de Cof(T ). O ponto importante agora e notar que se se tivermos a desigualdade estrita
n n

ent ao T n ao pode maximizar o m odulo de determinante entre as matrizes de A. De fato, considere a matriz T que e igual a ` matriz T , exceto sua i- esima linha, que e dada por
n j =1 n j =1

| det(T )|2 <

j =1

|Aij |2

j =1

|Cof(T )ij |2 ,

(8.153)

Tij

:=

|Aij |2

|Cof(T )ij |

1/2

Cof(T )ij ,

claro que j = 1 , . . . , n. E

n j =1 2 |Tij | =

n j =1

|Aij |2 ,

o que mostra que T A (para as demais linhas T coincide com T e n ao h a o que provar, pois T A). Fora isso,
n

det(T ) =
j =1

Tij Cof(T )ij , pois Cof(T )ij = Cof(T )ij , j a que T e T s o diferem na i- esima linha. Assim,

e conclu mos por (8.153) que ter amos | det(T )| < det(T ), contrariando a hip otese que | det(T )| e m aximo. Assim, devemos ter a igualdade em (8.152) e, pelos coment arios de acima, isso implica que existe i C tal que Tij = i Cof(T )ij e proporcional ` a i- esima linha de Cof(T ). Como i e arbitr ario, isso vale para para todo j , ou seja, a i- esima linha de T todo i. Agora, como as linhas de T s ao proporcionais a `s de Cof(T ), segue que
n

det(T ) =

n j =1

|Aij |2

j =1

|Cof(T )ij |

1/2

n j =1

|Cof(T )ij |2 =

n j =1

|Aij |2

1/2

n j =1

|Cof(T )ij |2

1/2

det(T ) =
j =1

Tij Cof(T )ij =

1 i

n j =1

|Tij |2 , =

1 i

n j =1

|Aij |2

e pela multilinearidade do determinante, que det(T ) = det(T ) = 1 n det(Cof(T )) . Dessas duas rela co es extra mos det(T )
n n n n n

1 = 1 n

i=1 j =1

|Aij | =

det(Cof(T )) det(T )

i=1 j =1

|Aij |2 .

JCABarata. Curso de F sica-Matem atica

Vers ao de 19 de junho de 2011.

Cap tulo 8

384/1943

Como a rela ca o (8.25) vale para qualquer matriz invers vel, tem-se det(Cof(T )) = det(T )n1 e, portanto, | det(T )|2 =
n

i=1 j =1

|Aij |2 . Por constru ca o, T maximiza | det(T )| em A. Como A A, segue que


n n

| det(A)| Isso prova o teorema.

i=1 j =1

|Aij |2 .

(8.154)

JCABarata. Curso de F sica-Matem atica

Vers ao de 19 de junho de 2011.

Cap tulo 8

385/1943

8.11

Exerc cios Adicionais

E. 8.43 Exerc cio. a) Determine o polin omio caracter stico da matriz

b) Verique explicitamente a validade do Teorema de Hamilton-Cayley para a matriz A. c) Usando o Teorema de Hamilton-Cayley calcule A1 .

2 7 5 A = 0 2 3 i 5 i . 0 0 1 4i

E. 8.44 Exerc cio. Repita o exerc cio anterior para as matrizes A1 2 = 0 0 1 4 + i 0 i , 2 7i 3

0 0 5 A2 = 8 0 . 8 3i 1 9i 4 5i

E. 8.45 Exerc cio. Considere em Cn o seguinte produto escalar


n

u, v

=
a=1

ua va pa ,

onde pa > 0 para a = 1, . . . , n. Seja uma matriz A, com elementos de matriz Aij . Mostre que, com o produto escalar , p e dado por o elemento de matriz (Ap )ij da adjunta Ap da matriz A pj (Ap )ij = Aji . (8.155) pi e denida de sorte que u, Av (Lembre-se que Ap
p

= Ap u, v

Mostre que para quaisquer u, v Cn vale u, v p = u, P v C , onde u, v C = n e o produto escalar usual em a=1 ua va 1 n p e a adjunta usual de A em rela c ao ao produto C e P = diag (p1 , . . . , pn ). Conclua disso que A = P A P , onde A escalar , C : (A )ij = Aji . 4 i/2 . Essa matriz n ao e auto-adjunta em rela c ao ao E. 8.46 Exerc cio. Determine os autovalores da matriz A = 2i 5

Para a matriz adjunta denida em (8.155), verique a validade das regras (Ap )p = A e (AB )p = B p Ap , para quaisquer matrizes A, B Mat (C, n). Calcule p .

para todos u, v Cn ).

produto escalar usual em C2 , mas possui autovalores reais. Justique esse fato mostrando, pelos exerc cios anteriores, que A e auto-adjunta em rela c ao ao produto escalar u, v p = 2u1 v1 + u2 v2 /2. Mostre a adjunta Ap em rela c ao a esse produto 4 i/2 = A e constate explicitamente que u, Av escalar e Ap = 2i 5
p

= Au, v

para todos u, v C2 . Determine os

autovetores de A e constate que os mesmos s ao ortogonais em rela c ao ao produto escalar , p .

JCABarata. Curso de F sica-Matem atica

Vers ao de 19 de junho de 2011.

Cap tulo 8

386/1943

O exerc cio que segue generaliza o Exerc cio E. 8.45. E. 8.47 Exerc cio. Seja um produto escalar em Cn . Pela Proposi c ao 3.5, p agina 197, existe uma unica matriz M Mat (C, n) auto-adjunta e de autovalores positivos (e, portanto, invers vel) tal que (x, y ) = x, M y C para todos x, y Cn . c ao ao produto escalar : (x, Ay ) = (A x, y ) Seja A Mat (C, n) e seja A Mat (C, n) sua adjunta em rela n 1 para todos x, y C . Mostre que A = M A M , onde A e a adjunta usual de A em rela c ao ao produto escalar , C . Mostre que para quaisquer matrizes A, B Mat (C, n) valem (A ) = A e (AB ) = B A . Calcule .

JCABarata. Curso de F sica-Matem atica

Vers ao de 19 de junho de 2011.

Cap tulo 8

387/1943

1 1

0
1
a

0 2 2
1

0
0

0
3

0
3
c

0
4

0
4
d

Figura 8.5: Forma can onica de uma matriz com 4 autovalores distintos 1 , 2 , 3 e 4 . Os s assumem apenas os valores 0 ou 1, de acordo com as regras explicadas acima. Todos os elementos fora da diagonal principal e da primeira supradiagonal s ao nulos. As setas indicam zeros que ocorrem na primeira supradiagonal nos pontos onde ocorre transi ca o entre os blocos, conseq u encia do fato de esses elementos estarem fora dos blocos.

Вам также может понравиться